Está en la página 1de 158

1.

Paciente mujer de 38 años de edad, sin antecedentes de importancia, que luego de


discusión familiar, presenta de manera brusca, cefalea muy intensa y persistente. Lo
relevante en el examen físico son los signos meníngeos presentes, pero no tiene
déficit neurológico, ¿Cuál de los siguientes hallazgos NO espera encontrar en las
primeras 8 horas?

A Alteraciones tomográficas compatibles con infarto encefálico.


B Alteraciones tomográficas compatibles con sangrado subaracnoideo.
C Presencia de hematíes crenados en el líquido cefalorraquídeo.
D Alteraciones del segmento ST y de la onda T del electrocardiograma.

2. Se trata de un paciente varón de 66 años, fumador crónico e hipertenso. Acude al


servicio de emergencias porque presenta de manera súbita, la pérdida de
movimientos y de la sensibilidad en las extremidades superior e inferior del lado
derecho y desviación de la mirada hacia el lado izquierdo. Al examen físico es más
prominente la debilidad y espasticidad en el brazo, además se encuentra afásico,
¿Cuál de las siguientes arterias está con mayor probabilidad involucrada?

A Cerebral posterior.
B Cerebral anterior.
C Cerebral media.
D Vertebro basilar.

3. Señale el esquema de tratamiento de la crisis asmática:

Juan tiene 72 años y es llevado al servicio de emergencia, porque presentó tos con
flema hace 1 semana, dificultad para respirar desde hace 4 días y alteración del estado
de conciencia hace 24 horas. Su médico de cabecera recetó amoxicilina y envío de
exámenes de laboratorio con Urea de 89 mg/dl, glucosa 110 mg/dl y EMO normal. Al
examen físico FC 64, FR 33, TA 110/50. Pulmones con crepitantes bilaterales en bases y
escasas sibilancias en base de pulmón derecho. ¿Cómo manejaría a este paciente,
según la escala de gravedad CURB65?

El paciente deberá ser ingresado al servicio de neumología para recibir antibiótico intravenoso.
El paciente debe ser manejado ambulatoriamente con un solo antibiótico.
El paciente es de bajo riesgo por lo que se envía al domicilio con familiares para que continúe
con el tratamiento de amoxicilina indicado
El paciente tiene una neumonía grave y debe ser ingresado a la unidad de cuidado
intensivo para su manejo.

4. Señale la prueba habitual de valoración de control glucémico para un paciente


diabético en el que se requiere conocer la glucemia de los últimos 3 meses:
A Glusosa plasmática capilar.
B Indice glucémico.
C Hemoglobina glicosilada.
D Fructosamina.

5. Uno de los principales objetivos del manejo nutricional y el soporte de ejercicio


moderado en pacientes con DM gestacional es:

A Controlar la cetonuria.
B Reducir la glucemia posprandial.
C Optimizar el control glicémico.
D Controlar la hemoglobina glicosilada.

6. Paciente asintomático, que al examen físico presenta un aumento del tiroides grado
II, sin nódulos palpables. Los exámenes reportan: TSH y T3 normales, T4 total baja,
colesterol alto, anticuerpos contra TPO negativos, la gamagrafía tiroidea pone de
manifiesto un aumento en la captación. Señale el diagnóstico:

A Bocio multinodular tóxico.


B Bocio difuso no tóxico.
C Tiroiditis de Hashimoto.
D Bocio multinodular no tóxico.

7. Una paciente de 71 años de edad. Tiene antecedentes de enfermedad coronaria y


elevación de sus lípidos en sangre. Acude por presentar cansancio, intolerancia al frío,
hipersomnia, caída del cabello y sequedad de piel. Los signos vitales son normales. En los
exámenes se confirman niveles de TSH de 12 mUI/L (3 veces el valor normal máximo),
hallados en un examen previo, con niveles de T4 dentro de rangos normales, pero en límites
bajos, ¿Cuál es la dosis diaria inicial de levotiroxina recomendada para tratar a su paciente?

A. 12.5 - 25 gramos

8. Una mujer de 47 años de edad, sedentaria, tiene un índice de masa corporal de 45 y


solicita su consejo médico, ¿Cuál de las siguientes complicaciones NO es frecuente en este
tipo de pacientes?

A Componente respiratorio obstructivo.


B Episodios de colecistitis acalculosa.
C Incremento de cáncer de vesícula.
D Componente respiratorio restrictivo.

9. Se trata de un joven de 16 años previamente sano. Pocos días atrás estuvo en contacto
con un niño con varicela. Acude porque desde hace unas 12 horas presenta febrícula,
malestar general y cefalea, y aparecieron lesiones dérmicas sugestivas de varicela. En estas
condiciones, de entre los siguientes fármacos, ¿Cuál es desaconsejado utilizar?

A Paracetamol.
B Aspirina.
C Aciclovir.
D Valaciclovir.

10. .Un varón soltero, de 26 años con antecedente de una relación sexual sin protección
hace 15 días, acude a su consulta por una lesión genital. Inició hace unos 6 días como una
mácula en el prepucio que progresó a pápula, con posterior salida de material purulento y
dolor. Al examen físico se encuentra una úlcera redondeada, algo deprimida, de
aproximadamente un centímetro de diámetro, de bordes difusos y suaves, irregulares y piel
eritematosa en su contorno. Se evidencia un exudado purulento, gris, abundante, en el
fondo de la úlcera. Además presenta un ganglio inguinal derecho palpable y doloroso, de 1.5
centímetros de diámetro. ¿Para cuál de los siguientes microorganismos direccionaría la
terapéutica.

A Treponema pálido.
B Haemophylus ducreyi.
C Chlamydia trachomatis.
D Herpes virus.

11. En un niño con diarrea aguda viral y que presenta manifestaciones de deshidratación
moderada, se deberá administrar, en las primeras 4 horas, la solución de rehidratación oral
de la OMS, a la dosis de:

A 75 ml/kg.
B 90 ml/kg.
C 200 ml/kg.
D 30ml/kg.

12. Un recién nacido a término de 8 días de vida, es traído a emergencia por presentar mala
succión, quejido y vómito en dos ocasiones. Tiene como antecedentes perinatales: parto
céfalo-vaginal, líquido amniótico claro, ruptura de membranas de 26 horas y un Apgar de
7/9. En el examen físico se lo encuentra pálido, vaso-contraído, con pobre respuesta a los
estímulos e hipoactivo. Temperatura 37 °C, frecuencia cardiaca 180 lpm (latidos por minuto),
frecuencia respiratoria 70 rpm (respiraciones por minuto), llenado capilar mayor a 2
segundos, saturación de oxígeno 83%; retracciones subcostales, abdomen suave, ruidos
hidroaéreos normales, ¿Cuál es el diagnóstico más probable en este caso?

A Enterocolitis necrotizante.
B Asfixia perinatal.
C Sepsis tardía.
D Taquipnea transitoria.

13. Una niña de 12 meses de edad presenta desde hace 2 días rinorrea serosa, tos no
productiva y febrícula. Al examen físico se encuentra febril, con secreción ocular bilateral,
congestión nasal y eritema faríngeo, ¿Cuál es el tratamiento más adecuado en este caso?

A Loratadina.
B Amoxicilina.
C Acetaminofén.
D Ambroxol.

14. Paciente de 25 años llega a la consulta porque después de un legrado por aborto
espontáneo hace 4 meses no ha menstruado. ¿Cuál es el diagnóstico?

A Síndrome de Sheehan.
B Endometriosis.
C Síndrome de Asherman.

15. En un sangrado uterino anormal de tipo anatómico, usamos la ecografía con infusión de
solución salina como método diagnóstico. ¿En qué caso está contraindicado su uso?

A Infección pélvica.
B Pólipos endometriales.
C Diabetes.
D Leiomiomas submucosos.
16. Llega un paciente con una herida penetrante por arma de fuego en el flanco derecho, se
acompaña de hipotensión y es evidente una evisceración intestinal. ¿Cuál es el manejo
apropiado en este paciente?

A Laparotomía exploratoria.
B Tomografía axial computarizada abdominal.
C Cierre de herida.
D Ecografía Focalizada en Trauma (FA
17. En revisión secundaria, indique las lesiones potencialmente fatales en trauma de tórax.
1. Hemotórax masivo. 2. Neumotoráx simple. 3. Lesión cardiaca cerrada. 4. Taponamiento
cardiaco. 5. Lesión cerrada de esófago.

A 2,3,5.
B 1,2,3.
C 1,3,5.
D 2,3,4.

18. Seleccione los parámetros radiográficos evaluados para diagnosticar displasia del
desarrollo de cadera. 1. Acetabular. 2. Barlow. 3. Perkins. 4. Hilgenreiner. 5. Shenton. 6.
Ortolani.

A 2, 3, 5, 6.
B 1, 3, 5, 6.
C 1, 3, 4, 5. APHS
D 1, 2, 4, 5.

20. En un paciente con riesgo suicida. ¿Cuál de los siguientes enunciados es un criterio
para hospitalización inmediata?

A El alto grado de letalidad del intento suicida.


B Tener una sólida red de apoyo psicosocial.
C No tener ninguna comorbilidad.
D Sexo femenino.

21. Relacione las causas de bocio endémico con sus características: Causas
Características 1. Alteraciones del metabolismo del yodo. 2. Bociógenos. a)
Interfieren en la captación del yodo. b) Aclaramiento renal de yodo aumentado. c)
Déficit en el aporte de yodo. d) Interfieren en la producción hormonal.

A 1ac, 2bd.
B 1ab, 2cd.
C 1bd, 2ac.
D 1bc, 2ad.

22. Todos son cambios en el estilo de vida de un paciente con Diabetes Mellitus tipo 2.
EXCEPTO:

A Se sugiere una disminución del 7 % de grasas en la dieta.


B Reducción de peso entre el 5 y 10 % en pacientes con DM2 con sobrepeso u obesidad.
C Fraccionar el total de la alimentación habitual del día en 5 o 6 porciones.
D Mantener una dieta fraccionada con tres ingestas diarias.

23. Se trata a un paciente adulto que padece de un proceso infeccioso de tejidos


blandos y usted debe conocer los criterios de sepsis severa para que, en caso de
presentarse, referirlo de manera oportuna para tratamiento de especialidad en un
nivel de mayor complejidad, ¿Cuál de los siguientes es un criterio de sepsis severa?

A Diuresis de 0,1 mL/Kg/h.


B Lactato menor a 1 mmol/l.
C Plaquetas de 150.000/mm3.
D Alcalosis metabólica.

24. Una paciente de 18 años de edad sin antecedentes clínicos de importancia, presenta
malestar general, decaimiento, tos y fiebre de 2 días de evolución. Al examen físico
tiene presión arterial: 110/70 mmHg, frecuencia cardiaca: 100 lpm (latidos por
minuto), frecuencia respiratoria: 32 rpm (respiraciones por minuto), temperatura
bucal de 39 OC, se ausculta broncofonía y pectoriloquia derecha unilateral. Usted
decide ingresarla y tratar con antibióticos, la paciente se ha mantenido más estable.
Al segundo día de tratamiento, presenta de manera brusca dolor torácico, disnea,
presión arterial: 70/40 mmHg, frecuencia cardiaca: 150 lpm (latidos por minuto),
frecuencia respiratoria: 44 rpm (respiraciones por minuto), a la percusión tiene
hiperresonancia y ausencia de ruidos pulmonares en el lado derecho. ¿Cuál es la
presunción diagnóstica más probable?

A Hemotórax.
B Derrame paraneumónico.
C Neumotórax a tensión.
D Neumotórax simple

25. Un joven de 18 años de edad, sin antecedentes patológicos, realiza intensa actividad
de físico culturismo. Acude a su consulta por anorexia, náusea, vómito, malestar
general e ictericia. Los signos vitales son normales y al examen físico lo llamativo, a
más de su hipertrofia muscular global, es que tiene ictericia generalizada. En los
exámenes de sangre destaca lo siguiente: bilirrubina total 15 mg/dL, directa 14
mg/dL, TGO y TGP elevadas al doble de lo normal y la fosfatasa alcalina está elevada
1,5 veces su valor normal. En este paciente, ¿Cuál es la etiología más probable de la
ictericia?

A Hepatitis autoinmune.
B Litiasis del colédoco.
C Hepatitis viral aguda.
D Consumo de esteroides.

26. Se trata de una paciente de 38 años de edad con enfermedad ácido péptica
sintomática, recurrente, sin sangrado digestivo. Tiene biopsia positiva para
Helicobacter pylori. De entre las siguientes alternativas, ¿Cuál es la más adecuada?

A Lansoprazol, subsalicilato de bismuto metronidazol y amoxicilina.


B Omeprazol, subsalicilato de bismuto metronidazol y clindamicina.
C Lansoprazol, amoxicilina, metronidazol y tetraciclina.
D Omeprazol, subsalicilato de bismuto metronidazol y tetraciclina.

27. En un paciente con antecedentes de reflujo gastro esofágico severo de larga


evolución existen varias posibilidades de complicaciones, ¿Cuál de las siguientes,
NO es una complicación del reflujo gastro esofágico?

A Arritmias.
B Tos crónica.
C Estreñimiento.
D Broncoespasmo.

28. Mujer de 18 años llega a la consulta por falta de menstruación. La anamnesis revela
un desarrollo de mamas normal pero poco vello púbico y axilar. Al examen físico las
mamas están en Tanner 4 y el vello púbico están en Tanner 1. ¿Cuál es el
diagnóstico más probable?

A Insensibilidad completa a los andrógenos.


B Síndrome de Kallman.
C Malformación de conductos de Müller.
D Retraso fisiológico.

29. La velocidad de ganancia de peso, promedio, por semana (kg / sem), en una
embarazada durante el segundo trimestre con un IMC pregestacional normal es de:

A 0.51Kg / sem.
B 0.42 Kg / sem.
C 0.28 Kg / sem.
D 0.22 g / sem.
30. En el embarazo los cambios fisiológicos como sensación de ardor estomacal y la
pirosis se atribuyen a:

A Mayor velocidad de vaciamiento gástrico.


B Cambio de posición del estómago en el primer trimestre.
C Presiones intraesofágicas más altas e intragástricas más bajas.
D Hipotonía del esfínter esofágico inferior.

31. Las siguientes son indicaciones de laparotomía en pacientes con lesión penetrante
de abdomen. EXCEPTO:

A Signos de irritación peritoneal.


B Alteración hemodinámica.
C Signos de penetración de la fascia.
D Distensión abdominal.

32. Causa de Abdomen agudo no quirúrgico, EXCEPTO:

A Peritonitis primaria.
B Pancreatitis aguda.
C Plastrón Apendicular con SIRS.
D Crisis Cetodiabética

33. Varón de 58 años de edad, acude por presentar un dolor a nivel inguinal izquierdo
que según relata se ha presentado hace pocos días luego de esfuerzo físico
realizado en la mudanza de casa. Su médico diagnostica una hernia inguinal grado II,
y recomienda corrección quirúrgica, la cual se efectúa en la semana siguiente. En
exámenes de laboratorio efectuados previa a la cirugía, destaca glicemia en ayunas,
de 118 mg. El acto quirúrgico se lleva a cabo sin complicaciones. En el
postoperatorio inmediato ¿Qué conducta sería la más adecuada respecto al empleo
de antibióticos?

A No está indicada profilaxis antibiótica post operatoria.


B Amoxicilina más Acido Clavulánico, 2 g. I.V. cada 8 horas.
C Cefazolina 1 gramo por vía intravenosa, dosis única.
D Continuar con el antibiótico indicado en el preoperato
34. Mujer de 50 años que presenta desde hace 12 horas dolor tipo cólico en cuadrante
superior derecho, 30 minutos después de ingesta de alimentos grasos. El dolor se
irradia hacia la región subescapular derecha. Se acompaña de síntomas
neurovegetativos. Desde hace 8 horas se nota coloración amarillenta de ojos y
sensación de alza térmica y escalofríos. Al examen físico presenta: FR 16/min, FC
90/min, TA 120/80 mmHg, T 39.3° C. Facies: álgica, sudorosa, fría, con palidez
generalizada. Ojos: escleras ictéricas. Abdomen: doloroso a la palpación superficial
y profunda en epigastrio e hipocondrio derecho. Exámenes: leucocitos 18000/µL,
neutrófilos 80%, bilirrubina total 8 mg/dL, bilirrubina directa 6.5 mg/dL, bilirrubina
indirecta 1.5 mg/dL, AST 248 UI, ALT 285 UI, fosfatasa alcalina 280 UI/dL, amilasa de
440 U/dL. El reporte de ecosonografía indica: vesícula con múltiples litos en su
interior y de diferente tamaño, paredes de 3 mm de espesor; colédoco de 16 mm de
diámetro y no se visualiza cálculos. De acuerdo al cuadro clínico, indique el
diagnóstico y el tratamiento más acertados.

Colecistitis aguda más colangitis aguda; terapia hidroelectrolítica, antibióticos, colecistectomía


A
con exploración de vías biliares.
Coledocolitiasis más colangitis aguda; antibióticos y colecistectomía con exploración de vías
B
biliares.
C Colecistitis crónica reagudizada; terapia hidroelectrolítica, antibióticos y colecistectomía.
Coledocolitiasis más colangitis aguda; terapia hidroelectrolítica, antibióticos y colecistectomía
D
con exploración de vías biliares.

35. Varón de 47 años de edad con antecedentes de miopía, dislipidemia y bloqueo A/V
de primer grado, acude con dolor ocular derecho, importante, que se irradia a sector
temporal homolateral. Además hay la presencia de hiperemia conjuntival, epífora y
edema palpebral, ¿Cuál sería el tratamiento de primera elección para este paciente?

A Betabloqueadores no selectivos.
B Pilocarpina al 2%.
C Anhidrasa carbónica tópica.
D Análogos de las prostaglandinas

36. A un paciente con un primer episodio psicótico, ¿qué se aplica una vez instaurado el
tratamiento farmacológico y de apoyo terapéutico?

A Tratamiento en hospital del día.


B Tratamiento de desintoxicación.
C Plan de actividad física y nutricional.
D Internación psiquiátrica forzosa.
37. Paciente masculino de 40 años, con antecedente de alcoholismo, el cual presenta
fiebre, tinte ictérico, taquipnea, con eritema palmar, temblor en manos, con manos
en puño y lenguaje agresivo y soez. ¿Què fármaco se debe suministrar?

A Haloperidol 5mg, vía Intramuscular, repitiendo cada 30 minutos, dosis máxima 20 mg/día.
B Paracetamol 1 gr Intravenoso STAT, luego cada 8 horas.
C Midazolam 5 mg intravenosos, repitiendo la dosis a los 30 minutos hasta suministrar 15 mg/día.
D Quetiapina 25 mg, vía oral, repitiendo la dosis a los 30 minutos, dosis máxima 50 mg/día

38. Señale en qué circunstancia la firma de aceptación del consentimiento informado


debe ser notarizada:

A Procedimientos diagnósticos.
B Transplante de órganos.
C Terapias nuevas de baja complejidad.
D Intervenciones quirúrgicas.

39. Señale el patógeno que causa neumonía extrahospitalaria (adquirida en la


comunidad) en pacientes con EPOC y tabaquismo:

A Chlamydia psittaci.
B Stafilococo pneumoniae.
C Haemophilus influenzae.
D Coxiella burnetii.

40. La celulitis en diabéticos ancianos es ocasionada por:

A Haemophilus influenzae.
B Pasteurella multocida.
C Estafilococo.
D Streptococcus agalactiae.

41. Indique el tratamiento que debe darse a una paciente que presenta en la primera
consulta, fiebre de 38 °C, dolor abdominal bajo, que se acompaña de disuria,
polaquiuria y dispareunia, al examen físico se encuentra: secreción vaginal
amarillenta abundante; al examen especular se evidencia cervix eritematoso que
sangra facilmente a la manipulación, no se encontraron adenomegalias inguinales,
vesículas, o úlceras genitales.

A Ciprofloxacina 500 mg (DU); Azitromicina 1g (DU).


B Azitromicina 1g (DU
C P Benzatina 2.400.000 UI (IM
D Ceftriaxona 250 mg IM (D
42. Cuál de las siguientes situaciones determina un resultado falso positivo en las
pruebas de tamizaje para VIH?

A Interferencia de factores reumatoideos.


B Disfunciones de las células B.
C Vacunación contra la gripe.
D Trasplante de médula ósea.

43. Una mujer soltera de 25 años de edad, acude al servicio de emergencia por presentar
desde hace aproximadamente 24 horas, dolor de cabeza de gran intensidad,
continuo, tipo contricción, localizado en región parietal y occipital, que se
incrementa con la palpación y con cambios bruscos de posición. Como
antecedentes refiere cefaleas frecuentes, de menor intensidad que la actual, por lo
que se auto medica ibuprofeno 400 mg, hasta tres veces al día. Al examen físico,
presión arterial 100/60 mmHg, frecuencia cardiaca 72 lpm (latidos por minuto), no
hay evidencia de patología en ojos, oídos, o senos de la cara. Dolor cervical a la
dígito presión. De acuerdo a estos datos ¿Cuál sería la causa más probable de este
dolor?

A Dolor por contractura muscular crónica.


B Hemorragia subaracnoidea.
C Encefalitis viral aguda.
D Migraña típica en paciente joven.

44. Una mujer de 42 años de edad, desde hace 2 días luego de ingerir alimentos,
presenta dolor epigástrico moderado, que se ha intensificado de manera paulatina,
es continuo y se irradia hacia la espalda. Se acompaña de esporádico vómito de
contenido alimentario que no calma el dolor. Al examen físico, al tomar la presión
arterial, se provoca la contractura espástica de la musculatura extensora y aductora
de los dedos de la mano correspondiente. La amilasa se encuentra elevada 7 veces y
la lipasa 3 veces, con respecto a sus valores normales. El reporte ecográfico indica
páncreas incrementado de volumen e hipoecogénico. ¿Qué hallazgo de laboratorio
podría explicar la contractura espástica descrita?

A Hipocalcemia.
B Hiponatremia.
C Hipocloremia.
D Hipopotasemia.
45. Un paciente varón de 55 años, con sobrepeso, presenta polidipsia y poliuria. En este
caso, ¿Cuál de los siguientes resultados en sus pruebas de sangre confirmarían el
diagnóstico de diabetes mellitus?

A Cifra de hemoglobina glicosilada A1c de 5,6%.


B Test de tolerancia a la glucosa de 142mg/dL.
C Nivel de glicemia tomada al azar de 210 mg/dL.
D Dos valores de glicemia en ayunas de 112 mg/dL.

46. Un recién nacido a término de 18 horas de vida, presenta ictericia en zona I y


hepatoesple-nomegalia. Los exámenes complementarios arrojan los siguientes
resultados: Hematocrito 40%, Bilirrubina Total 7 mg/dl, Bilirrubina directa 0.5 mg/dl,
reticulocitos 10%, microesfe-rocitos en el frotis de sangre periférica, ¿Cuál sería la
conducta más adecuada en este caso?

A Realizar ecografía hepática.


B Indicar fototerapia.
C Suspender lactancia materna.
D Exanguineotransfusión.

47. La presencia de los siguientes elementos en un niño de 4 años de edad, sugiere


maltrato infantil. EXCEPTO:

A Niño o niña con manifiesta timidez.


B Antecedentes de fracturas repetidas.
C Numerosas cicatrices en la piel.
D Estreñimiento a petición.

48. Un niño de 3 meses presenta vómito y diarrea desde hace 3 días, recientemente
cambia de lactancia materna a fórmula láctea. Se nota marcadamente irritable, con
una frecuencia cardíaca de 170/ min, respiraciones de 50/min, temperatura de 37.6°C
y presión arterial 60/30 mmHg. La fontanela anterior está marcadamente deprimida y
mucosas orales secas. La piel menos turgente, vuelve lentamente a su estado
normal. Llenado capilar a los 4 segundos. Electrolitos séricos: sodio 165 mEq/l;
potasio 5.3 mEq/l; cloro 148 mEq/l y bicarbonato 8 mEq/l. ¿Cuál es la estrategia
indicada para el manejo de este niño?

Bolo intravenoso de solución salina al 0.9% seguido de rehidratación calculada para 48


A
horas
B Rehidratación calculada para 48 horas, sin bolo intravenoso inicial
C Bolo intravenosos de dextrosa al 5% seguido de rehidratación calculada para 24 horas
D Bolo intravenoso de lactato de Ringer seguido de rehidratación calculada para 24 horas

49. Un niño de 8 años de edad presenta crisis convulsivas, ¿Cuál de los siguientes es
un criterio para diagnosticar estatus epiléptico?

A Duración mayor a 30 minutos.


B Examen neurológico postictal anormal.
C Ausencia de fiebre durante la crisis.
D Crisis tónico-clónico generalizadas.

50. El mecanismo correcto de acción de los anticonceptivos a base de progesterona es:

A Provocan un espesamiento del moco cervical.


B Evitan la ovulación al suprimir liberación de FSH.
C Aumentan la motilidad de las trompas de Falopio
D Generan un endometrio secretor.

51. Paciente en tercer día post parto, que presenta temperatura de 38.4 °C y malestar
general. Los otros signos vitales son normales. Tiene abdomen suave, fondo uterino
doloroso, a nivel de ombligo. Loquios asalmonados de mal olor, ¿Cuál de los
siguientes gérmenes está comúnmente asociado a la endometritis post parto?

A Estreptococo grupo A.
B Escherichia Coli.
C Bacteroides species.
D Chlamydia Trachomatis.

52. Una mujer de 60 años de edad. Tiene antecedentes de hematoquecia por


diverticulosis colónica. El día de hoy se repite un episodio de sangrado indoloro, los
signos vitales son estables. En este contexto, ¿Cuál de las siguientes medidas NO
está indicada?

A Tomografía intestinal contrastada.


B Estudios de imagen con isótopos.
C Antibióticos por vía intravenosa.
D Realizar colonoscopia terapéutica.

53. Señale la afirmación correcta relacionada a los signos y síntomas de la apendicitis


aguda:
A La falta de apetito temporal es un síntoma poco frecuente en cuadros apendiculares.
El signo de Blumberg es positivo cuando hay dolor en fosa ilíaca izquierda al comprimir fosa
B
ilíaca derecha.
El dolor durante el examen rectal es característico en los apéndices inflamados de localización
C
paracecal ascendente.
El signo de Rovsing es el dolor que se presenta en el cuadrante inferior derecho al
D
ejercer presión en el cuadrante inferior izquierdo.

54. Mujer de 45 años, acude a emergencias por presentar desde hace dos días fiebre de
38.5° C y dolor en hipocondrio derecho, al inicio tipo cólico, ahora constante, que se
irradia a región torácica derecha posterior. También refiere náusea y vómito. La
causa aparente es la ingesta de comida abundante que incluyó carne de chancho y
papas con mayonesa. El examen físico revela un índice de masa corporal de 28,5,
temperatura de 38,7 °C, frecuencia cardíaca 110 por minuto. Ruidos hidroaéreos
presentes pero disminuidos. Hemiabdomen superior doloroso, especialmente bajo
borde costal derecho a la inspiración profunda. ¿La mejor combinación de
exámenes que usted podría utilizar para precisar el diagnóstico, es?

A Determinación de amilasa y lipasa pancreáticas.


B Endoscopia digestiva alta y biopsia gastroduodenal.
C Solicitar ecografía abdominal y biometría hemática.
D Resonancia magnética y colangio-pancreatografía retrógrada endoscópi

55. Indique la respuesta correcta. Una ama de casa de 57 años de edad, sin
antecedentes patológicos previos, con un IMC de 29, acude a su consulta porque
tiene el deseo de dejar de fumar. Ella refiere que ha tratado por todos los métodos,
ha recibido ayuda psicológica en reiteradas ocasiones, pero su dependencia física al
cigarrillo le han hecho imposible dejarlo definitivamente. Usted dispone de algunas
estrategias terapéuticas. ¿Cuál de las siguientes sería la más indicada en este tipo
de pacientes?

A Chicle de nicotina.
B Cigarrillo electrónico.
C Vareniclina oral.
D Terapia de reemplazo de nicotina más soporte de grupos de apoyo.

56. Señale qué principio de la bioética permite al paciente rechazar un tratamiento:

A No maledicencia.
B Justicia.
C Autodeterminación.
D Beneficencia.

57. Paciente femenina con enfermedad catastrófica, solicita a su médico le ayude a


terminar con su vida, este le administra una sustancia específica a sabiendas que
causará la muerte de la paciente. Según el Código Integral Penal del Ecuador. ¿Qué
tipo de delito comete el médico?

A Femicidio.
B Sicariato.
C Asesinato.
D Abandono de personal.

58. Las siguientes sintomatologías se asocian a la encefalopatía metabólica, EXCEPTO:

A Convulsiones.
B Trastornos psíquicos.
C Trastornos de la conciencia.
D Hipertensión intracraneal.

59. Una paciente de 29 años de edad, sin antecedentes patológicos de importancia.


Desde hace 5 días presenta fiebre, malestar general, decaimiento, tos no productiva
y dolor en punta de costado del lado derecho. Acude a consulta y al examen físico
tiene presión arterial de 120/80 mm Hg, frecuencia cardiaca de 98 latidos por minuto,
frecuencia respiratoria de 22 respiraciones por minuto; temperatura bucal 39 0C.
Tiene incremento del frémito y broncofonía en región subescapular derecha, el
hemitórax izquierdo es normal, ruidos cardiacos normales, abdomen normal. En la
radiografía de tórax se aprecia imagen radio opaca con broncograma aéreo limitado
al lóbulo inferior derecho. La oximetría de pulso de 91% al aire ambiente. En la
biometría hemática se encuentran 8.500 leucocitos/mm3 con 70% de segmentados y
30% de linfocitos. La conducta más apropiada será:

A Reposo en casa, administrar tratamiento sintomático vía oral y reevaluar a las 48 horas.
B Hospitalizar, administrar un beta lactámico y un macrólido, vía intravenosa.
C Hospitalizar, tomar cultivos y no administrar antibióticos hasta tener resultados.
D Reposo en casa, administrar amoxicilina clavulanato vía oral y reevaluar a las 48 horas.

60. ¿Cuál de los siguientes fármacos usados para el tratamiento de diabetes disminuye
la producción hepática de glucosa?

A Sulfanilureas.
B Rosiglitazona.
C Biguanidas.
D Nateglidina.

61. La deficiencia de folato en la anemia megaloblástica causa:

A Necrosis aséptica.
B Síndrome torácico agudo.
C Anemia microcítica.
D Abortos recidivantes.

62. Usted atiende a un paciente de 19 años de edad que refiere haber recibido
tratamiento ambulatorio para fiebre reumática a los 7 años de edad. Desde hace 3
días presenta un cuadro de infección faringoamigdalina y al momento se queja de
fiebre, decaimiento, malestar general y artralgias. Desea conocer si puede tratarse
de una reactivación de la fiebre reumática. ¿Cuál de las siguientes condiciones
corresponde con un criterio mayor de Jones para fiebre reumática?

A Presencia de eritema marginado.


B Velocidad de eritrosedimentación elevada.
C Leucocitosis mayor a 12.000/mm3.
D Prolongación del intervalo PR

63. Un escolar de 4 años de edad, eutrófico, acude a consulta porque desde hace 10
días presenta tos intensa, inicialmente y luego de un resfriado, era una tos seca.
Progresivamente se ha transformado en una tos productiva, con esputo claro y
espeso. Ha tomado varios medicamentos para eliminar la flema, sin embargo, la tos
persiste y le impide dormir normalmente. Al examen físico se encuentra afebril, con
buen estado general. Tose en forma repetitiva y a la auscultación presenta estertores
roncantes difusos, especialmente en la espiración. La Rx de tórax es normal, con
esta información. Seleccione el diagnóstico:

A Adenoiditis.
B Laringitis aguda.
C Bronquitis aguda.
D Bronquiolitis.

64. Señale los signos y síntomas que pertenezcan a la enfermedad por reflujo
gatroesofágico en pacientes pediátricos: 1. Disfagia. 2. Artralgia. 3. Regurgitación
recurrente con o sin vomito. 4. Perdida o escasa ganancia de peso.

A 1, 2, 4.
B 1, 2, 3.
C 2, 3, 4.
D 1, 3, 4. DRP

65. Un niño de 6 años de edad, que vive en una región tropical del país, acude a
consulta porque en los últimos 10 días presenta tos y esputo sanguinolento.
Además, dolor abdominal ocasional, así como deposiciones diarreicas abundantes y
con moco, que se producen por temporadas, desde hace un año. Ocasionalmente
presenta también vómitos. En este año, por tres ocasiones presentó lesiones
irregulares, longilíneas, a manera de trayectos indurados, eritematosos y
pruriginosos en tórax, abdomen, glúteos y miembros inferiores. Estas lesiones
desaparecieron en uno o dos días. Su peso está en el percentil 5 para la edad. Su
talla en el percentil 10. La biometría hemática reporta únicamente una marcada
eosinofilia, con 600 eosinófilos/ml de sangre. Se realiza un coproparasitario que
reporta la presencia de larvas parasitarias. Seleccionar el literal que contenga el
parásito más probable de acuerdo con el cuadro clínico de este niño, y el
medicamento con el que debería realizarse el tratamiento:

A Enterobius vermicularis; mebendazol, 100 mg dos veces diarias, por 7 días.


B Ascaris lumbricoides; albendazol, 200mg en una sola toma, por una sola vez.
C Strongyloides stercoralis; ivermectina, una tableta de 3 mg en ayunas, una vez.
D Giardia lamblia; metronidazol, 250 mg, 1 c/día, luego de una comida, 10 días.

66. Paciente femenina de 5 años de edad, con disuria, polaquiuria y dolor suprapúbico.
Presenta también hematuria visible en ropa interior. ¿Cuál es el diagnóstico clínico?

A Cólico renal.
B Pielonefritis.
C Bacteriuria asintomática.
D Cistitis.

67. La radiografía de caderas de una niña de 3 meses de edad presenta signos de


luxación congénita de su cadera izquierda. Identificar el literal que describe en mejor
forma las características radiográficas de esta patología:

Presencia del núcleo femoral izquierdo, ángulo acetabular izquierdo inferior de 27 grados,
A línea de Perkins alineada desde el extremo lateral del techo acetabular izquierdo con la epífisis
femoral.
Hipoplasia del núcleo femoral izquierdo, ángulo acetabular izquierdo de 26 grados,
B línea de Perkins alineada desde el extremo lateral del techo acetabular izquierdo con la
epífisis femoral.
C Hipoplasia del núcleo femoral izquierdo, ángulo acetabular izquierdo de 36 grados,
línea de Perkins desplazada hacia afuera del techo acetabular izquierdo.
Presencia del núcleo femoral izquierdo, ángulo acetabular izquierdo de 28 grados,
D
línea de Perkins desplazada hacia afuera del techo acetabular izquierdo.

68. Un lactante mayor de 24 meses de edad tiene un peso de 10kg, una talla de 70 cm y
un perímetro cefálico de 43 cm. El peso está bajo el percentil 5, la talla en el percentil
5 y el perímetro cefálico en el percentil 25. Al examen físico presenta astenia,
reducción del tejido celular subcutáneo y de las masas musculares. Abdomen algo
prominente. Piel seca, pelo escaso, descolorido, sin brillo y fácilmente desprendible.
No presenta visceromegalias. Si el percentil 50 del peso para esta edad es de 12 kg,
su pérdida de peso es de 16.6%. Seleccione el diagnóstico más probable para este
niño:

A Desnutrición calórica proteica de segundo grado.


B Desnutrición calórica proteica de primer grado.
C Desnutrición proteica calórica o kwashiorkor.
D Desnutrición calórica proteica de tercer grado.

69. Un paciente de 3 años de edad, tiene desde hace 5 días vómito y heces acuosas, sin
moco ni sangre. En el examen físico no se encuentran signos de deshidratación.
¿Cuál de estas opciones sería la mejor alternativa terapéutica?

A Ayuno hasta que ceda el vómito y antibiótico oral.


B Sales de rehidratación oral y alimentación habitual.
C Metoclopramida y sales de rehidratación oral.
D Racecadotrilo y restaurador de flora.

70. Paciente puerperal posterior a cesárea con diagnóstico de infección pélvica. ¿Cúal
es el esquema de tratamiento estandar de oro?

A Vancomicina y Metronidazol.
B Secnidazol y Ciprofloxacina.
C Clindamicina y Gentamicina.
D Vancomicina y Ciprofloxacina.

71. Se recomienda un manejo expectante en embarazos con ruptura prematura de


membranas entre las siguientes semanas de gestación:

A Menor de 20 semanas.
B 24-34.6 semanas.
C 20-24 semanas.
D mayor 35 semanas.
.

72. ¿Cuál es el examen de laboratorio más útil y de fácil acceso para evaluar deficiencia
de hierro en el embarazo?

A Ferritina sérica.
B Transferrina sérica.
C Hematocrito.
D Hemoglobina.

73. Una paciente de 78 años de edad fue sometida a un examen de fondo de ojo con
dilatación pupilar. Pocas horas más tarde una hija le llama a usted porque la
paciente se queja de intenso dolor en su ojo izquierdo, tiene visión borrosa con el
mismo ojo, náusea y vómito. Al examinarla, el ojo derecho está normal en forma y
función. En el ojo izquierdo hay enrojecimiento intenso, disminución severa de la
agudeza visual a visión solo de luz y la palpación refleja ojo duro y doloroso. El resto
del examen físico no presenta particularidades. Hasta que el oftalmólogo se haga
cargo del paciente, ¿Cuál de los siguientes fármacos debe utilizarse como parte
imprescindible del tratamiento emergente de primera elección?

A Difenhidramina 25 mg, 1 tableta cada 8 horas.


B Carbamazepina 100 mg, 1 tableta cada 12 horas.
C Acetazolamida 250 mg, 1 tableta cada 6 horas.
D Amitriptilina 25 mg, 1 tableta cada 12 horas.

74. Varón de 28 años de edad, en la mañana al salir de casa, en forma apresurada, no se


percata de un desnivel de la calzada y al parecer “se tuerce” el tobillo derecho. Con
ayuda de su hermano, regresa a casa y dos horas después al persistir el dolor que le
ocasionó este evento, llama al médico de su empresa, el cual en el examen físico
encuentra lo siguiente: dolor cuando apoya el pie o cuando camina, edema
moderado sobre los maléolos, equimosis moderada, inestabilidad unilateral leve.
¿Qué tipo de esguince presenta el paciente?

A Grado II.
B Grado I.
C Grado IV
D Grado III.
75. Un varón de 23 años de edad, previamente sano, acude a su consulta por presentar
desde hace 24 horas fiebre, escalofrío, malestar general, dolor perineal, disuria y
polaquiuria. En el examen físico presión arterial: 120/80mmHg, frecuencia cardiaca:
100 lpm (latidos por minuto), frecuencia respiratoria: 18 rpm (respiraciones por
minuto), temperatura bucal 39,5 OC, ruidos cardio-respiratorios y examen abdominal
son normales. Al tacto rectal siente intenso dolor interno y el examen de orina revela
abundantes leucocitos. Junto con medidas de hidratación y analgesia, ¿Qué tipo de
antibiótico es el recomendado como primera línea de tratamiento?

A Fluoroquinolonas.
B Aminoglucósidos.
C Cefalosporinas.
D Amoxicilina clavulanato.

76. La tos crónica con imagen radiográfica normal, se debe a la administración de:

A Dextrometorfán.
B Amitriptilina.
C Hidrocodona.
D Captopril.

77. Niño de 9 años, acude a consulta con tos leve con expectoración mucosa, disnea
espiratoria, el cuadro se ha presentado por algunas ocasiones. Al examen físico se
encuentra algo cianótico, se auscultan sibilancias e hipertimpanismo. Una de las
sospechas diagnósticas es el asma, con la finalidad de confirmar o descartar el
diagnóstico usted solicita algunos estudios funcionales pulmonares. Los siguientes
son resultados de pruebas funcionales pulmonares normales. EXCEPTO:

A FEF 25-75% = 86%.


B FVC menor a 68%.
C FEV 1 89%.
D FEV 1/ FVC = 90%.

78. Un hombre de 45 años, asintomático, acude a un control médico de rutina. Al


examen físico lo relevante es su presión arterial de 130/84, con índice de masa
corporal en 26. El examen de sangre reporta triglicéridos de 150mg/dL y colesterol
total de 240mg/dL, con colesterol LDL de 180mg/dL. El paciente desea conocer si
esos resultados son anormales. Según la clasificación de Fredrickson, ¿Qué tipo de
dislipidemia tiene este paciente?

A IIb.
B I.
C IIa.
D III.

79. Señale el medicamento que se usa como alternativa en el tratamiento sistémico del
acné:

A Azatioprina.
B Metformina.
C Fenitoína.
D Dexametasona.

80. Un síntoma de la anemia por deficiencia de complejo B es:

A Glositis.
B Síndrome de mano-pie.
C Abombamiento craneal.
D Priapismo.

81. Se trata de un paciente varón de 72 años. Presenta pérdida súbita del estado de
conciencia. Usted lo valora. Ante la aplicación del estímulo doloroso: el paciente
tiene flexión normal de sus extremidades, sin llegar a localizar el dolor, emite
sonidos guturales incomprensibles y abre los ojos, ¿A cuántos puntos equivale esta
respuesta en la escala del coma de Glasgow??

A 10.
B 4.
C 6.
D 8.
Escala de Glasgow: respuesta de apertura ocular: (2)al dolor, respuesta motora (4) flexión
normal, respuesta verbal(2)sonidos guturales, total puntación Glasgow:8

82. Un paciente de 21 años de edad sin antecedentes clínicos de importancia, presenta


un cuadro compatible con neumonía comunitaria y decide hospitalizarlo. Tres días
más tarde, con adecuada evolución clínica, usted solicita una radiografía de tórax
para control previo al alta, pero todavía persiste el infiltrado radiológico. En estas
condiciones y expresado en semanas, usualmente, ¿Cuál es el mínimo tiempo
promedio que demoran en desaparecer las alteraciones radiográficas?

A 3.
B 2.
C 4.
D 1.
La resolución de los infiltrados radiográficos a menudo ocurre varias semanas 4 a
meses después de la mejoría clínica, especialmente en el anciano y en la neumonía
multilobal

83. Un paciente sufrió hace 24 horas un infarto agudo de miocardio con elevación del
segmento ST en las derivaciones II, III, y aVF y fue sometido a tratamiento
farmacológico e intervencionista pero su frecuencia cardiaca actual es de 30 lpm
(latidos por minuto), la presión arterial es de 140/60 mmHg, frecuencia respiratoria:
18 rpm (respiraciones por minuto), está consciente y orientado pero se siente algo
mareado. La auscultación pulmonar es normal y no tiene ruidos cardiacos
sobreañadidos. En el electrocardiograma se evidencia presencia de ondas P
normales con una frecuencia de 84 por minuto y complejos QRS amplios, de 120
milisegundos, con frecuencia de 30 por minuto. ¿Cuál es el tratamiento de elección?

A Cardioversión eléctrica.
B Atropina 1 miligramo intravenoso.
C Marcapasos temporal endovenoso.
D Norepinefrina en infusión continua

84. Se trata de un paciente de 68 años de edad, con diabetes, hipertensión arterial y


fumador crónico. Durante una consulta médica de rutina, súbitamente presenta dolor
de pecho, pérdida de conciencia, no respira y no tiene pulsos palpables. Usted inicia
RCP con compresiones torácicas e insuflaciones con balón y mascarilla facial para
ventilación. De entre las siguientes maniobras, ¿Cuál NO está recomendada?

A Compresiones de más de 5 centímetros de profundidad.


B Más de 100 compresiones torácicas por minuto.
C Relación compresión ventilación de 30:2.
D Insuflaciones para lograr hiperventilación.

85. Paciente de 8 años, que pesa 24 kg, con estatus convulsivo, señale el fármaco y
dosis correcta de anticonvulsivante que debe ser suministrado inmediatamente.

A Diazepam vía intramuscular 2 mg STAT.


B Midazolam 5 mg intramuscular STAT.
C Fenobarbital 480 mg intravenoso STAT.
D Levetiracetam 4 mg vía oral STA

86. Un niño de 3 años de edad acude a consulta porque desde hace 5 días presenta
deposiciones blandas, con moco, sin sangre, en una frecuencia de 4 por día.
Además ha presentado fiebre y vómito en dos ocasiones. Al examen físico no se lo
encuentra deshidratado, ¿Cuál es la mejor conducta en este caso?

A Usar antiparasitario empírico.


B No tratar hasta tener exámenes.
C Usar antibioticoterapia empírica.
D Utilizar el plan A de la OMS

87. Una mujer de 16 años llega a la consulta por un chequeo rutinario. La anamnesis
revela amenorrea primaria, ella dice que ha aumentado la cantidad de ejercicio que
realiza. Durante el examen físico se encuentra un IMC de 21, desarrollo mamario
Tanner estadio 1 y FSH elevado. La prueba de BhCG fue negativa. ¿Cuál es el
siguiente paso?

A Prescribir anticonceptivos orales.


B Medir niveles de TSH.
C Pedirle que disminuya las horas de ejercicio que realiza y que regrese en 2 meses.
D Realizar un cariotipo.

88. Llega a la consulta una mujer de 27 años con un retraso en su menstruación de 2


meses. La anamnesis revela un aumento de 5 kg durante 6 meses anteriores (IMC
27), sequedad de piel, intolerancia al frío e irritabilidad. Durante el examen físico no
se evidencias signos alarmantes. Una prueba de BhCG es negativa. Con su posible
diagnóstico según la clínica. ¿Cuál es el tratamiento?

A Bromocriptina.
B Pastillas anticonceptivas orales.
C Metformina.
D Levotiroxina.

89. ¿Cuál es el esquema de tratamiento del ácido fólico en una mujer con epilepsia
tratada con anticonvulsivantes que este planificando un embarazo?

A 5 microgramos vía oral, al día, hasta la semana 5 de gestación.


B 5 miligramos vía oral, al día, hasta las 12 semanas de gestación.
C 2 miligramos vía oral, tres veces al día, hasta la 40 semanas de gestación.
D 2 miligramos vía parenteral, a la semana, hasta 20 semanas de gestación.

90. En la Clave Azul obstétrica el manejo correcto de impregnación con sulfato de


magnesio para la prevención de las convulsiones es:

A Administrar 5 ampollas al 20 % + 400 ml de solución isotónica pasar a 50 ml/hora.


B Administrar 1 gramo/hora IM de SO4MG en ampollas al 20 %
C Administrar 2 ampollas 20 % IM de SO4Mg con un contenido total de 4 gramos.
D Administrar 2 ampollas 20 % IV de SO4Mg + 80 ml solución isotónica a 101 gotas/minuto.

91. Un varón de 19 años presenta dolor abdominal de 12 horas de evolución. Al


momento el dolor se localiza en fosa ilíaca derecha, se acompaña de náusea, escaso
vómito y no ha realizado la deposición. Al examen físico el abdomen es suave,
doloroso a la palpación superficial en fosa ilíaca derecha, con signo de rebote
positivo. La apendicectomía inmediata reveló apendicitis grado II (dos). Recibe
ampicilina sulbactam por vía intravenosa, dosis única transoperatoria. ¿Cuál es la
mejor conducta a seguir?

A No requiere otro tratamiento antibiótico adicional.


B Asociación de gentamicina y metronidazol por 7 días.
C Metronidazol intravenoso cada 8 horas por 3 días.
D Asociación de ciprofloxacina y clindamicina por 7 días

92. Mujer de 76 años de edad. A raíz de caída de su propia altura refiere dolor a nivel de
la región inguinocrural que se irradia a la cara interna del muslo derecho. En el
examen físico se encuentra limitación para la marcha y la bipedestación, rotación
externa y acortamiento de la extremidad. En la radiografía se aprecia una fractura
por debajo del borde del cartílago articular y con orientación oblicua. Según la
clasificación anatómica de las fracturas intra articulares del extremo proximal del
fémur, ¿Qué tipo de fractura tiene la paciente?

A Basicervical.
B Transcervical.
C Capital.
D Subcapital.

93. Mujer de 32 años de edad, 2 hijos de 3 y 5 años, antecedentes de un episodio


depresivo a los 20 años de edad. Al llegar su esposo le entrega una nota en la que se
“despide” y le encarga “velar por sus hijos”, luego de lo cual se encierra en su
habitación. Luego de un largo tiempo el esposo logra ingresar a la habitación y
encuentra a la paciente llorosa, angustiada y con una apreciable cantidad de
medicamentos que han sido sacados de sus envases, al parecer listos para ser
ingeridos. El médico de urgencias que es llamado, realiza una valoración mediante la
escala de Patterson y obtiene un puntaje de 7. ¿Cuál sería su decisión?

A Observación en su domicilio.
B Sugerir consulta con psicólogo.
C Sugerir consulta con psiquiatra.
D Traslado a un hospital.

94. Los siguientes enunciados corresponden a los factores causales de la distanacia


(obstinación terapéutica), EXCEPTO:

Deseo del paciente crítico por eliminar su propia existencia o de sus representantes legales,
A
para evitar la prolongación del sufrimiento del paciente.
Adopción de medidas terapéuticas que contemplan más los aspectos científicos de la
B
enfermedad que al enfermo afectado de un proceso irreversible.
Desconocer el derecho del paciente o de sus representantes legales, a rechazar el inicio o
C continuación de tratamientos médicos que prolonguen el sufrimiento del enfermo crítico o su
agonía.
Convencimiento de algunos médicos de que la vida biológica es un bien por el que
D luchar, al margen de consideraciones sobre la calidad de esa vida.

95. Un adulto de 48 años de edad, alcohólico crónico, presenta un cuadro compatible


con neumonía. Al examen físico se evidencia gingivitis importante. Para el
tratamiento de este paciente, además de los gérmenes habituales involucrados en la
etiología de la neumonía comunitaria, ¿Cuál tipo de gérmenes debe ser
obligatoriamente cubierto?

A Pseudomonas.
B Aspergilus.
C Porfiromonas.
D Acinetobacter.

96. El uso de los bloqueadores b cardioselectivos en el tratamiento de hipertensión


arterial están contraindicados en caso de:

A Infarto al Miocardio.
B Taquiarritmias ventriculares.
C Bloqueo cardíaco de segundo o tercer grado.
D Taquicardia sinusal.

97. Un paciente de 59 años de edad, con diagnóstico de diabetes mellitus tipo II


diagnosticada hace 4 años, bajo tratamiento con antidiabéticos orales. En el control
de rutina usted encuentra que el índice de masa corporal está en 24 Kg/m2, la
presión arterial es 110/70 mm Hg y el examen físico es normal. Las cifras de
colesterol total son de 150 mg/dL con HDL de 40 mg/dL, triglicéridos de 110 mg/dL y
su hemoglobina glicosilada es de 7%, ¿En qué tiempo debería realizarse otro
examen de lípidos en sangre?
A 12 meses.
B 3 meses.
C 8 meses.
D 4 meses.

98. Señale el resultado de laboratorio que se encuentra con relación a la desnutrición


aguda:

A Capacidad total transporte de hierro sérico 480 ug/100 mL


B Nitrógeno de úrea 28 mg /100 mL
C Creatinina sérica 1.8 mg/100 mL
D Albúmina sérica 2.5 g /100 mL

99. Paciente de 21 años, sexualmente activa con varias parejas sexuales, que en
ocasiones utiliza preservativo, refiere que desde hace 3 días presenta secreción
vaginal y prurito vulvar. En el examen físico se observa eritema vulvar y secreción
vaginal amarilla, espesa, espumosa; fétida y con un pH de 7. No hay dolor pélvico a
la palpación ni adenopatías inguinales. Al tacto vaginal no se palpan masas
anexiales y no hay dolor a la movilización del cérvix. ¿Cuál es el diagnóstico de la
paciente?

A Vaginosis bacteriana.
B Tricomoniasis vaginal.
C Candidiasis vulvovaginal.
D Enfermedad pélvica inflamatoria

100. Un adulto de 55 años de edad previamente sano, acude a su consulta por tos
seca de 8 días de evolución, que desde hace 4 días se acompaña de fiebre y poca
expectoración. Desde hace 24 horas refiere sentir dolor torácico derecho. Al examen
físico el paciente está consciente, orientado, tiene una presión arterial de 100/60
mmHg, frecuencia cardiaca: 100 lpm (latidos por minuto), frecuencia respiratoria: 18
rpm (respiraciones por minuto), se auscultan estertores crepitantes subescapulares
derechos. En sangre los leucocitos están en 8.500/mm3 con 66% de segmentados,
28% de linfocitos, la creatinina es de 0,9mg/dL y la urea de 20 mg/dL. La radiografía
de tórax muestra un pequeño infiltrado intersticial en base derecha. ¿Cuál es el
puntaje del CURB-65?

A 3 puntos.
B 4 puntos.
C 1 punto.
D 0 punto
En la escala de severidad CURB-65 para neumonía se valora: confusión, urea:> de 44
mg/dl o BUN >de 19 mg / dl, frecuencia respiratoria (Mayor de 30 por minuto), Presión
sanguínea (PAS menor de 90 mmHg o PAD menor de 60 mmHg), y edad >65años, a cada
uno se le asigna un puntaje de 1, como el paciente no cumple con ningún criterio, el
puntaje de CURB-65 es de 0 puntos

101. Un paciente adulto permanece inmovilizado por una enfermedad neurológica


paralizante aguda y lo alimentan por vía nasogástrica. Recibe los aportes de
calorías, proteínas y de líquidos con electrolitos, calculados para su condición. El
paciente cotidianamente recibe dosis de complejo B, ácido ascórbico, vitamina A, y
recibe gel de aluminio para prevención de sangrado digestivo. Los signos vitales y
los datos de la biometría hemática, urea, glucosa y creatinina séricas son normales.
En lo referente a la concentración de iones en sangre, ¿Cuál es el principal riesgo
metabólico de este paciente?

A Hiponatremia.
B Hipercalcemia
C Hipokalemia
D Hipermagnesemia

102. RN a término, Peso al nacimiento: 2,3 kg; Talla: 49 cm; PC: 35 cm; APGAR: 7 /
10; es reactivo al manejo, su respiración es normal y presenta buen reflejo de
succión. Se lo mantiene en observación. A las 4 horas de edad presenta glucemia de
50 mg/dl. Aún no se lo ha puesto en contacto con su madre. Determine el
diagnóstico y terapéutica:

A Retardo de crecimiento intrauterino; dextrosa al 5% en AD + controles de glucemia.


B Retardo de crecimiento intrauterino; dextrosa al 10% en AD + controles de glucemia.
C Control insuficiente en el embarazo; dextrosa al 10% en AD + controles de glucemia.
Retardo de crecimiento intrauterino; lactancia materna a libre demanda + controles de
D
glucemia.
Si estuviera bajo la cifra de 45 mg/dl tendría una hipoglucemia, que necesitaría algo
más que lactancia materna, como la administración de una solución dextrosada.

103. Una niña de 4 años de edad acude hoy al Centro de Salud para un control.
Tiene un IMC de 17 que en la curva de crecimiento de la OMS corresponde a una
puntuación Zeta (Zeta escore o desviaciones estándar) de +1 a +2. De acuerdo a los
estándares de crecimiento de la OMS, ¿Cuál de las siguientes opciones sería el
diagnóstico más probable?

A Riesgo sobrepeso.
B Obesidad leve.
C Dentro de lo normal.
D Sobrepeso evidente.

104. Mujer de 37 años, gestas 5, para 4, abortos 1. Consulta por presentar


sintomatología variada, en la que se incluye: pérdida de peso, amenorrea secundaria
de 8 meses de duración, después del último parto, a pesar de que no pudo dar de
lactar, caída de vello púbico, disminución del volumen mamario, decaimiento y piel
seca. Tiene como antecedente personal hemorragias post parto, que en el último fue
exagerada, con signos de choque hipovolémico, el cual precisó transfusión de
sangre, ¿Cuál de las siguientes glándulas cree usted que es responsable del cuadro
clínico?

A Ovarios.
B Tiroides.
C Hipófisis.
D Suprarrenales.

105. Indique la familia a la cual pertenece el virus que produce el molusco


contagioso y su tiempo de incubación.

A Virus del papiloma humano, 8 a 12 semanas.


B Herpesviridae, de 3 a 5 semanas.
C C. trachomatis, 1 a 2 semanas.
D Poxviridae, 2 a 7 semanas

106. ¿Cuál es el criterio clínico más importante para establecer el diagnóstico de


metritis puerperal?

A Escalofrío.
B Dolor abdominal a la palpación.
C Fiebre.
D Loquios fétidos.

107. Paciente de 21 años con embarazo de 28 semanas por FUM, refiere fiebre,
dolor costovertebral, náuseas y vómito de 48 horas de evolución. El EMO presenta
piuria y hematuria. ¿Qué diagnóstico tiene la paciente y qué estudio debe solicitar?

A Cistitis aguda, solicitar ultrasonografia renal.


B Pielonefritis, solicitar cultivo y antibiograma de orina.
C Litiasis renal, solicitar urografía  con contraste.
D Bacteriuria asintomática, solicitar hemocultivo.

108. En el segundo trimestre del embarazo. ¿Qué valores de hemoglobina y


hematócrito se consideran anemia?

A Hb mayor a 11 g/dL y Hcto 33%


B Hb mayor a 12 g/dL y Hcto 36%
C Hb menor a 10.5 g/dL y Hcto 32%
D Hb menor a 10 g/dL y Hcto 30%

109. Señale la indicación de referencia de un paciente quemado a un centro


especializado:

A Quemadura de tercer grado.


B Quemadura de segundo grado.
C Quemadura de espesor parcial del 5 % de TBSA.
D Quemadura de glúteo.

110. Son síntomas y signos de Rinosinusitis aguda, EXCEPTO:

A Epistaxis recurrente.
B Descarga nasal purulenta.
C Dolor facial.
D Hipersensibilidad senos paranasales.

111. Para el tratamiento de un primer episodio de depresión mayor, moderada o


severa, los diversos grupos de antidepresivos han demostrado tener niveles
similares de efectividad. ¿Cuál de ellos tiene mayor tolerabilidad y mejor relación
riesgo-beneficio?

A Antidepresivos duales.
B Inhibidores selectivos de la recaptación de serotonina (ISRS)
C Antidepresivos tricíclicos.
D Inhibidores de la mono amino oxidasa.

112. ¿En qué caso el consentimiento informado debe ser escrito y notariado?

A Procedimientos endoscópicos y biopsias.


B Exámenes radiológicos que impliquen anestesia y/o medios de contraste.
C Donante vivo y trasplante de órganos.
D Tratamientos de radioterapia y quimioterapia.

113. En los cuadros de artropatía psoriásica. ¿Cuál de los siguientes parámetros


clínicos o de laboratorio es más común entre las mujeres en relación a los hombres?

A Tener artritis deformante.


B Tener artritis de afectación solo distal.
C Tener artritis de afectación simétrica.
D Ser seropositivo para factor reumatoide.

114. Señale la dosificación ideal (objetivo terapéutico) de HbA1c en pacientes


ancianos con múltiples enfermedades crónicas y dificultad para las actividades de la
vida diaria:

A HbA1c 7.0%
B HbA1c 10.0%.
C HbA1c 6.5 %.
D HbA1c 8.0 %.

115. La evolución natural de la nefropatía diabética se caracteriza por los


siguientes eventos. EXCEPTO:

A Hiperperfusión glomerular.
B Acidosis tubular.
C Prostatitis.
D Albuminuria.

116. Señale la patología que ocasiona anemia megaloblástica:

A Síndrome de Mallory-Weiss.
B Tumor esofágico.
C Hernia del hiato.
D Gastrectomía total.

117. Se trata de un paciente de 47 años de edad, que hace 2 semanas tuvo un


episodio de gastroenteritis aguda que revirtió de manera espontánea. Desde hace 48
horas, sin causa aparente, presenta hormigueos y debilidad progresiva en los
miembros inferiores que al momento le impide la deambulación normal. Al examen
físico de las extremidades inferiores la motricidad es 3/5, no tiene reflejos
tendinosos y la sensibilidad está conservada. El resto del examen neurológico es
normal, así como los exámenes de sangre y el análisis del líquido cefalorraquídeo,
¿Cuál es el diagnóstico más probable?

A Síndrome de Guillain Barré.


B Esclerosis múltiple.
C Mielitis transversa.
D Síndrome de Brown Sequard.
118. Un paciente obeso con diagnóstico de hipertensión arterial y diabetes, en
tratamiento farmacológico con orlistat, metformina, enalapril e hidroclorotiazida, se
queja de aparecimiento de heces grasosas, flatulencia y urgencia para la defecación
que en ocasiones llega a la incontinencia fecal. ¿Cuál de los fármacos es el que con
mayor probabilidad puede ocasionar estos síntomas?

A Hidroclorotiazida.
B Orlistat.
C Metformina.
D Enalapril.

119. RN de 36.3 semanas por FUM, nació por cesárea sin labor, con membranas
íntegras. Líquido amniótico claro con grumos. Crecimiento adecuado para su edad
gestacional. Al nacimiento presenta taquipnea, quejido, cianosis, retracciones
costales y subcostales. Se le administra oxígeno y una hora después presenta
mejoría. Seleccione el diagnóstico correcto:

A Síndrome de aspiración de meconio.


B Neumonía neonatal.
C Taquipnea transitoria del recién nacido.
D Enfermedad de membrana hialina.

120. Seleccione las medidas generales que debe recibir un paciente menor de 5
años con neumonía adquirida en la comunidad:

A Dar antitusígenos, tratar la fiebre, no forzar la alimentación sólida.


B Tratar la fiebre, dar líquidos, no forzar la alimentación sólida.
C Dar mucolíticos, antitusígenos, tratar la fiebre.
D Dar líquidos, fisioterapia respiratoria, tratar la fiebre.

121. Un adulto joven de 19 años previamente sano, desde hace 6 meses tiene
episodios de tos, especialmente nocturna, siente que tiene secreciones pero es
difícil expectorar, estos episodios se acompañan de disnea, duran unas horas y
ceden de manera paulatina. Para aliviar la sintomatología recibe salbutamol. ¿Cuál
es el mecanismo de acción de este fármaco?

A Bloquea receptores beta adrenérgicos.


B Estimula receptores alfa adrenérgicos.
C Bloquea receptores muscarínicos.
D Estimula la adenilciclasa.
122. Complete el siguiente enunciado: Una de las contraindicaciones para el uso
del dispositivo intrauterino de liberación de levonorgestrel (LNG-IUS) es __________
presentando una tasa de expulsión de _________

A Metrorragia de origen desconocido, 20 %.


B Mujeres infectadas por el virus de inmunodeficiencia humana (VIH
C La existencia de leiomiomas que deformen la cavidad del útero, 10 %.
D La existencia de leiomiomas que no deformen la cavidad uterina, 30 %.

123. Complete el siguiente enunciado: Los diámetros oblícuos del plano de


entrada de la pelvis se extienden desde las…………… a ………………….

A Sincondrosissacroiliacas, eminencia iliopectíneaipsilateral.


B Sindesmosissacroiliacas, eminencia iliopectínea contralateral.
C Sincondrosissacroiliacas, borde lateral de sínfisis del pubis.
D Sindesmosissacroiliacas, espina ciática contralateral

124. Ante la necesidad de maduración pulmonar fetal, ¿Cuál de los siguientes


conceptos es ERRADO?

A La betametasona es la droga más recomendada.


B Disminuye el distress respiratorio y hemorragia cerebral.
C TRH, ambroxol y betametasona son equivalentes en eficacia.
D Está indicada entre 24 y 34 semanas.

125. Identifique la etapa de choque hemorrágico en la que se encuentra un


paciente traumatizado, que al examen físico presenta una FC: 125 lpm, presión
arterial 90/60mmHg, FR: 35 rpm, diuresis: 10cc/H, en la evaluación del sistema
neurológico presenta confusión, con una pérdida hemática de 1500 a 2000 cc de
sangre.

A Clase II.
B Clase IV.
C Clase III.
D Clase I.

126. Elija el término correcto para la siguiente definición: Los síntomas somáticos
pueden manifestarse de forma brusca por una descarga súbita y aislada del sistema
nervioso vegetativo junto a un miedo intenso a morir o a perder el control. La
duración de los síntomas es de aproximadamente entre 15 y 30 minutos y su
expresión máxima es en los primeros 10 minutos.

A Ansiedad
B Manía.
C Angustia.
D Euforia

127. Durante el Control prenatal específico en mujeres con DM uno de los


exámenes a solicitarse con su debida frecuencia es la ecografía. De la tabla expuesta
a continuación, relacione el período del embarazo con el objetivo para el cual se
solicita la ecografía. Semana gestacional Objetivo de la ecografía 1. Semanas 11 a
14. 2. Semanas 20 a 22. 3. Semanas 28 a 30. 4. Semana 36 . a. Realizar ecografía para
valorar crecimiento fetal, perímetro abdominal fetal y determinación de percentil para
descartar macrosomía fetal incipiente. b. Realizar ecografía para valorar crecimiento
fetal y el volumen del líquido amniótico. c. Realizar ecografía para determinar
marcadores ecográficos de AC d. Realizar una ecografía morfológica fetal (detalle
anatómico)

A 1b, 2c, 3d, 4a.


B 1c, 2d, 3a, 4b.
C 1a, 2b, 3c, 4d.
D 1d, 2a, 3b, 4c.

128. Indique el enunciado perjudicial en pacientes con diabetes:

A Dieta hipocalórica baja en carbohidratos


B Consumo mínimo de grasas trans.
C Consumo de edulcorantes sin calorías.
D Consumo de antioxidantes u oligoelementos

129. La presente imagen sobre lesiones de piel se relaciona con el siguiente


diagnóstico:
A Erisipela.
B Dermatitis por contacto.
C Celulitis.
D Angioedema.

130. El paludismo se caracteriza por la presencia de crisis febriles que tienen una
determinada periodicidad, la misma que orienta sobre el tipo de Plasmodium
infectante. Los episodios febriles se presentan con intervalos variables y conocidos,
que en la mayoría de ellos son conocidos por su periocidad. Seleccione el tipo de
Plasmodium que presenta estos episodios clínicos de fiebre con el mayor intervalo
de tiempo:

A P. malariae.
B P. ovale.
C P. falciparum.
D P. vivax.

131. Un paciente de 72 años, con nefropatía crónica, en tratamiento clínico. Acude


a emergencias por palpitaciones. Al examen físico la presión arterial es 140/90 mm
Hg, frecuencia cardiaca: 78 lpm (latidos por minuto), ritmo irregular, frecuencia
respiratoria: 20 rpm (respiraciones por minuto). Está consciente y orientado. En los
exámenes de laboratorio lo relevante es un sodio de 136 mEq/L, potasio de 6 mEq/L,
urea 40 mg/dL y creatinina 2,4 mg/dL, glicemia de 120 mg/dL y bicarbonato normal.
Para su desequilibrio electrolítico, ¿Cuál de las siguientes sustancias NO ayudaría?

A Salicilato.
B Insulina.
C Salbutamol.
D Bicarbonato.

132. Una paciente de 18 años de edad sin antecedentes clínicos de importancia,


presenta malestar general, decaimiento, tos y fiebre de 2 días de evolución. Al
examen físico tiene presión arterial: 110/70 mmHg, frecuencia cardiaca: 100 lpm
(latidos por minuto), frecuencia respiratoria: 32 rpm (respiraciones por minuto),
temperatura bucal de 39 OC, se ausculta broncofonía y pectoriloquia derecha
unilateral. Usted decide ingresarla y tratar con antibióticos, la paciente se ha
mantenido más estable. Al segundo día de tratamiento, presenta de manera brusca
dolor torácico, disnea, presión arterial: 70/40 mmHg, frecuencia cardiaca: 150 lpm
(latidos por minuto), frecuencia respiratoria: 44 rpm (respiraciones por minuto), a la
percusión tiene hiperresonancia y ausencia de ruidos pulmonares en el lado
derecho. ¿Cuál es la presunción diagnóstica más probable?
A Hemotórax.
B Derrame paraneumónico.
C Neumotórax a tensión.
D Neumotórax simple

133. Un paciente con antecedentes de regurgitación y pirosis que actualmente se


queja de disfagia y dolor torácico retroesternal acude a su consulta. Al examen físico
se encuentra peso 82 kilogramos, talla 1,70 metros, signos vitales presión arterial
110/70 mm Hg, frecuencia cardiaca 72 lpm (latidos por minuto), pulso regular,
frecuencia respiratoria 16 rpm (respiraciones por minuto), temperatura bucal 36,5oC.
Resto del examen no presenta particularidades remarcables, ¿Cuál de los siguientes
fármacos es de primera elección para este tipo de alteración?

A Ibuprofeno.
B Verapamilo.
C Omeprazol.
D Loperamid

134. Un niño de 3 años de edad acude a emergencia por presentar desde hace 2
días fiebre, vómito e irritabilidad. En el examen físico se encuentra letargia, signos
de Kernig y Brudzinski positivos. El Gram del líquido cefalorraquídeo arroja como
resultado un coco bacilo Gram negativo. ¿Cuál de las siguientes opciones
corresponderá al agente etiológico más probable?

A Haemophilusinfluenzae.
B Listeria monocitogenes.
C Neisseriameningitidis.
D Escherichiacoli.

135. La lactancia materna está contraindicada en las siguientes situaciones.


EXCEPTO:

A Mastitis Aguda.
B Fenilcetonuria.
C VIH y carga viral alta.
D Galactosemia.

136. Un niño de 10 años de edad presentó hace 2 días rinorrea y tos. Hoy presenta
fiebre alta, dolor torácico, tos húmeda. Al examen presenta dificultad respiratoria y
taquipnea, ¿Cuál de los siguientes NO es un criterio para la selección de la
antibioticoterapia empírica en este caso?

A Grupo de edad.
B Análisis del esputo.
C Epidemiología local.
D Hallazgos radiográficos.

137. Paciente de 49 años, acude a la consulta por presentar durante los últimos
seis meses episodios de sensación de calor intenso localizado en cara, cuello y
tórax. Son de corta duración (minutos). Estos episodios son seguidos de sudoración
profusa con la misma localización y se asocian a palpitaciones y angustia. Se repiten
de 5 a 6 veces al día y en la noche le impiden el sueño. También refiere sequedad
vaginal que provoca dispareunia y pérdida del “gusto por la vida”. La fecha de la
última menstruación es hace 4 meses y sus ciclos menstruales han sido irregulares
desde hace 18 meses. Usted deduce que son síntomas compatibles con
premenopausia. ¿Para confirmarla usted pediría?

A Determinación de progesterona.
B Hormona liberadora de gonadotropinas.
C Hormona folículo estimulante.
D Determinación de estrógenos.

138. La forma de prescripción de dosis de impregnación de Sulfato de Magnesio,


como tratamiento para prevenir la eclampsia es:

A 4 gramos de Sulfato de Magnesio, vía intravenosa a pasar en 20 minutos.


B 8 gramos de Sulfato de Magnesio, vía intravenosa a pasar en 10 minutos.
C 6 gramos de Sulfato de Magnesio, vía intravenosa a pasar en 20 minutos.
D 3 gramos de Sulfato de Magnesio, vía intravenosa a pasar en 20 minutos.

139. La inductoconducción de la labor de parto con oxitocina es uno de los


procedimientos más frecuentes en obstetricia, ¿Cuál de los siguientes conceptos
NO es correcto en relación al uso de oxitocina?

A La respuesta del útero es variable.


B Produce efecto de taquifilaxis.
C Tiene efecto antidiurético.
D La vida media es de 30 minutos
140. Mujer de 26 años, ingresada a sala de alto riesgo obstétrico, por embarazo a
término complicado con preeclampsia severa y manejada con sulfato de magnesio a
dosis de impregnación de 6 g y de mantenimiento a 2 g/hora. Desde su ingreso
recibe hidratación intravenosa, una dosis de misoprostolintravaginal de 25 mcg y
una de hidralazina 5 mg IV. Al momento presenta sensación de calor y eritema en
cara y tronco, leve somnolencia, hiporeflexiapatelar y disminución de la variabilidad
de la frecuencia cardíaca fetal. ¿Cuál de los medicamentos recibidos explicaría las
manifestaciones actuales de la paciente?

A Fluidos intravenosos.
B Sulfato de magnesio.
C Hidralazina.
D Misoprostol.

141. Usted va a empezar a realizar una investigación observacional analítica para


buscar la relación que existe entre la obesidad y los videos juegos en estudiantes de
secundaria, mayores de 13 años y menores de 18 años. Para la expresión de
voluntariedad y ejercicio de autonomía. ¿Qué documentos deben firmar los
participantes?

A Consentimiento informado del participante y de sus padres o representante legal.


Consentimiento informado firmado por el participante y la carta de aprobación del director
B
escolar.
C Consentimiento informado firmado por los padres y la carta de aprobación del director escolar.
D Carta de aprobación del director(a

142. Las siguientes son enfermedades que se asocian con disfagia y se alivian
con tratamientos específicos. EXCEPTO:

A Los cánceres de cabeza y cuello avanzados.


B El Síndrome de Plummer-Vinson o de Paterson-Kelly.
C Neuralgia del nervio glosofaríngeo.
D Globushystericus.

143. Un paciente de 50 años de edad, acude a un control de rutina. En el examen


físico encuentra un índice de masa corporal de 28, presión arterial 124/80 mm Hg, sin
otras particularidades. En los exámenes de laboratorio se reporta: glicemia 88
mg/dL, colesterol total 240 mg/dL, colesterol HDL 30 mg/dL, colesterol LDL 170
mg/dL, triglicéridos 210 mg/dL y TSH elevada con T4 normal. Según la escala de
Frederickson, que es la más utilizada por la OMS, ¿Cuál es la clasificación de esta
dislipidemia?
A II A.
B III.
C II B.
D I.

144. Una persona con diabetes tiene una crisis hiperglicémica y usted sospecha
en un cuadro hiperosmolar y necesita calcular la osmolaridad plasmática. A más de
la glicemia, el sodio y el potasio, ¿Qué otro elemento de la sangre debe tomar en
cuenta para el cálculo convencional de la osmolaridad plasmática?

A Nitrógeno ureico.
B Albúmina.
C Proteínas totales.
D Magnesio y calcio.

145. Una agricultora de 48 años de edad. Desde hace 3 semanas tiene fiebre, tos,
escasa expectoración hemoptoica y dolor abdominal. Varias pruebas de
baciloscopia han sido negativas. En la radiografía pulmonar demuestra un infiltrado
intersticial basal derecho, la biometría reporta 12.000 leucocitos /mm3, con 60% de
neutrófilos, 25% de linfocitos y 15% de eosinófilos. De la siguiente lista, ¿Cuál es el
microorganismo implicado con más probabilidad?

A Cryptococcusneoformans.
B Ascarislumbricoides.
C Tripanosoma Cruzy.
D Ameba histolítica.

146. Un guía de turismo de selva acude por presentar una lesión en la piel del
dorso de la muñeca provocada por una picadura de insecto. Desde hace un par de
meses, la pápula tomó un color violeta, tiende a crecer y a ulcerarse. Al examen
físico vemos una lesión papular indurada, color rojo vinoso, con halo eritematoso,
de unos 6 centímetros de diámetro, cubierta de una capa escamosa y en el centro
tiene una costra redondeada de 4 centímetros de diámetro, no es dolorosa. El
resultado del frotis encuentra macrófagos y escasos microorganismos en formas
amastigotes. ¿Cuál de los siguientes fármacos está indicado como tratamiento de
primera línea para esta lesión?

A Estreptomicina más tetraciclina.


B Albendazol por tres días seguidos.
C Antimoniato de meglumina.
D Ceftriaxona diaria por siete días.

147. Las siguientes afirmaciones son correctas en relación a la rubéola.


EXCEPTO:

A Es una enfermedad usualmente benigna, excepto para el embrión durante el embarazo.


B La fiebre de la rubéola es alta, dura 3 días y coincide con el exantema generalizado.
C Presenta una erupción máculopapular eritematosa, leve, acompañada de linfadenopatías.
D En adolescentes y adultos, presenta con frecuencia poliartralgias y poliartritistransitoria

148. Miriam dio a luz hace 3 horas por cesárea, usted le recomienda que alimente
al recién nacido con leche materna, pero la madre le dice que no tiene leche. Escoja
la opción correcta en relación a la lactancia post cesárea.

Le pide a Miriam que mezcle leche infantilizada con leche del banco y espera a que la producción
A
propia aumente.
B Le da leche de fórmula para complementar hasta que Miriam produzca mas leche materna.
C Le solicita acudir a un banco de leche humana para que el niño reciba leche materna.
Le tranquiliza y le dice que ahora tiene calostro, verificando la posición correcta para dar
D
de lactar.

149. Un bebé nace prematuro. Su madre desea conocer qué posibilidad tiene de
sobrevivir, ¿Cuál de los siguientes factores está asociado a mayor supervivencia
con menor morbilidad?

A ECO fetal normal.


B Mejor APGAR.
C Tamizaje normal.
D Mayor peso.

150. Madre acude a la consulta con su hijo de 2 semanas de edad, no se realizó


controles prenatales durante el embarazo y tuvo parto en casa en su comunidad. El
niño presenta desde hace 2 días fiebre, irritabilidad, falta de apetito y decaimiento. Al
examen físico: P: 110/min, PA: 60/45 mmHg, FR: 18/min, T: 38 °C. El infante esta
letárgico, tiene respiraciones superficiales, fontanelas abultadas, tono y fuerza
muscular disminuidos en las extremidades. Corazón, pulmones, abdomen y llenado
capilar normales. Se realiza una punción lumbar que muestra una pleocitosis con
predominio de neutrófilos, glucosa disminuida y proteínas elevadas. ¿Cuál de los
siguientes patógenos es el causante del cuadro clínico?

A Streptococo del grupo B.


B Influenza tipo A.
C Streptococoviridans.
D Streptococopneumoniae.

151. Un niño de 4 meses de edad presenta irritabilidad, vómito y diarrea desde


hace 12 horas. Hace 24 horas recibió las vacunas correspondientes a su edad en
base al esquema nacional. Al examen físico presenta una temperatura de 38.0 oC,
frecuencia cardíaca 110 lpm (latidos por minuto), frecuencia respiratoria 46 rpm
(respiración por minuto), llenado capilar < 2 seg. ¿Cuál sería la conducta más
adecuada?

A Prescribir un antidiarreico.
B Administrar antitérmico.
C Realizar punción lumbar.
D Investigar la causa de la diarrea.

152. En las maniobras de Leopold, la situación del feto hace referencia a:

A La relación entre el polo fetal que se pone en contacto con el estrecho superior de la pelvis.
B La relación del eje longitudinal del feto con el eje longitudinal de la madre.
C La relación del dorso del feto con la parte derecha o izquierda de la madre.
D La relación de la fontanela posterior del feto con la pelvis.

153. Paciente 31 años, tiene 3 partos anteriores, al momento cursa embarazo de


35 semanas, de evolución normal hasta hace 4 horas en que presentó salida de
líquido transparente por vagina, en cantidad abundante. El examen abdominal
constata el feto único, cefálico, vivo. No hay actividad uterina. La maniobra de
Valsalva no puede confirmar la salida de líquido por introito, ¿Cuál de los siguientes
pediría para confirmar mejor el diagnóstico de rotura de membranas?

A pH vaginal.
B Examen especular.
C Cristalografía.
D Ecografía obstétrica

154. Una mujer de 42 años fue expulsada de su vehículo durante una colisión
vehicular. Camino al departamento de emergencia personal paramédico reporta:
frecuencia cardiaca de 130/min, presión arterial de 90/45 mmHg y frecuencia
respiratoria de 34/min. Presenta una herida en extremidad inferior derecha a la que
se realiza presión directa para controlar el sangrado. Al momento la paciente se
encuentra ansiosa y confusa. Seleccione el grado de hemorragia según pérdida
sanguínea estimada y elija el tipo de restitución de líquidos.

A Hemorragia grado III y necesita cristaloides.


B Hemorragia grado IV, necesita cristaloides y sangre.
C Hemorragia grado II y necesita cristaloides.
D Hemorragia grado III y necesita cristaloides y sangre.

155. Varón de 63 años de edad, previamente sano, acude a su consulta por


presentar varios meses de polaquiuria, tenesmo vesical, disuria y malestar general.
Al examen físico lo relevante es una próstata dolorosa al tacto rectal. El examen de
orina ha revelado leucocitos y piocitos pero sin bacterias, la tinción BAAR no
reporta gérmenes, ¿Qué tipo de tratamiento es el recomendado en este caso?

A Gentamicina 240 miligramos vía intramuscular, en dosis única diaria, por 3 días.
B Ciprofloxacina 500 miligramos cada 12 horas, vía oral por 30 días.
C Ampicilina 500 miligramos cada 6 horas, vía oral, por 14 días.
D Trimetoprimsulfametoxazol 160/800 miligramos, vía oral, cada 12 horas, por 5 días

156. Mujer de 59 años de edad, consulta por un cuadro de instauración


progresiva, que se ha incrementado en los dos últimos meses y que se caracteriza
por sensación de rigidez matinal de la articulación coxo femoral derecha, por
espacio aproximado de 15 minutos. Se acentúa al subir las gradas y tiende a
disminuir conforme se mantiene en movimiento. La radiografía de cadera muestra
osteofitosacetabulares y disminución del espacio articular, ¿Cuál es el tratamiento
de primera elección?

A Ketorolaco.
B Metamizol.
C Paracetamol.
D Glucosamina.

157. Paciente de 68 años, diagnosticado con diabetes se hospitaliza por


descompensación de valores glicémicos. Señale la opción que refleja un modelo
autonomista en la relación clínica de este paciente.

A El paciente menciona que dejará que el médico tome las decisiones por él.
B Establecer con el paciente los objetivos que se quieren lograr y que él quiera lograr.
C Paciente debe seguir estrictamente indicaciones médicas aún cuando tiene opiniones contrarias.
D El médico toma todas las decisiones porque el paciente no sabe nada de medicina.
158. Indique en cuál de las siguientes patologías hipertensivas se utiliza
fentolamina:

A Hipertensión arterial maligna.


B Encefalopatía hipertensiva.
C Feocromocitoma.
D Disección aórtica.

159. Madre lleva a la consulta a su niño de 5 años de edad por cuanto presenta
lesiones de piel que aparentemente se exacerban con el consumo de alimentos.
¿Cuál diagnóstico está relacionado con la dieta?

A Dermatitis de contacto.
B Acné vulgaris.
C Dermatitis herpetiforme.
D Dermatitis atópica.

160. Señale cuál es la manifestación clínica de la escabiosis reincidente:

A Máculas azuladas.
B Sarpullido generalizado.
C Sarpullido local.
D Pseudoliendres.

161. De las siguientes opciones. ¿Cuál es una complicación metabólica de la


pancreatitis aguda?

A Hipotrigliceridemia.
B Hipercalcemia.
C Hiperglucemia.
D Hiponatremia.

162. Se trata de una paciente de 51 años de edad con antecedentes de dolor


somático crónico, acostumbra automedicarse acetaminofén. Acude a emergencias
por cuadro confusional agudo. Luego de realizar exámenes, se descartan procesos
infecciosos u ocupativos del sistema nervioso. Usted sospecha en intoxicación por
acetaminofén. Como primera opción de tratamiento, ¿Cuál fármaco está indicado?

A Bicarbonato de sodio.
B Solución polarizante.
C N acetilcisteína.
D Fenitoina sódica.

163. Una paciente campesina de 22 años de edad presenta un cuadro compatible


con pancreatitis aguda. En la ecosonografía se evidencia páncreas incrementado de
tamaño, vesícula biliar libre de cálculos, pero dentro del colédoco se visualiza una
imagen alargada, sin sombra acústica, de un centímetro de longitud. En este
contexto, entre las siguientes alternativas, ¿Cuál es el parásito más probablemente
implicado?

A Fasciola hepática.
B Trichuristrichura.
C Strongiloidesstercolaris.
D Ascarislumbricoides.

164. Un paciente de 21 años de edad, con reciente diagnóstico de diabetes


mellitus dependiente de insulina. Acude a control y solicita información respecto a
los síntomas de hipoglicemia. De entre los siguientes síntomas o signos, ¿Cuál NO
es habitual en hipoglicemia?

A Bradicardia.
B Diaforesis.
C Hambre.
D Palidez.

165. Un niño de 12 meses presenta desde hace 48 horas fiebre y tos. Tiene el
esquema de vacunación completo para su edad. Al examen físico: frecuencia
respiratoria 50 rpm (respiración por minuto), frecuencia cardiaca 100 lpm (latidos por
minuto), temperatura 39 0C, retracciones subcostales, estertores y crepitantes
bilaterales. La radiografía de tórax revela múltiples infiltrados alveolares bilaterales y
neumatoceles. ¿Cuál sería el agente etiológico más probable?

A Micoplasmapneumoniae.
B Haemophilusinfluenzae.
C Estafilococo aureus.
D Estreptococo pneumoniae

166. Un niño de 3 años de edad acude a emergencia por presentar desde hace 2
días fiebre, vómito, irritabilidad. Hoy presenta crisis convulsivas. En el examen físico
se encuentra letargia, signo de Brudzinski positivo, ¿Cuál será el diagnóstico más
probable?
A Convulsiones febriles.
B Meningitis bacteriana.
C Intoxicación aguda.
D Deshidratación hiponatrémica

167. Un lactante mayor de 30 meses de edad acude a consulta con su madre; por
presentar desde hace 5 días rinorrea, inicialmente serosa luego amarilla y espesa.
Desde hace 2 días tiene fiebre y tos productiva que ha aumentado cada día. Hoy
amaneció con secreción ocular amarilla y densa. El niño está irritable, come poco,
tiene astenia. No tiene antecedentes quirúrgicos ni de hospitalizaciones. Peso= 13
kg; temperatura axilar: 38.3 °C; FC= 100 x 1''; FR= 32 x 1''. Presenta congestión
rinofaríngea y de membranas timpánicas, con ausencia de reflejo a la luz. Campos
pulmonares limpios. La biometría hemática presenta 11 000 leucocitos con
predominio de segmentados (78%). Los frotis de secreción ocular y nasal
demuestran la presencia de cocobacilos gramnegativos pleomórficos. ¿Cuál será el
germen involucrado más probable?

A Streptococcuspyogenes.
B Haemophilusinfluenzae.
C Pseudomonaaeruginosa.
D Staphylococcusaureus.

168. Mujer de 24 años, con un índice de masa corporal de 26 y antecedentes


heredofamiliares de diabetes mellitus II. Ha recibido recientemente cefalosporina de
2ª. generación por faringitis aguda. Acude por presentar desde hace tres días prurito
vulvovaginal importante, asociado a disuria terminal. Si usted sospechara
candidasisvulvovaginal, ¿Qué características debería tener la secreción?

A Amarillenta, homogénea, líquida, espumosa.


B Blanca, en acúmulos, adherente, que produce eritema.
C Blanca, escasa, en acúmulos, sin mal olor.
D Grisácea, homogénea adherente, mal olor.

169. Paciente de 52 años en buen estado de salud y sin antecedentes de


importancia, en menopausia de 20 meses. Consulta por presentar sofocos
frecuentes diurnos y nocturnos y resequedad vaginal que causa dispareunia, que
producen disminución de la calidad de vida y de su productividad. Además ha
desmejorado su relación familiar por cambios importantes de carácter, ¿Cuál sería la
mejor opción terapéutica para controlar los sofocos y resequedad vaginal?

A Venlafaxina.
B Fito estrógenos.
C Estrógenos.
D Ansiolítico

170. Mujer de 35 años de edad, divorciada, tiene un hijo de 7 años. Refiere que
desde hace 3 meses, empieza a dormir a su hora habitual y se despierta a las 2 ó 3
de la madrugada y no vuelve a dormir. Además cansancio matinal, irritabilidad, ha
perdido el gusto por realizar actividades que antes le agradaban, ha notado
disminución en atención, concentración y en memoria inmediata. En el último mes
además, ha presentado dolores articulares que cambian de localización en pocos
días. Es la primera ocasión que presenta un cuadro de estas características. ¿Cuál
de los siguientes fármacos le parece el más indicado?

A Amitriptilina.
B Fluoxetina.
C Zopiclona.
D Alprazolan.

171. ¿Cúal grupo farmacológico se usa como primera opción para el manejo
sintomático de un episodio de agitación psicomotriz de origen orgánico, una vez
identificada la causa de base?

A Anticonvulsivantes.
B Antipsicóticos.
C Antidepresivos.
D Benzodiacepinas.

172. Varón de 35 años de edad. Consulta por alteración del sueño, dice que desde
hace 3 meses, empieza a dormir a su hora habitual y se despierta a las 2 o 3 de la
madrugada y no vuelve a dormir. Refiere además cansancio matinal, irritabilidad,
perdida del gusto por realizar actividades que antes le agradaban, ha notado
disminución en atención, concentración y en memoria inmediata, ¿Cuál de los
siguientes fármacos le parece el más indicado?

A Clonacepan.
B E-Zopiclona.
C Fluoxetina.
D Amitriptilina.

173. En relación con el secreto profesional, todos los enunciados son verdaderos.
EXCEPTO:
El secreto médico es la categoría que se asigna a toda información que es revelada por un/a
A
usuario/a al profesional que le brinda la atención de salud.
Bajo cualquier circunstancia es obligación de los profesionales de la salud de
B establecimientos públicos o privados presentar una denuncia, al conocer la comisión de
un presunto delito.
No hay obligación de los profesionales de la salud de establecimientos públicos o privados, de
C denunciar un presunto delito cuando el conocimiento de los hechos esté amparado por el
secreto profesional.
Existen casos donde la ruptura del secreto profesional o el denunciar, no causan daño al
D
paciente, sino lo protegen de situaciones de vulnerabilidad e indefensión.

174. Señale el objetivo terapéutico para control de hiperlipidemia que debe tener
un paciente adulto con diabetes:

A Triglicéridos; 150mg/dl (1.7mmol/L).


B Lípidos LDL ; 100mg/dL (5.6 mmol/L
C Triglicéridos ; 100mg/dL (2.6 mmol/L
D Lípidos HDL ; 180mg/dL (10mmol/L

175. ¿Cúal es la combinación de insulina cuya duración efectiva es de 15 -18


horas?

A 75/25 (75 % de lisproprotamina y 25 % de lispro


B 50/50 (50 % de lisproprotamina y 50 % de lispro
C 70/30 (70 % de aspartprotamina y 30 % de aspart).
D 70/30 (70 % de NPHy 30 % de insulina simple

176. Una persona con diabetes que se controla con antidiabéticos orales e
insulina, por decisión propia dejó de administrarse la insulina desde hace 2
semanas. Usted recibe al paciente con cetoacidosis diabética y la respuesta
ventilatoria a la acidosis metabólica es la esperada, ¿Cuál es el límite inferior del pH
sanguíneo ante el cual se podría administrar bicarbonato de sodio por vía
endovenosa?

A 7,21.
B 7,11.
C 6,99.
D 7,35.

177. Un niño de 5 años de edad presenta súbitamente fiebre alta, cefalea, vómito y
convulsiones tónico - clónico generalizadas. En el examen físico se encuentra
rigidez de nuca. El examen de líquido cefalorraquídeo revela hipoglucorraquia,
proteinorraquia y pleocitosis con predominio de polimorfonucleares, ¿Cuál sería la
mejor alternativa terapéutica en este caso?

A Claritromicina + Amikacina.
B Cefotaxima + Vancomicina.
C Ampicilina + Gentamicina.
D Aciclovir + Ceftriaxon

178. Jorge de 52 años de edad acude al servicio de Emergencia porque desde


hace 2 horas presenta dolor retroesternal severo, punzante y que se irradia al brazo
izquierdo; con sensación de sofocación. Tiene antecedentes de ser fumador de 2
cajetillas diarias los últimos 10 años. Al examen físico presenta: presión arterial
150/90, frecuencia cardíaca 80 por minuto, frecuencia respiratoria 24 por minuto.
Seleccione al mejor indicador de que el paciente está presentando un infarto de
miocardio.

A Lactato deshidrogenasa.
B Creatinfosfoquinasaisoenzimas MB.
C Troponina I cardioespecífica.
D Electrocardiograma.

179. Un paciente varón de 72 años de edad con antecedentes de alcoholismo


crónico y cirrosis. Acude a emergencias por decaimiento, fiebre y dolor abdominal
de 2 días de evolución. Al examen físico, lo relevante es la temperatura de 38,8
grados centígrados y frecuencia cardiaca 100 lpm (latidos por minuto) y presencia
de ascitis, con dolor difuso a la palpación abdominal y peritonismo. La tomografía
descarta compromiso visceral abdominal. La punción peritoneal reporta líquido con
300 polimorfonucleares por microlitro. Entre los siguientes gérmenes, ¿Cuál NO está
implicado habitualmente en la etiología de este cuadro?

A Enterobacterias
B Anaerobios.
C Estafilococo.
D Estreptococo.

180. A su consulta es llevado un niño de 4 años por presentar un cuadro diarreico


de 2 días de evolución, sin moco ni sangre. Al examen físico se encuentra afebril, su
frecuencia cardiaca es de 85 latidos por minuto, frecuencia respiratoria de 30 por
minuto, y tensión arterial de 100/60 mmHg. Al examen físico se lo nota irritable e
intranquilo, no hay presencia de ojos hundidos, no presenta signo de pliegue
cutáneo. El abdomen es suave, depresible, los ruidos hidro-aéreos están
aumentados, no parece producirse dolor a la palpación profunda. Señale el manejo
del paciente:
A Plan A de rehidratación y prescribir Zinc.
B Plan B de rehidratación y prescribir antibiótico.
C Plan B de rehidratación y prescribir Zinc.
D Plan A de rehidratación y prescribir antibiótico.

181. Un adolescente de 12 años, presenta cefalea desde hace 3 días. El dolor se


acompaña de rinorrea y tos productiva de 10 días de evolución y de predominio
nocturno. Al examen físico goteo retrofaríngeo en moderada cantidad, auscultación
pulmonar normal. ¿Cuál sería la conducta más adecuada en este caso?

A Pedir una tomografía de cavum y senos paranasales.


B Solicitar una radiografía de senos paranasales.
C Iniciar antibioticoterapia empírica.
D Realizar un cultivo de la secreción retrofaríngea.

182. Señale la afirmación correcta en relación con el cáncer de cuello uterino:

A El piometra está presente en más de la mitad de las pacientes.


B El sangrado uterino anormal se observa en la mayoría de las pacientes.
C En etapas iniciales suele producir sintomatología urinaria y rectal.
D La secreción vaginal serosanguinolienta es infrecuente.

183. Los siguientes son factores de riesgo para presentar parto prematuro,
EXCEPTO:

A Hipertensión gestacional.
B Malformaciones congénitas en el feto.
C Rotura prematura de membranas.
D Antecedente de parto pretérmino.

184. ¿Cuál es el agente etiológico de mayor frecuencia en la mastitis puerperal?

A U. urealyticum.
B Escherichiacoli.
C Clostridiumsordellii.
D Staphylococcusaureus.
185. Mujer de 24 años, acude a sala de emergencia de hospital por presentar dolor
en hipogastrio, tipo cólico, de moderada intensidad, asociado desde hace 48 horas
con sangrado vaginal escaso. Sus antecedentes gineco-obstétricos son: gesta 4,
aborto 1, cesárea 2, ciclos regulares, amenorrea de 6 semanas. Exploración física:
signos vitales normales. Abdomen suave, doloroso en hipogastrio. Al examen
genital se constata el sangrado vaginal, cérvix cerrado, útero ligeramente aumentado
de tamaño, suave, regular, globoso, anexos no palpables. Dos exámenes de
gonadotropina coriónica con 48 horas de intervalo reportan: 1.800 y 4.000 UI/ml y la
progesterona está en 22 ng/ml, ¿Cuál de los siguientes es el diagnóstico más
probable?

A Amenaza de aborto.
B Embarazo ectópico.
C Aborto incompleto.
D Mola hidatiforme.
186. Señale un factor de riesgo de infecciones de sitio quirúrgico que depende del
paciente.

A Profilaxis inadecuada.
B Hipotermia, hipoxia.
C Operación reciente.
D Necrosis del tejido.

187. Señale un signo que represente déficit agudo de volumen extracelular:

A Hiperazoemia.
B Aumento de peso.
C Edema intestinal.
D Soplo.

188. Paciente acude con su hijo de 12 meses a vacunación en el centro de salud.


Le corresponden sarampión, rubeola, parotiditis (SRP) y fiebre amarilla. La paciente
indica a la enfermera que le administre solamente la vacuna de SRP. ¿Cuál es la
conducta a seguir en esta situación?

Se le brinda información sobre los beneficios y riesgos de la vacuna de fiebre amarilla y se le


A
solicita que regrese otro día una vez que reflexione sobre la información brindada.
Se le sugiere que acuda a un centro privado de vacunación para que le apliquen vacuna contra
B
la fiebre amarilla y que firme el consentimiento informado que rechaza el procedimiento.
Se le solicita que firme el consentimiento informado donde rechaza el procedimiento y no se le
C
aplica ninguna vacuna porque el esquema debe completarse estrictamente.
Brindar información sobre los beneficios y riesgos de la vacuna de fiebre amarilla, y firmar
D
el consentimiento informado donde rechaza el procedimiento y se aplica sólo SRP.

189. ¿Cuál es el principal pilar de tratamiento en la infección por Cólera?

A Acortar el período de la excreción bacteriana.


B Restablecer los fluidos y electrolitos.
C Administrar antibióticos como terapéutica inicial.
D Asegurar una nutrición adecuada.

190. Un recién nacido de 33 semanas 6 días de edad gestacional, sin antecedentes


prenatales de importancia, nace por parto céfalovaginal luego de amniorexis
instrumental. No alcanzó a recibir corticoides prenatales. Presenta desde el
nacimiento quejido, aleteo nasal, retracciones y cianosis. En el examen físico se
encuentra una frecuencia respiratoria de 80 lpm (latidos por minuto), crepitantes
bilaterales y mala entrada de aire. ¿Cuál es el diagnóstico más probable en este
caso?

A Neumotórax.
B Membrana hialina.
C Hernia diafragmática.
D Cardiopatía congénita.

191. Una niña de 5 años presenta dolor intenso de oído derecho de 24 horas de
evolución. La niña se encuentra febril. Al examen otoscópico neumático del oído
derecho se evidencia un eritema intenso de la membrana timpánica con
abombamiento moderado de la misma; además se evidencia líquido y niveles hidro-
aéreos detrás de la membrana timpánica. El diagnóstico correcto de la condición de
la paciente es:

A Otitis externa complicada.


B Otitis media aguda (OMA).
C Otitis media aguda (OMA
D Otitis media con derrame (OMD

192. Paciente de 4 años de edad, quien está conciente, afebril, presenta estridor
inspiratorio leve y tos seca "perruna". Indique el diagnóstico correcto:

A Neumonía.
B Bronquilotis.
C Crup moderado.
D Amigdalitis aguda.

193. Paciente masculino de 5 años, en consulta se detecta ascenso testicular


indoloro. Se logra bajar el testículo al escroto, sin embargo, el cordón espermático
se encuentra a tensión. Indique el diagnóstico:

A Criptorquidia.
B Torsión testicular.
C Testículo retráctil.
D Hernia inguinal.

194. Una niña de 5 años de edad sin antecedentes de importancia, presenta desde
hace 2 días fiebre, disuria y polaquiuria. Trae un examen de orina tomado previo
aseo genital y de la mitad de la micción, que revela nitritos y estereasa leucocitaria
positivos. En el urocultivo usted esperaría encontrar

A Shigella> 100.000 UFC/ml.


B Klebsiella 5.000 a 10.000 UFC/ml.
C E. Coli> 100.000 UFC/ml.
D Estreptococo pyogenes 10.000 – 100.000 UFC/ml.

195. A su consulta general acude un paciente de 15 meses de edad masculino con


su madre para vacunación, el paciente ha cumplido con el esquema de vacunación
vigente. ¿Cuál vacuna es la siguiente a administrarse según el Ministerio de Salud
Pública del Ecuador?

A Vacuna contra virus del papiloma humano (HPV


B Vacuna de difteria y tétano (Td
C Vacuna de poliomielitis (OPV
D Vacuna contra varicela.

196. Mujer de 34 años, acude a sala de emergencia de hospital por presentar dolor
en fosa iliaca derecha, de intensidad progresiva. Desde hace 48 horas presenta
sangrado vaginal intermitente, escaso. Sus antecedentes Gineco-obstétricos: gesta
4, aborto 1, cesárea 2, ciclos regulares, amenorrea de 6 semanas; y gonorrea hace 5
años. Exploración física: signos vitales normales. Abdomen suave, doloroso en
hipogastrio y fosa iliaca derecha. Al examen genital se constata el sangrado vaginal,
cérvix cerrado, útero ligeramente aumentado de tamaño, anexos no palpables, el
derecho sensible. La gonadotropina coriónica es positiva. ¿Cuál de los siguientes es
el diagnóstico más probable?

A Amenaza de aborto.
B Aborto incompleto.
C Embarazo ectópico.
D Mola hidatiforme.

197. Paciente de 32 años de edad, sin antecedentes personales de importancia,


G4 P3, acude al Hospital Gíneco Obstétrico Isidro Ayora con un embarazo de 32
semanas, cefalea, TA de 150/120, edema en piernas, cara y manos, ROTS 3/5. Sus
indicaciones de ingreso a la paciente incluyen los siguientes medicamentos.
EXCEPTO:

A Betametasona 12 mg IM 2 dosis cada 24 horas.


B Nifedipina 10 mg VO cada 20 minutos.
C Hidralazina IV bolo de 5 mg stat.
D Sulfato de magnesio IV 6 g de carga y 1 g/hora mantenimiento

198. Paciente de 49 años, con antecedentes de hipertensión arterial y diabetes


gestacional previa, en su único embarazo. Menstruaciones regulares y normales
hasta hace 3 años, en que se hicieron frecuentes y prolongadas, (3-4 semanas de
sangrado). Índice de masa corporal de 29,4, presión arterial de 146/92 mm Hg. Al
examen genital presenta sangrado uterino, el resto del aparato genital es normal,
¿Cuál de los siguientes procedimientos usaría inicialmente para manejar el caso?

A Papanicolaou.
B Legrado instrumental.
C Histeroscopia.
D Ecografía transvaginal.

199. Niño de 8 años, circuncidado, con antecedente de testículo en ascensor,


refiere dolor súbito a nivel del testículo izquierdo con aumento de volumen en esa
misma área. El examen físico muestra asimetría escrotal y reflejo cremasteriano
lento. ¿Cuál es la sospecha diagnóstica y qué prueba la confirma?

A Torsión testicular – Ecografía doppler.


B Priapismo – Tomografía computarizada.
C Orquiepididimitis – Ecografía simple.
D Parafimosis – Ecografía simple.

200. En la clasificación TNM para cáncer de próstata un estadio T3b se refiere a:


A El tumor es un hallazgo histológico fortuito en más del 5 % del tejido resecado.
B El tumor invade una o ambas vesículas seminales.
C Extensión extracapsular incluida la afectación microscópica del cuello de la vejiga.
D El tumor afecta a más de la mitad de un lóbulo, pero no a los dos lóbulos.

201. Varón de 57 años de edad, diagnosticado hace 12 años de hipertensión


arterial. Ha recibido varios tratamientos sin que se logre una estabilización de su
hipertensión. Como parte de la valoración para utilizar otro esquema, el cardiólogo le
solicita un examen de fondo de ojo. El resultado del mismo dice: “Presencia de
edema retiniano, manchas algodonosas y hemorragias, esclerosis y lesiones
espásticas de las arteriolas retinales, ¿A qué grado de retinopatía hipertensiva
corresponde este cuadro según la clasificación de Keith, Wagener y Barker?

A Grado IV.
B Grado II.
C Grado I.
D Grado III

202. En qué grupo poblacional se recomienda tratar siempre una bacteriuria


asintomática?

A Pacientes pediátricos.
B En todos los grupos poblacionales sin excepción.
C Embarazadas.
D Tercera edad.

203. La hipertensión portal es una de las primeras manifestaciones de una


patología cirrótica, pero también otras patologías prehepáticas e intrahepaticas
pueden causarla. En el tratamiento profiláctico para evitar hemorragia variceal indica
el uso de los betabloqueadores no selectivos (BBNS). Señale cual es un efecto
secundario al uso de estos medicamentos:

A Insuficiencia cardiaca.
B Impotencia.
C Enfermedad pulmonar obstructiva grave
D Bloqueo auriculo ventricular de II grado.

204. Se trata de un paciente de 33 años con antecedentes de una bulla (bula)


pulmonar derecha. Mientras realizaba una actividad física presentó un intenso dolor
en hemitórax derecho, acompañado de disnea de reposo y signos físicos de
neumotórax simple. La PaO2 está reducida, la PaCO2 elevada, pero su saturación de
oxígeno sube significativamente ante la administración de oxígeno suplementario,
¿Cuál es el principal mecanismo de hipoxemia en este paciente?

A Reducción de la superficie de intercambio.


B Incremento del espacio muerto fisiológico.
C Trastorno de la relación V/Q tipo “shunt”.
D Alteración intrínseca de la difusión.

205. Hombre de 69 años se presenta a consulta quejándose de dolor de pecho que


ha empeorado en las últimas semanas, el dolor se acompaña de episodios de
síncope, especialmente cuando realiza algún tipo de esfuerzo; también refiere
dificultad respiratoria durante las noches a la hora de acostarse. Al examen físico
presenta: P: 89/min, TA: 135/89 mmHg, FR: 16/min, T: 37 °C. La auscultación cardiaca
revela un soplo de tipo sistólico crescendo-decrescendo en foco aórtico con un
desdoblamiento paradójico de S2. ¿Cuál sería el diagnóstico?

A Insuficiencia Mitral
B Insuficiencia Tricuspídea
C Estenosis Pulmonar
D Estenosis Aórtica

206. Una paciente diagnosticada de anorexia nerviosa tiene en promedio de 8-13


episodios de conductas compensatorias inapropiadas por semana. Escoja la opción
que corresponde al nivel de gravedad de esta paciente.

A Leve.
B Moderado.
C Extremo.
D Grave.

207. A su consulta general acude un paciente de 2 meses, sexo masculino es


llevado para vacunación, el lactante ha sido vacunado con la BCG y con Hepatitis B
pediátrica al nacimiento. Indique el esquema que corresponde a la edad actual del
lactante

A Vacuna antipoliomielítica, Pentavalente, Influenza, Neumococo pediátrica.


B Vacuna antipoliomielítica, Pentavalente, Rotavirus, Influenza.
C Vacuna antipoliomielítica, Pentavalente, Fiebre Amarilla, Neumococo pediátrica
D Vacuna antipoliomielítica, Pentavalente, Rotavirus, Neumococo pediátrica.
208. Un lactante mayor de 24 meses de edad tiene un peso de 10kg, una talla de
70 cm y un perímetro cefálico de 43 cm. El peso está bajo el percentil 5, la talla en el
percentil 5 y el perímetro cefálico en el percentil 25. Al examen físico presenta
astenia, reducción del tejido celular subcutáneo y de las masas musculares.
Abdomen algo prominente. Piel seca, pelo escaso, descolorido, sin brillo y
fácilmente desprendible. No presenta visceromegalias. Si el percentil 50 del peso
para esta edad es de 12 kg, su pérdida de peso es de 16.6%. Seleccione el
diagnóstico más probable para este niño:

A Desnutrición calórica proteica de segundo grado.


B Desnutrición calórica proteica de primer grado.
C Desnutrición proteica calórica o kwashiorkor.
D Desnutrición calórica proteica de tercer grado.

209. Recién nacido de 37 semanas de edad gestacional y 3800 gramos de peso al


nacimiento. A las 4 horas de vida se lo encuentra con una frecuencia cardiaca de 120
latidos por minuto, frecuencia respiratoria de 50 respiraciones por minuto, temperatura de
36.8 oC; llenado capilar menor a 2 segundos, hipoactivo, tembloroso y con una succión
lenta. Los exámenes revelan: hematocrito 53 %, glicemia 30 mg/dl ¿Cuál es la causa de su
sintomatología?

A Hipoglicemia
B Poliglobulia
C Deshidratación
D Hipotermia

210. El maltrato infantil tiene un espectro clínico de presentación muy amplio,


¿Cuál es la causa más común de muerte por malos tratos físicos?

A Lesiones intraabdominales.
B Quemaduras extensas.
C Muerte por asfixia.
D Traumatismo craneoencefálico.

211. Paciente de 32 años, acude a control después de 15 días de parto vaginal


refiriendo malestar general, alza térmica no cuantificada y dolor en mama derecha
desde hace tres días. El embarazo y parto fueron normales. Al momento se
encuentra en lactancia. No tiene antecedentes importantes. Gesta 3, para 3. Signos
vitales normales, a excepción de temperatura de 38,1oC. Mamas hipertróficas, con
red de Haller evidente, con fisura en pezón derecho; en el cuadrante superoexterno
de mama derecha se observa piel eritematosa con piel de naranja de alrededor de 5
cm de superficie, bajo la cual se palpa masa dura, dolorosa, irregular de la misma
dimensión. En axila derecha se palpa ganglio linfático de 2 cm, regular, móvil,
doloroso. Mama izquierda normal. El resto del examen físico es normal, inclusive el
examen genital. ¿Cuál es el manejo más adecuado para este caso?

A Suspensión de la lactancia.
B Drenaje quirúrgico inmediato.
C Ceftriaxona 1 g. IM cada día/7 días.
D Dicloxacilina 500 mg VO cada 6 horas/7 días.

212. En las pacientes con diagnóstico de preeclampsia, ¿Cuál de los siguientes


signos es criterio de severidad?

A Transaminasa (GOT) 55 UI/ml.


B Proteinuria de 1 g en 24 horas.
C Creatinina de 1,3 mg/dL.
D Diuresis de 40 ml por hora.

213. Paciente de 38 años de edad, sin antecedentes médicos de importancia.


Desde hace seis días presenta estornudos y rinorrea inicialmente acuosa, bilateral,
que hace 24 horas se torna amarillenta. Se acompaña de hiposmia, goteo
retrofaringeo, tos irritativa y moderado dolor facial. Al examen físico, lo relevante es
la temperatura bucal de 37,8 grados. Según los lineamientos generales
recomendados, a más de un analgésico adecuado, ¿Cuál es la conducta a seguir?

A Mantener una conducta expectante con respecto a los antibióticos.


B Iniciar tratamiento empírico con un antibiótico de amplio espectro.
C Realizar tomografía de senos de la cara, antes de iniciar antibiótico.
D Realizar radiografía de senos de la cara, antes de iniciar antibiótico.

214. Varón de 43 años de edad, con diagnóstico de lumbalgia aguda. Como parte
del tratamiento no farmacológico, ¿Cuál de las siguientes medidas NO
recomendaría?

A Aplicación de frio local.


B Reposo máximo 2 días.
C Masajes.
D Aplicación de calor local.

215. Una mujer de 36 años, casada, empleada pública, presenta desde hace 7
meses episodios de taquicardia, falta de aire, temblor y mareo, que se repiten dos
veces al mes. Define a estos episodios como que “siente que va a morir”. Los
estudios de ECG y de hormonas tiroideas, son normales. Adicionalmente, tiene
miedo a salir de casa y evita ciertas situaciones específica. Cuando está en
compañía o se convence de que no pasará nada supera ese miedo. Al examen
psiquiátrico la paciente se manifiesta muy intranquila, pide ayuda al médico
constantemente, está orientada, no se objetivan alteraciones de la sensopercepción,
presenta ideas fijas con respecto a su padecimiento somático, tiene una leve
hipertimiadisplacentera y se encuentra eubúlica. Le administraron una
benzodiacepina y el episodio cede, aunque persiste el miedo a que se repita. ¿Cuál
es el diagnóstico de esta paciente?

A Trastorno de ansiedad generalizada.


B Trastorno de ansiedad social.
C Trastorno de pánico.
D Fobia específica.

216. Mujer de 45 años de edad, es llevada al servicio de urgencias por su esposo y


sus tres hijos. Relata que la parte distal de su pierna derecha está paralizada y que
todo su hemicuerpo derecho estaba demasiado pesado. Esta sintomatología se
había presentado de forma brusca pocas horas antes, a raíz de una discusión con la
hija mayor, cuyo novio no agrada a la paciente. En la exploración se encuentra buen
nivel de conciencia, no se encuentran déficits cognitivos y tiene angustia en relación
a sus síntomas físicos. Signos vitales en límites de normalidad, examen neurológico
normal, ¿Cuál es el diagnóstico más probable?

A Trastorno somatomorfo.
B Trastorno de adaptación.
C Trastorno de conversión.
D Reacción a estrés agudo.

217. Un hombre de 48 años acude a la consulta por cefalea fronto occipital y dolor
cervical desde hace más de una semana. Ha tomado antiinflamatorios, con alivio
parcial de sus molestias; en los últimos días ha tenido problemas para conciliar el
sueño y durante el día está de muy mal humor. Se divorció hace 4 meses y vive solo.
Trabaja como gerente de empresa desde hace varios años, su trabajo es de mucha
presión y el ambiente laboral se ha vuelto más difícil en las últimas dos semanas.
Fuma desde la juventud, 5-6 cigarrillos al día, alcohol social, no utiliza drogas, no
realiza ejercicio físico. Su padre era hipertenso y murió por un infarto a los 68 años.
Su madre es diabética y tiene sobrepeso. Una hermana mayor es hipertensa. En la
exploración física se encuentra: peso 76 kg, talla 1.63 m, IMC 28.6, PA 130/80, FC 84
x min, FR 16 x min, existe una contractura muscular dolorosa en la región cervical,
sin limitación de la movilidad. La actitud más importante a tomar es

A Enfatizar en el cambio en su régimen de vida y alimentación.


B Indicar una Monitorización Ambulatoria de la Presión Arterial (MAPA
C Iniciar medicación para controlar su presión arterial.
D Sugerir tomar un descanso de su trabajo durante unos días.

218. El régimen antibiótico de erradicación de cepas resistentes de Helycobacter


pylori es:

A Furazolidina + claritromicina + tetraciclina


B Amoxicilina + levofloxacina + tinidazol.
C Metronidazol + rifabutina + tinidazol.
D Rifabutina + Tinidazol + claritromicina.

219. En un paciente con cetoacidosis diabética, a más de la corrección de la


hiperglucemia, se deberán realizar las siguientes acciones. EXCEPTO:

A Medición y reposición de bicarbonato.


B Medición y reposición de potasio.
C Medición y reposición de calcio.
D Medición y reposición de fosfato.

220. Paciente adulto con antecedente de traumatismo hace dos semanas que
presenta Hb 16 g/dl, albúmina menor a 2.8 g /100 ml, linfocitopenia de 1500 u /L, con
deficiente cicatrizaciòn de heridas, desprendimiento fácil del cabello y edema,
estatura normal, ¿Cuál es el diagnóstico?

A Anemia.
B Cretinismo.
C Marasmo.
D Kwashiorkor.

221. Un paciente de 60 años, sin antecedentes de importancia. Durante el último


examen de control anual tuvo presencia de hematuria en la tira reactiva “dipstick”,
que fue confirmada con la presencia de 3 a 5 hematíes dismórficos, por campo de
alto poder. Se acompaña de proteinuria de más de 500 mg/dL y de cilindros con
eritrocitos. Con mayor probabilidad, ¿Cuál es la estructura afectada?

A Glomérulo.
B Túbulo distal.
C Túbulo colector.
D Túbulo proximal.
222. Se trata de un paciente joven con antecedentes de asma. Acude porque
presenta una nueva crisis, como parte del tratamiento usted administra dosis
repetidas de beta dos agonistas inhalados. De entre las siguientes alteraciones,
¿Cuál NO es atribuible a este fármaco?

A Hipokalemia.
B Hipomagnesemia.
C Hipofosfatemia.
D Hiponatremia.

223. Paciente de 17 años, acude al servicio de emergencia por dolor tipo cólico en
hipogastrio, de moderada intensidad, que aparece al inicio de la menstruación y se
asocia con lipotimia. La sintomatología se ha repetido en el último año casi con cada
menstruación, generalmente dura 36 a 48 horas. El dolor se alivia con medidas
generales. La paciente no ha iniciado su relación sexual y su menarquia fue a los 15
años. Sus ciclos son regulares desde hace 1 año. El examen físico es normal. En el
manejo de la paciente, ¿Cuál de las siguientes es la mejor opción?

A Laparoscopia diagnóstica.
B Observación y analgésicos.
C Antibióticos de amplio espectro.
D Realizar tomografía pélvica.

224. El enunciado correcto en referencia al parto pretérmino es:

A El incremento del pH vaginal previene el riesgo de parto pretérmino.


B La proteína C reactiva en frotis vaginal previene el riesgo de parto pretérmino.
C El uso de progesterona aumenta la incidencia de parto pretérmino.
D Las modificaciones cervicales valoradas por ecografía, son un marcador predictor.

225. ¿Qué mecanismo fisiológico en el embarazo produce incremento de la


excreción urinaria de proteínas?

A Disfunción endotelial glomerular.


B Disminución de la reabsorción tubular de las proteínas filtradas.
C Disminución de la tasa de filtración gomerular.
D Alteración del sistema renina-angiotensina-aldosterona.
226. Adolescente femenina de 14 años de edad, refiere que desde hace 3 días
presenta dolor en su oído derecho, el cual ha ido aumentando progresivamente de
intensidad, además desde la noche de ayer presenta alza térmica a 40oC. Su médico
realiza examen otoscópico, y encuentra una moderada supuración de color
amarillento. En un principio prescribe paracetamol 500 mg y amoxicilina 500 mg
cada ocho horas durante tres días. Ante la persistencia de la sintomatología después
de cuarenta y ocho horas, sugiere cambiar el esquema antibiótico. ¿Cuál sería el
antibiótico de elección?

A Azitromicina.
B Cotrimoxazol.
C Ceftriaxona.
D Cefuroxima.

227. Señale en qué caso se respeta la objeción de conciencia del médico:

A Suicidio médicamente asistido.


B Petición expresa y reiterada del paciente.
C Padecimientos graves insoportables.
D Enfermedad clínica irreversible e incurable.

228. Una paciente es sometida a legrado instrumental y pese a que el


procedimiento se desarrolló sin complicaciones, reclama al médico por no haber
sido informado previamente de la necesidad y riesgos del procedimiento, ¿Cuál de
los principios de la bioética no respetó el médico?

A De autonomía.
B De no maleficiencia.
C De beneficiencia.
D De justicia.

229. Paciente de 38 años, sin antecedentes de interés, refiere que desde hace 3
días presenta sensación distérmica, tos con expectoración blanquesina y malestar
general. El paciente no ha utilizado antibióticos durante el último año por ninguna
causa. Al examen físico se encuentra alerta, febril (38.9°C), frecuencia cardíaca de 90
lpm, frecuencia respiratoria de 25 por minuto, tensión arterial 120/80 mmHg, no
cianosis ni signos de dificultad respiratoria. En la base pulmonar derecha se
ausculta estertores basales. La radiografía de tórax muestra una opacidad no
segmentaria en el lóbulo inferior del pulmón derecho con presencia de broncograma
aéreo. ¿Cuál es el tratamiento antibiótico empírico inicial para éste paciente?
A Moxifloxacino 400 mg por vía oral una vez al día durante 5 días.
B Levofloxacino 750 mg por vía oral una vez al día durante 5 días.
C Doxiciclina 100 mg cada 12 horas por vía oral durante 5 días.
D Amoxicilina 1 gr más claritromicina 500 mg por vía oral cada 12 horas durante10 días.

230. Durante el transcurso de una enfermedad crónica un paciente debe recibir


fármacos que son excretados por vía renal. Clásicamente se utiliza la ecuación de
Cockroft-Gault para calcular el aclaramiento (clearence) de creatinina. Ese método
permite obtener rápidamente un valor aproximado del aclaramiento de creatinina,
¿Cuál de los siguientes parámetros NO toma en cuenta la ecuación mencionada?

A Edad del paciente.


B Creatinina sérica.
C Volumen urinario.
D Peso del paciente.
231. A nivel del mar, durante la atención emergente de un adulto previamente
sano, hospitalizado por un evento agudo, usted realiza una gasometría arterial. Los
resultados son: pH: 7,32, PaCO2: 50 mmHg, PaO2: 86 mmHg, HCO3: 24 mEq/L,
SaO2: 95%, ¿Cuál es la interpretación?

A Alcalosis respiratoria.
B Acidosis metabólica.
C Acidosis respiratoria.
D Alcalosis metabólica.

232. Un paciente joven, por lo demás sano, que vive en la costa, tuvo
antecedentes de insuficiencia respiratoria, ocasionada por edema pulmonar de
altura, hace 2 años. Por razones laborales, requiere trasladarse a una altura de 3.500
metros sobre el nivel del mar. De entre las siguientes alternativas, ¿Cuál fármaco es
el MENOS indicado como profilaxis para este problema?

A Hidralazina.
B Nifedipina.
C Sildenafil.
D Acetazolamida

233. Un niño de 18 meses de edad, tuvo un resfriado común hace 5 días y desde
hace 48 horas presenta fiebre de 38 C e irritabilidad. Al realizar la otoscopia se
encuentra el tímpano eritematoso, abombado y no se visualiza el triángulo luminoso.
¿Además de recibir tratamiento sintomático, cuál sería la mejor alternativa
antimicrobiana?
A Ciprofloxacino.
B Amoxicilina.
C Gentamicina.
D Dicloxacilina.

234. Una madre en período de lactancia ha sido sometida a una extracción


dentaria bajo anestesia local. El odontólogo le prescribe ibuprofeno y amoxicilina
luego del procedimiento. El niño tiene 4 meses de edad y se alimenta
exclusivamente con seno, ¿Cuál sería la conducta más adecuada en este caso?

A Suspender el analgésico a la madre.


B Continuar con la lactancia materna.
C Administrar fórmula al bebé.
D Adelantar el inicio de la ablactación.

235. Un paciente de 3 años de edad tuvo hace 6 días un resfrío que se resolvió
espontáneamente. Desde hace 2 días presenta unas lesiones vesiculares no
dolorosas ni pruriginosas alrededor de las fosas nasales, las cuales se encuentran
cubiertas de un exudado amarillento, de aspecto parecido a la miel. ¿Cuál sería la
conducta más adecuada?

A Referir al dermatólogo.
B Cultivar el exudado de las lesiones.
C Administrar dicloxacilina.
D Solicitar biometría hemática

236. Una bebé de 21 días de vida, es llevada a un control de niño sano. Tiene
antecedentes familiares de displasia de caderas. ¿Cuál de los siguientes hallazgos
en el examen físico le permitiría diagnosticar con mayor precisión displasia del
desarrollo a esta edad?

A Maniobra Ortholani Positiva.


B Acortamiento de pierna.
C Signo de Galeazzi positivo.
D Signo de Trendelemburg.

237. Una mujer de 60 años de edad, vendedora y consumidora de fritada, con


antecedentes de estreñimiento crónico, acude por un dolor abdominal en fosa ilíaca
izquierda que progresivamente se torna intenso en el lapso de 12 horas y se
acompaña de fiebre. Al examen físico la presión arterial: 140/90mmHg, frecuencia
cardiaca: 98 lpm (latidos por minuto), frecuencia respiratoria: 20 rpm (respiraciones
por minuto), temperatura de 38,4 oC, ruidos cardio respiratorios normales, abdomen
suave, depresible, doloroso a la palpación en fosa ilíaca izquierda. El examen de
orina es normal, en la biometría encontramos leucocitos de 12.500/mm3, con 80% de
segmentados y 20% de linfocitos, la química sanguínea reporta urea 24 mg/dL y
creatinina 0,9 mg/dL. ¿Cuál es el examen de elección en estas condiciones?

A Laparoscopia diagnóstica.
B Enema de bario doble contraste.
C Colonoscopia.
D Tomografía contrastada.

238. Indique. ¿Cuál es el componente de la solución salina?

A Solución salina 0.9% contiene 109 mEq/L de cloro.


B Solución salina al 0.9% contiene: 154 mEq/L de sodio.
C Solución salina al 0.9% contiene: 4 mEq/L de sodio.
D Solución salina 0.9% tiene un pH de 8.

239. Mujer de 54 años, acude a emergencias por presentar desde hace dos días
fiebre de 39° C y dolor en hipocondrio derecho, al inicio tipo cólico, irradiado a
hombro derecho. También refiere náusea y vómito. La paciente tiene un índice de
masa corporal de 29, frecuencia cardíaca 110 latidos por minuto. Ruidos hidroaéreos
presentes pero disminuidos. Hemiabdomen superior doloroso, especialmente bajo
borde costal derecho a la inspiración profunda. La biometría hemática reporta
leucocitos de 12.150/ mm3, con neutrófilos de 82%, ¿Cuál de los siguientes
exámenes le ayudaría a confirmar el diagnóstico?

A Prescribir tomografía abdominal.


B Indicar colangio resonancia.
C Endoscopia digestiva alta.
D Solicitar ecografía abdominal.

240. Un varón de 56 años de edad, sin antecedentes relevantes, tiene el


diagnóstico de hipertrofia prostática benigna y recibe tamsulosina, ¿Cuál de los
siguientes efectos adversos aparece con relativa frecuencia, cuando se utiliza este
fármaco?

A Caída del cabello.


B Hipotensión ortostática.
C Insuficiencia hepática.
D Hipoglicemia nocturna.

241. Son factores de riesgo para la demencia tipo Alzheimer. EXCEPTO:

A Hipertensión arterial.
B Tabaquismo.
C Sexo masculino.
D Edad avanzada.

242. Relacione las siguientes categorías y los procesos en relación con el consentimiento
informado (CI):

a. El profesional de la salud, basándose en la


1. Niños/as mayores de 12 años y
valoración clínica, actuará en consideración al
adolescentes.
interés superior y beneficio del paciente.
b. Uno de los abuelos del menor, bajo cuya
2. Padres menores de edad emancipados. patria potestad viva, podrá suscribir el
consentimiento informado.
c. El consentimiento podrá ser suscrito por los
3.Menor de edad de padres menores de edad
mismos, tanto para intervenciones clínicas en
no emancipados.
ellos o en sus hijos.
d. Informar al menor verbalmente el
procedimiento médico que se le va a realizar,
4. Disputa entre representantes legales de utilizando términos sencillos, claros y con
menores de edad. calidez. Solicitar además consentimiento
informado escrito de los padres o
representantes legales en caso de requerirse.

A 1d, 2b, 3c, 4a.


B 1d, 2c, 3b, 4a.
C 1a, 2c, 3d, 4b.
D 1a, 2c, 3b, 4d.

243. Señale el esquema de tratamiento de la crisis asmática:

Agonistas B2 de corta duración ; corticoideo nebulizado; corticoide intravenoso en


A
bolo.
B Agonista B2 de larga duración ; dosis bajas de corticoide inhalado.
C Dosis bajas de corticoide inhalado.
D Agonistas B2 de larga duración ; dosis altas de corticoide inhalado ; corticoide oral.

244. ¿Cuál es el diagnóstico de un paciente que presenta eritema en forma de


parches en cuero cabelludo, la frente, pliegue nasolabial y conducto auditivo externo
con descamación y prurito?

A Dermatitis de contacto alérgica.


B Dermatitis seborreica.
C Dermatitis atópica
D Psoriasis.

245. Los siguientes hongos producen micosis superficiales en la cara. EXCEPTO:

A Dermatofitos.
B Especies de Malassezia.
C Especies de Cándida.
D Esporotricosis.

246. Seleccione los criterios que corresponden a un caso de dengue sin signos de
alarma: IgM positiva. Exantema. Sangrado de mucosas. Leucopenia. Elevación de
AST mayor o igual a 1 000. Prueba de torniquete positivo.

A b, d, e.
B a, d, f.
C c, d, f.
D b, d, f.

247. Un niño de 5 años de edad presenta súbitamente fiebre alta, cefalea, vómito y
convulsiones tónico - clónico generalizadas. En el examen físico se encuentra
rigidez de nuca. El examen de líquido cefalorraquídeo revela hipoglucorraquia,
proteinorraquia y pleocitosis con predominio de polimorfonucleares. ¿Cuál sería la
mejor alternativa terapéutica en este caso?

A Claritromicina + Amikacina.
B Cefotaxima + Vancomicina.
C Ampicilina + Gentamicina.
D Aciclovir + Ceftriaxona.

248. Una mujer de 68 años de edad fue gastrectomizada por cáncer gástrico hace
2 meses, y hace un mes, tuvo un evento de tromboembolismo pulmonar que fue
tratado con anticoagulantes. Recibe tratamiento ambulatorio regular para su cáncer
y las condiciones clínicas de la paciente son estables. Si se mantienen estables las
condiciones, ¿Por cuánto tiempo deberá tomar la medicación anticoagulante?

A Por un año más.


B Indefinidamente.
C Por seis meses más.
D Por dos meses más.

249. Un paciente varón de 56 años de edad con antecedentes desde hace 2 años
de varios episodios de pérdida súbita y transitoria de consciencia, con recuperación
espontánea y completa a los pocos segundos, en cada ocasión. Acude a su consulta
por un nuevo episodio ocurrido 1 hora antes. La presión arterial: 110/60 mmHg,
frecuencia cardiaca: 78 lpm (latidos por minuto), frecuencia respiratoria: 14 rpm
(respiraciones por minuto), temperatura 36,8 oC, sin déficit motor, la auscultación
pulmonar es normal, a la auscultación cardiaca se encuentra un soplo sistólico en
diamante, grado II, en el segundo espacio intercostal derecho, irradiado a vasos del
cuello. En el electrocardiograma se aprecian complejos QRS de 110 milisegundos
con onda S de 26mm en V1 y V2 con R de 22 mm y onda T asimétricas negativas en
V5-V6. ¿Cuál es la probabilidad del riesgo de mortalidad para este paciente en los
dos próximos años?

A Del 1 al 5%.
B Del 8 al 15%.
C Del 40 al 100%.
D Del 20 al 35%.

250. Un lactante de 1 año que pesa 9 kg y vive en Quito acude con su madre para
control de niño sano. Se encuentra en buen estado general. Trae consigo los
resultados de una biometría: Leucocitos 8 500 /mm3; Eritrocitos: 3 980 000 / mm3;
Hematocrito (Hcto): 30%; Hemoglobina (Hb): 10 g/dl; Volumen Corpuscular Medio
(VCM): 71 fL; Concentración de Hemoglobina Corpuscular Media (CHCM): 23%;
Morfología eritrocitaria: hipocromía. Los exámenes de orina y heces son normales.
Seleccionar el diagnóstico y el tratamiento apropiado para este paciente:

Anemia normocíticanormocrómica; Hierro elemental de 1 – 2 mg/kg/día, dos días a la semana


A
para prevenir una anemia grave.
Anemia macrocíticahipocrómica; Ácido fólico 1 mg, diario durante 1 mes, luego evaluación para
B
verificar la respuesta al tratamiento.
Anemia microcíticahipocrómica; Sulfato ferroso de 4 – 6 mg/kg/día, dividido en 2 o 3 dosis al
C
día, inicialmente durante un mes.
Anemia microcíticahipocrómica; hierro elemental, 4 – 6 mg/kg/día, dividido en 2 o 3 tomas,
D
durante un mes, inicialmente.

251. Un paciente de 12 meses de edad presentó hace 2 días rinorrea y tos


“perruna”. Hoy presenta cianosis y disfonía. Al examen físico se encuentra estridor
en reposo y retracciones subcostales ¿Cuál es el diagnóstico en este caso?

A Bronquiolitis
B Neumonía
C Bronquitis
D Laringitis

252. En la Clave Azul obstétrica el manejo correcto de impregnación con sulfato


de magnesio para la prevención de las convulsiones es:

A Administrar 5 ampollas al 20 % + 400 ml de solución isotónica pasar a 50 ml/hora.


B Administrar 1 gramo/hora IM de SO4MG en ampollas al 20 %
C Administrar 2 ampollas 20 % IM de SO4Mg con un contenido total de 4 gramos.
D Administrar 2 ampollas 20 % IV de SO4Mg + 80 ml solución isotónica a 101 gotas/minuto.

253. Las caracteristicas de la triada de Beck son, EXCEPTO:

A Hiperresonancia a la percusión.
B Disminución de la presión arterial.
C Elevación de la presión venosa.
D Ruidos cardiacos apagados.

254. Varón de 19 años, presenta desde hace dos días fiebre, mialgias, disfonía,
odinofagia y tos. Los síntomas se incrementan por lo cual acude a consulta. En el
examen se encuentra temperatura de 40oC, frecuencia cardiaca 100 latidos por
minuto, frecuencia respiratoria 18 respiraciones por minuto, rinorrea serosa,
orofaringe congestiva con exudado blanco nacarado y adenopatías retroauriculares.
Auscultación respiratoria normal. ¿Cuál sería la mejor conducta en este caso?

A Cultivo de secreción faríngea.


B Tratamiento sintomático.
C Antibioticoterapia empírica.
D Pruebas antigénicas rápidas.

255. Paciente varón de 65 años, bebedor habitual, que se presenta con intenso
dolor epigástrico después de una comida abundante, describe al dolor como
“puñalada”, con irradiación a la espalda y que se alivia cuando se inclina hacia
adelante. Refiere náusea y vómito abundantes. Al examen físico presenta FC: 112
latidos por minuto, TA: 90/60 mmHg, temperatura: 38,8 grados C; el abdomen se
observa distendido, con defensa involuntaria y dolor al palpar todo el hemiabdomen
superior, los ruidos hidroaéreos están disminuidos. En los exámenes de laboratorio
se encuentra hemoconcentración, hiperglucemia, hiperazoemia e hiperamilasemia.
Se realizó ecografía que no dio información útil por la presencia de abundantes
gases intestinales. Se solicitó TAC que se muestra en la imagen. ¿Cuál es el
diagnóstico a plantear en este paciente?

A Pancreatitis aguda necrosante grave.


B Pancreatitis aguda edematosa leve.
C Úlcera péptica gastroduodenal perforada.
D Apendicitis aguda complicada.

256. ¿Qué se observa en la obesidad central?

A Triglicéridos disminuidos.
B Colesterol HDL elevado.
C Resistencia a la insulina.
D Colesterol LDL disminuido.

257. El hallazgo clínico más habitual en los casos de neumonía infantil es:

A La presencia de estridor.
B La presencia de taquipnea.
C La presencia de auscultación pulmonar anormal.
D La presencia de tos.

258. Una paciente de 27 años, previamente sana. Luego de un viaje vacacional


presenta distensión abdominal con dolor cólico y flatulencia, con episodios
intermitentes de diarrea acuosa. Al examen físico lo llamativo son los borborigmos y
distensión abdominal. No tiene fiebre. El examen coproparasitario fue negativo y la
biometría es normal. De entre los siguientes, ¿Qué parásito es el más probablemente
implicado?

A Estrongiloides.
B Giardia.
C Ameba.
D Áscaris.

259. Se trata de una paciente de 24 años de edad, con antecedentes de epilepsia y


pesa 60 Kg. Una hora antes de su ingreso presentó un “ataque” convulsivo
generalizado, luego de 15 minutos repitió otra crisis convulsiva que duró unos 3
minutos y actualmente presenta dos crisis convulsivas seguidas, razón por la que
ingresa a emergencias. Usted decide administrar un fármaco intravenoso. En estas
condiciones, y entre las siguientes alternativas, ¿Qué fármaco escogería usted, y
cuál es la dosis más apropiada?

A Midazolam de 0,6 a 1,2 miligramos.


B Diazepam de 1,4 a 1,8 miligramos.
C Midazolam de 18 a 22 miligramos.
D Diazepam de 5 a 10 miligramos.

260. En pacientes con trauma, en la valoración primaria, en la sección de


circulación. Identifique la primera lesión más importante que pone en riesgo la vida:

A Hemotórax masivo.
B Hemoperitoneo masivo.
C Fracturas pélvicas.
D Taponamiento cardíaco.

261. Paciente masculino de 33 años con trabajo extenuante que le ocasiona


estrés, con antecedentes de enfermedad ulcerosa péptica, presenta vómitos de
contenido no bilioso y alcalosis metabólica intensa con hipopotasemia. Indique a
que complicación de la enfermedad ulcerosa péptica se refiere el enunciado.

A Úlcera péptica perforada.


B Obtrucción pilórica.
C Gastritis crónica.
D Síndrome de Zollinger - Ellison.
262. Un hombre de 44 años de edad llega por primera vez a su consulta. En la sala
de espera se le solicita que llene varios formularios, con datos de su historial
médico familiar y personal, ante lo cual el paciente asiente amablemente. En la
consulta, el médico constata que los documentos están vacíos, a pesar de que el
paciente ha tenido el tiempo necesario para llenarlos. Al preguntarle ¿por qué no
llenó los documentos? el paciente dice en tono sarcástico: estoy seguro que un
médico con su fama y educación puede hacer una historia clínica. El médico le
pregunta al paciente si todo está bien, con lo que el paciente responde: “Si, todo
está de maravilla, no hay ningún problema”. Indique, ¿Cuál de las siguientes
opciones describe la actitud del paciente?

A Desplazamiento.
B Agresión pasiva.
C Proyección.
D Comportamiento impulsivo.

263. ¿Cuál de las siguientes situaciones representa una complicación grave en la


agitación psicomotriz?

A Hiperpatía.
B Hiperventilación.
C Somnolencia.
D Ataque de pánico.

264. Varón de 23 años de edad, padece de obesidad desde hace 6 años y no tiene
antecedentes de alteraciones mentales. Desde hace dos semanas no sale de su
habitación, manifiesta estar triste y preocupado porque ha escuchado que sus
vecinos planifican matarlo. Refiere que espían sus movimientos y además saben lo
que él piensa, ¿Cuál sería su tratamiento de elección?

A Fluoxetina 20 mg/día.
B Risperidona 6 mg/día.
C Olanzapina 15 mg/día.
D Ácido Valproico 1 gr/día.

265. En relación con el secreto profesional, confidencialidad y privacidad en


adolescentes, se deben tomar en cuenta las siguientes consideraciones. EXCEPTO:

La protección de la confidencialidad es un componente esencial de la atención de salud para


A
Adolescentes, ya que es compatible con el desarrollo progresivo de su madurez y su autonomía.
Los adolescentes deben ser informados sobre políticas de confidencialidad del servicio al que
B
asisten, incluyendo las circunstancias en las que puede ser necesario romper la confidencialidad.
Los profesionales de la salud deben garantizar a los/las adolescentes la confidencialidad de la
C
información que les ha sido revelada en la consulta.
Para la prestación de servicios de salud a los/las adolescentes siempre es necesaria la
D
Presenciade uno de los padres o el representante legal.

266. Una mujer de 18 años de edad, por primera vez acude por lesiones faciales.
Usted encuentra comedones abundantes y pocas pápulas pequeñas eritematosas,
algo dolorosas, diseminadas en frente, nariz y mejillas. A más de las medidas
higiénicas usuales. ¿Cuál es el tratamiento de elección?

A Isotretinoina por vía oral.


B Progesterona en micro dosis.
C Peróxido de benzoilo.
D Tetraciclinas.

267. Un paciente varón de 67 años, con sobrepeso, es diagnosticado de diabetes


mellitus. Se realizaron las recomendaciones pertinentes de ejercicio, dieta y se
prescribió un hipoglicemiante oral. Luego de pocos días de iniciado el tratamiento,
el paciente presentó ansiedad, escalofrío, sudoración, piel fría y palpitaciones. ¿Cuál
de los siguientes hipoglicemiantes orales podría relacionarse, con mayor frecuencia,
con la sintomatología descrita?

A Glibenclamida.
B Metformina.
C Poglitazona.
D Acarbosa.

268. Una paciente de 32 años, sin antecedentes de importancia, es traída al


servicio de emergencias por alteración aguda del estado de conciencia, no se
conocen los detalles. Al examen físico lo relevante es un estado confusional y una
respiración profunda, pero rítmica. La gasometría arterial reporta: pH 7,18, PaO2 de
100 mm Hg, respirando por catéter nasal a 2 litros/minuto, PaCO2: 23 mm Hg, HCO3:
10 mEq/L. En cuanto al equilibrio ácido base, ¿Cuál es el diagnóstico?

A Alcalosis metabólica.
B Acidosis metabólica.
C Alcalosis respiratoria.
D Acidosis respiratoria.
269. Acude a su consulta un paciente varón de 58 años, fumador de 15 cigarrillos
diarios desde los 20 años de edad hasta hace 3 años, con disnea progresiva desde
hace 10 años, que al momento es de medianos esfuerzos. Desde hace 2 días
presenta rinorrea acuosa y pocos estornudos. Al examen físico presión arterial:
120/70 mmHg, frecuencia cardiaca: 80 lpm (latidos por minuto), frecuencia
respiratoria: 24 rpm (respiraciones por minuto), temperatura bucal 36,7 oC, peso 60
Kg, talla 166 cm, tórax en tonel, percusión hiperresonante bilateral, sus ruidos
pulmonares y cardíacos son hipofonéticos, sin ruidos sobreañadidos. ¿Qué
volumen o capacidad pulmonar estará incrementado?

A Volumen residual.
B Capacidad inspiratoria.
C Capacidad vital.
D Volumen de reserva espiratoria.

270. Un varón de 19 años de edad previamente sano, acude por presentar desde
hace una semana, malestar general y dolor con edema en la muñeca y en el tobillo
del lado izquierdo, además ha presentado odinofagia y fiebre de 5 días de evolución.
Al examen físico presión arterial: 110/70 mmHg, frecuencia cardiaca: 110 lpm
(latidos por minuto), frecuencia respiratoria: 22 rpm (respiraciones por minuto),
saturación de oxígeno 94% respirando aire ambiente, temperatura bucal 38,7 oC, se
ausculta un soplo holosistólico grado II ubicado en punta, que se incrementa en
espiración, ruidos respiratorios y abdomen normales. Al electrocardiograma se
evidencia ritmo sinusal con intervalo PR de 260 milisegundos. En sangre la
biometría demuestra unos leucocitos de 12.000/mm3 con 78% de segmentados, la
velocidad de sedimentación es de 30 mm/h y los títulos de antiestreptolisina O están
en 320 Unidades/ml. ¿Qué fármaco está recomendado como parte de la primera línea
de tratamiento en este paciente?

A Metilprednisolona 2 gramos diarios.


B Diclofenac sódico 50 miligramos cada 12 horas.
C Aspirina de 80 a 300 miligramos diarios.
D Prednisolona 60 miligramos diarios.

271. A su consulta acude un paciente de 3 años de edad por presentar tos desde
hace 4 días. Al examen físico el paciente se encuentra febril (38 grados C),
frecuencia cardiaca de 95 por minuto, frecuencia respiratoria de 52 por minuto y
saturación de oxígeno 93%. No se evidencia estridor y a la auscultación pulmonar no
presenta disminución de murmullo vesicular ni sibilancias. El diagnóstico del
paciente es:

A Resfriado común
B Crup viral.
C Neumonía.
D Bronquitis aguda

272. El tratamiento de una criptorquidea verdadera en un lactante de 9 meses de


edad es:

A Tratamiento expectante hasta el año de edad


B Inducción hormonal para descenso testicular
C Orquidopexia
D Tratamiento expectante hasta los 3 años de edad

273. Acude una paciente de 9 años de edad en compañía de su madre en busca de


continuar con su esquema de vacunación, a sabiendas que la paciente ha sido
inmunizada correctamente hasta la fecha. ¿Cuál de las siguientes vacunas esta
indicada en esta niña según el Esquema Nacional de Vacunación del Ministerio de
Salud Pública del Ecuador

A Vacuna de fiebre amarilla (FA


B Vacuna contra virus del papiloma humano (HPV).
C Vacuna de varicela.
D Vacuna de difteria y tétano (Td

274. Una adolescente cuya menarquia se presentó hace 2 años, consulta por
palidez, cansancio y anorexia. Al examen se aprecia palidez palmar y un soplo
sistólico en mesocardio grado 2/6. La biometría hemática revela un recuento
eritrocitario disminuido y glóbulos rojos hipocrómicos y microcíticos. El recuento
leucocitario y el plaquetario son normales, ¿Qué prueba de laboratorio solicitaría
para establecer el origen de esta anemia?

A Electroforesis de hemoglobina.
B Recuento de reticulocitos.
C Aspirado medular.
D Dosificación de ferritina.

275. Niño de 4 años de edad es llevado a consulta por presentar fiebre de 38 °C


desde hace 2 días acompañada de rinorrea y tos. Al examen físico tenemos paciente
febril con astenia pero que responde a estímulos; en la boca se observan unas
manchas pequeñas de color rojo con una base de coloración azul grisáceo; también
rash maculo-papular que a la digitopresión desaparece en la cara; el resto del
examen es normal. Madre refiere que tiene en casa una hija de 10 meses de edad y
que luego de una reacción adversa a la vacuna de la influenza, ella y su esposo
decidieron no administrar vacunas a sus hijos. ¿Cuál vacuna debe ser administrada
a la hermana de 10 meses inmediatamente?

A Vacuna de la varicela.
B Vacuna de sarampión.
C Vacuna para rubeola.
D Vacuna de influenza.

276. Una vez que la hemoragia postparto inmediata ha sido identificada, indique el
tratamiento farmacológico de primera opción y la dosis correcta.

Oxitocina 10Ul/mLlM (o 5 UllV lento), o 20-40 Ul en 1000 mL de solución cristaloide en


A
InfusiónIV a 250 mL/h.
Acidotranexamico 500mg intravenoso a pasar diluido en 200 ml de solución fisiológica y luego
B
500 mg intramuscular cada 8 horas.
C Misoprostol, 200 mcg VO, 1er minuto posparto, 300 mcg cada hora por 6 horas.
D Ergonovina 0,2 mg IM, 1er minuto, Ergonovina repetir cada 2 a 4 horas, máximo 5 dosis (1 mg

277. Mujer de 27 años, soltera, gesta 0, con múltiples parejas sexuales, algunas de
ellas ocasionales. Acude a su consulta porque desea cambiar el preservativo como
método anticonceptivo ya que ha escuchado de su eficacia baja. ¿Cuál de los
siguientes métodos está contraindicado por la posibilidad de infección pélvica?

A Hormonales combinados inyectables.


B Dispositivo intrauterino con cobre.
C Dispositivo intrauterino con progestágeno.
D Anticonceptivos orales combinados.

278. En los pacientes con diabetes mellitus tipo 2 (DM2), se recomienda la


siguiente distribución de los tipos de grasa dietaria considerando el total de calorías
diarias. EXCEPTO:

A Ácidos grasos trans1 %.


B Ácidos grasos poli insaturados10 %.
C Ácidos grasos mono insaturados 12-20 %.
D Ácidos grasos saturados 30-45 %.

279. Mujer de 17 años consulta por falta de menstruaciones normales. La


anamnesis revela menarquia a los 14 años, la paciente realiza 3 horas de ejercicio
diario. Tiene un IMC de 17.5 y caracteres sexuales secundarios normales. El eco
pélvico no revela ninguna anormalidad. ¿Cuál es el siguiente paso a seguir?

A Aplicar los criterios de Rotterdam.


B Medir niveles de TSH.
C Dosificación de CA 125.
D La paciente debe dejar de hacer tanto deporte y/o aumentar su ingesta calórica.

280. Mujer de 35 años con antecedente de cistitis aguda hace dos semanas
tratada con antibióticos. Acude por presentar desde hace una semana prurito
vulvovaginal, asociado a disuria terminal. Al examen físico se encuentra sobrepeso,
y en la exploración genital se aprecia eritema en vulva y vagina, con leucorrea
blanca en acúmulos, adherente a paredes vaginales. Tiene antecedentes
heredofamiliares de diabetes mellitus II. En el examen de laboratorio de la secreción
vaginal, ¿Qué debería encontrar?

A pH> 4.5, polimorfonucleares normales, células clave, test de aminas +.


B pH<4.5, polimorfonucleares normales, bacilos Gram positivos.
C pH<4.5 polimorfonucleares aumentados, bacilos Gram positivos e hifas.
D pH> 4.5, polimorfonucleares aumentados y microrganismos móviles.

281. Mujer de 35 años, cursa con embarazo de 32 semanas, acude al servicio de


emergencia por presentar desde hace dos horas, sangrado vaginal rojo de moderada
a abundante cantidad, que apareció luego de relación sexual. Antecedentes Gineco-
obstétricos: gestas 5, partos 2, cesárea 0, abortos 2, sin control prenatal. Al examen
físico: signos vitales normales, fondo uterino corresponde a amenorrea, feto único,
transverso, frecuencia cardíaca fetal 150 lpm (latidos por minuto), tono uterino
normal. ¿Cuál de los siguientes procedimientos diagnósticos estaría
contraindicado?

A Monitoreo fetal electrónico.


B Resonancia magnética.
C Tacto vaginal.
D Ecografía obstétrica.

282. Paciente masculino de 60 años con antecedentes de estreñimiento y


obesidad acude por cuadro de dolor abdominal en el cuadrante inferior izquierdo
que se acompaña de astenia y alza térmica. Usted solicita una tomografía de
abdomen en la que se evidencia una diverticulitis del colon sigmoideo asociado a un
absceso menor a 2 cm. El tratamiento es:
A Drenaje percutáneo.
B Antibioticoterapia.
C Laparotomía.
D Laparoscopia y drenaje de absceso.

283. Un paciente varón de 55 años, con sobrepeso, presenta polidipsia y poliuria.


En este caso, ¿Cuál de los siguientes resultados en sus pruebas de sangre
confirmarían el diagnóstico de diabetes mellitus?

A Cifra de hemoglobina glicosilada A1c de 5,6%.


B Test de tolerancia a la glucosa de 142mg/dL.
C Nivel de glicemia tomada al azar de 210 mg/dL.
D Dos valores de glicemia en ayunas de 112 mg/dL.

284. Paciente de 40 años refiere fiebre, sudoración nocturna, pérdida de peso y


disnea de un tiempo evolución de 2 meses. A la exploración física TA: 120/80,
FC:80lpm, Sat O2: 96%, Tº: 38 ºC, dolor torácico en hemitórax anterior derecho,
murmullo vesicular disminuido en base derecha. En la radiografía de tórax se
observa radiopacidad homogénea unilateral derecha, borramiento del ángulo
costofrénico derecho de borde cóncavo sin presencia de condensaciones. ¿Cuál es
el diagnóstico?

A Neumonía bacteriana.
B Tuberculosis pleural.
C Asma persistente.
D Abseso pulmonar.

285. Paciente de 38 años de edad, sometida a colecistectomía convencional de


vías biliares hace 5 años, acude con masa tumoral en región abdominal que
disminuye de tamaño con el reposo; le preocupa porque va aumentando de tamaño
progresivamente. Al examen físico presenta: masa supraumbilical media de
aproximadamente 15 cm de diámetro, blando, no doloroso, reductible; defecto
aponeurótico de 7 cm de diámetro. Es diagnosticada de hernia incisional. Durante la
evaluación preoperatorio el examen de sangre demuestra: hemoglobina (Hb) 6 g/L,
hematocrito (Hcto) 24%, plaquetas 100 000, tiempo de protrombina (TP) 11
segundos, tiempo parcial de tromboplastina (TPT) 35 segundos. Como preparación
para la cirugía, seleccione la decisión más acertada:

Sangre total, hasta elevar la Hb a 10 g/dL, porque tiene los elementos sanguíneos
A
completos.
B Concentrados de plaquetas para incrementar su número por trombocitopenia existente.
Eritrocitos congelados, hasta obtener Hb de 10 g/dL y por tener mayor porcentaje de glóbulos
C
rojos.
D Concentrados de eritrocitos porque es el producto preferido y no sobrecarga el volumen.

286. Paciente de 28 años, primigesta, con embarazo de 39 semanas, al tacto


vaginal presenta cuello uterino de consistencia blanda, posición anterior, con
dilatación de 3 cm, borramiento del 60% y la presentación del feto OIIA se encuentra
en el III plano de Hodge (localización -1). ¿A qué índice de Bishop corresponde e
indique si es favorable o desfavorable para inducción de trabajo de parto?

A 8 puntos, desfavorable.
B 2 puntos, favorable.
C 3 puntos, desvaforable.
D 10 puntos, favorable.

287. En una paciente con Clave Azul obstétrica. ¿Cuál es un signo inminente de
intoxicación por sulfato de magnesio y su manejo médico?

A El aumento del reflejo rotuliano, administrar hidralazina vía parenteral.


B El aumento de la presión arterial. Administrar nifedipina vía oral.
C La desaparición de reflejos rotulianos, gluconato de calcio al 10%.
D El aumento de las contracciones uterinas, administrar gluconato de calcio al 20%.

288. El mecanismo correcto de acción de los anticonceptivos a base de


progesterona es:

A Provocan un espesamiento del moco cervical.


B Evitan la ovulación al suprimir liberación de FSH.
C Aumentan la motilidad de las trompas de Falopio
D Generan un endometrio secretor.

289. Señale el antibiótico de primera elección usado en la profilaxis de la


Colecistectomía.

A Cefuroxima.
B Cefazolina más metronidazol.
C Ampicilina sulbactam.
D Cefazolina.
290. Varón de 53 años de edad, sin antecedentes de importancia. Al
despertar en la mañana presenta en forma brusca sangrado profuso de las fosas
nasales, que no cede a la presión digital, por lo que su esposa procede a colocar un
tapón de algodón. Quince minutos después al no ceder la hemorragia, deciden
acudir a un servicio de urgencias. Una vez allí, el médico al retirar los restos de
algodón evidencia la persistencia del sangrado. Al realizar el examen no logra
localizar el sitio de la hemorragia, ¿Cuál de los siguientes procedimientos sería de
primera elección?

A Ligadura arterial.
B Cauterización con nitrato de plata.
C Taponamiento posterior.
D Taponamiento anterior.

291. José tiene 39 años, se encuentra desempleado, su hijo nació con una
malformación de la oreja y es motivo de discusiones en su hogar, ha estado triste
por más de 4 meses; ya no sale con sus amigos que era algo que le gustaba mucho,
acude a la consulta solicitando ayuda porque ha pensado que sería mejor si
estuviera muerto, sabe que su pensamiento es malo, pero esa idea es recurrente.
Seleccione la severidad de riesgo suicida en este caso:

A Extremo.
B Grave.
C Moderado.
D Leve.

292. De los siguientes casos, seleccione aquel que requiere la firma del
consentimiento informado.

A Procedimientos de reproducción asistida.


B Situaciones de emergencia.
C Tratamientos exigidos por la ley como en pandemias.
D Papanicolaou para despistaje de lesiones preinvasoras.

293. Los siguientes parámetros, según las tablas de Framingham, se utilizan para
calcular el riesgo cardiovascular. EXCEPTO:

A Diabetes.
B Presión arterial.
C Tabaquismo.
D Colesterol LDL.

294. Una paciente de 58 años de edad, sin antecedentes de importancia. Está


asintomática, pero durante los controles médicos subsecuentes, usted verifica que
las cifras de presión arterial de la paciente están en 140/98 mm Hg, y decide iniciar
terapia farmacológica. De entre las siguientes alternativas farmacológicas, ¿Cuál
tiene MENOR impacto sobre la mortalidad, morbilidad y el riesgo de insuficiencia
cardiaca?

A Calcioantagonistas.
B Diuréticos.
C Alfabloqueantes.
D Betabloqueantes.

295. Un varón de 20 años de edad, residente en Riobamba, con antecedentes de


diabetes mellitus tipo I, desde los 14 años, tuvo amigdalitis aguda hace 3 días.
Desde hace 48 horas presenta nausea, vómito, poliuria, sed intensa y dolor
abdominal. Al examen físico algo somnoliento pero orientado, presión arterial 110/70
mmHg, frecuencia cardiaca 120 lpm (latidos por minuto), frecuencia respiratoria 18
rpm (respiraciones por minuto), con respiraciones profundas, orofaringe congestiva,
sin alteraciones en examen torácico cardiopulmonar, abdomen doloroso de manera
difusa, ruidos hidroaéreos presentes. En laboratorio glicemia 300mg/dL, gasometría
arterial pH: 7,16, PaCO2: 18mmHg, HCO3: 8mEq/L, sodio 144 mEq/L, potasio sérico
6mEq/L. En orina se descarta infección, pero tiene cuerpos cetónicos ++++. ¿Cuál es
la enzima cuya actividad incrementada, directamente deriva en la producción de
cuerpos cetónicos?

A Lipasa de lipoproteína.
B Cinasa de piruvato.
C Palmitoiltransferasa de carnitina I.
D Fructosa 2-6 fosfatasa.

296. Una paciente de 35 años de edad, sin antecedentes patológicos de


importancia. Hace 7 días tuvo una infección bacteriana aguda de las vías
respiratorias altas y está recibiendo tratamiento con ampicilina 500 miligramos cada
6 horas y paracetamol, 1 gramo cada 8 horas. Actualmente acude a su consulta
porque presenta anorexia y náusea, aunque sin vómito, además ha notado un tinte
rojizo en la orina. Al examen físico la presión arterial es de 120/80 mmHg, frecuencia
cardíaca 80 lpm (latidos por minuto), frecuencia respiratoria 16 rpm (respiraciones
por minuto), temperatura bucal 36.8 oC y edema palpebral bilateral. El examen de
sangre reporta 7.500 leucocitos/mm3, 60% de neutrófilos, 20% de linfocitos, 14% de
eosinófilos y 6% de monocitos, glicemia 80 mg/dL, creatinina de 2,6 mg/dL y urea de
26 mg/dL. Solicita un examen de orina que reporta: densidad 1.015, pH 5.5, color
rojizo, ligeramente turbio, proteínas 30 mg/dL, hemoglobina positivo, piocitos 20 por
campo, presencia de cilindros granulosos, cilindros eritrocitarios y leucocitarios,
además, se observan leucocitos polimorfonucleares eosinófilos. La fracción
excretada de sodio es mayor a 1%. El valor de complemento C3 es normal. ¿Cuál es
el diagnóstico más probable?

A Necrosis tubular aguda.


B Glomérulonefritis postinfecciosa.
C Embolismo renal agudo.
D Nefritis intersticial.

297. A su consulta es llevado un niño de 4 años por presentar un cuadro diarreico


de 2 días de evolución, sin moco ni sangre. Al examen físico se encuentra afebril, su
frecuencia cardiaca es de 85 latidos por minuto, frecuencia respiratoria de 30 por
minuto, y tensión arterial de 100/60 mmHg. Al examen físico se lo nota irritable e
intranquilo, no hay presencia de ojos hundidos, no presenta signo de pliegue
cutáneo. El abdomen es suave, depresible, los ruidos hidro-aéreos están
aumentados, no parece producirse dolor a la palpación profunda. Señale el manejo
del paciente:

A Plan A de rehidratación y prescribir Zinc.


B Plan B de rehidratación y prescribir antibiótico.
C Plan B de rehidratación y prescribir Zinc.
D Plan A de rehidratación y prescribir antibiótico.

298. Paciente femenina de 60 años es diagnosticada de carcinoma endometrial


tipo 1. En su historia ginecológica se encuentra que tuvo 3 gestas con 2 partos y un
aborto. Menarquia a los 12 años. Inició su menopausia a los 56 años. Tiene obesidad
y diabetes mellitus. Ha fumado desde los 22 años una media cajetilla diaria. De los
siguientes, todos son factores de riesgo para cáncer endometrial excepto: Opciones
de respuesta a. Obesidad b. Tabaco c. Diabetes mellitus d. Menopausia tardía

A Respuesta incorrecta 1.
B Respuesta incorrecta 2.
C Respuesta correcta B.
D Respuesta incorrecta 3.

299. Paciente que presenta un accidente de tránsito, es llevado a la sala de


emergencia con disnea, dolor torácico, taquicardia y ausencia de ruidos
respiratorios en hemitórax derecho, usted sospecha de un neumotórax a tensión.
¿Cuál es el sitio en el que se debe colocar la aguja de descompresión en esta
patología?
A Segundo espacio intercostal, línea medio clavicular del hemitórax afectado.
B Cuarto espacio intercostal, línea medio clavicular del hemitórax afectado.
C Segundo espacio intercostal, línea axilar anterior del hemitórax afectado.
D Sexto espacio intercostal, línea axilar anterior del hemitórax afectado.

300. Se trata de una paciente de 26 años, previamente sana y con vida sexual
activa. Refiere que desde hace doce horas siente escozor intenso en el ojo
izquierdo, que a momentos es ardor, y que se acompaña de enrojecimiento ocular.
Al examen físico lo relevante en el ojo afectado es el edema inflamatorio palpebral,
quemosis conjuntival, la presencia de abundante secreción espesa, amarillo
verdosa, que persiste, ¿Cuál de los siguientes gérmenes está implicado con mayor
probabilidad, en la etiología del presente caso?

A Clamidia.
B Tricomona.
C Neisseria.
D Herpesvirus.

301. Señale que examen determina una extensión extracapsular (local) de un


cáncer de próstata:

A Gammagrafía ósea.
B Biopsia de próstata.
C Ecografía transrectal.
D Uretrocistoscopia.

302. Paciente masculino de 40 años, con antecedente de alcoholismo, el cual


presenta fiebre, tinte ictérico, taquipnea, con eritema palmar, temblor en manos, con
manos en puño y lenguaje agresivo y soez. ¿Què fármaco se debe suministrar?

A Haloperidol 5mg, vía Intramuscular, repitiendo cada 30 minutos, dosis máxima 20 mg/día.
B Paracetamol 1 gr Intravenoso STAT, luego cada 8 horas.
C Midazolam 5 mg intravenosos, repitiendo la dosis a los 30 minutos hasta suministrar 15 mg/día.
D Quetiapina 25 mg, vía oral, repitiendo la dosis a los 30 minutos, dosis máxima 50 mg/día.

303. Una persona de 24 años de edad, previamente sana, es rescatada después de


haber permanecido 3 días sin beber agua ni ingerir alimentos. En los exámenes de
laboratorio usted encuentra la creatinina sérica de 3 mg/dL. ¿Cuál de las siguientes
alternativas tiene los valores urinarios que son concordantes con el cuadro descrito
para esta persona?

A Osmolaridad menor a 300 mosm /L y sodio mayor a 40 mmol/L.


B Osmolaridad mayor a 600 mosm /L y sodio mayor a 40 mmol/L.
C Osmolaridad mayor a 600 mosm/L y sodio menor a 10 mmol/L.
D Osmolaridad menor a 300 mosm /L y sodio menor a 10 mmol/L.

304. Una niña de 7 años de edad acude hoy a emergencias debido a la presencia
de cefalea y hematuria. Hace 3 semanas tuvo una piodermitis tratada con
antibioticoterapia de forma satisfactoria. En el examen físico se encuentra: presión
arterial 150/105 mm Hg, frecuencia cardiaca 110 lpm (latidos por minuto), frecuencia
respiratoria 26 rpm (respiraciones por minuto) y edema pretibial. El elemental y
microscópico de orina, revela hematíes campo lleno, cilindros hemáticos, leucocitos
3-4 por campo y proteinuria. El C3 se encuentra bajo. ¿Cuál de las siguientes
opciones sería el diagnóstico más probable?

A Cistitis hemorrágica.
B Glomerulonefritis.
C Enfermedad de Berger.
D Síndrome nefrótico.

305. Un lactante de 13 meses de edad. Peso: 8 kg; Talla: 73 cm; PC: 44 cm. Acude
a emergencia por haber presentado desde hace 4 horas, dos crisis convulsivas
generalizadas, una que duró 15 minutos y la otra 20 minutos, aproximadamente.
Luego de las convulsiones permanece aletargado y aparentemente dormido. Su
madre refiere que desde hace 48 horas presenta fiebre, cada vez más alta, astenia,
anorexia y siente algo de rigidez en sus músculos. Se encuentra adormecido, muy
poco reactivo a los estímulos, signos de Brudzinski y Kernig positivo. Babinski
bilateral, positivo. Una tía del lactante tiene epilepsia. Con estos datos. Seleccione el
diagnóstico más probable

A Meningitis.
B Convulsiones febriles.
C Convulsiones complejas por fiebre.
D Crisis convulsiva de origen epiléptico.

306. En un embarazo pretérmino, que cursa con acortamiento de la longitud


cervical y contracciones uterinas, el fármaco de primera elección para maduración
fetal es:

A Betametasona 12 mg IM cada 12 horas, 2 dosis.


B Dexametasona 6 mg IM cada 24 horas, 2 dosis.
C Betametasona 12 mg IM cada 24 horas, 2 dosis.
D Dexametasona 6 mg IM cada 12 horas, 2 dosis.

307. Carolina de 24 años de edad presenta fiebre de 39°C tres días después de
haber sido sometida a cesárea. Su hijo nació en buenas condiciones, sin signos de
enfermedad y se encuentra en buen estado de salud. Se palpa el fondo uterino
blando y doloroso y se aprecian loquios de mal olor. Al tacto vaginal presenta dolor
en cuello que se incrementa con los movimientos. Se solicita biometría hemática
que reporta: hemoglobina 11 g/dl, leucocitos 12500 (segmentados 75%, linfocitos
25%); análisis de orina normal. ¿Cuál de los siguientes esquemas antibióticos es el
más adecuado para el tratamiento de la endometritis?

A Ampicilina - Sulbactam
B Metronidazol – Clindamicina
C Clindamicina - Gentamicina
D Gentamicina - Ampicilina

308. En la pelvis materna los límites del estrecho superior incluyen:

A Atrás limitado por el promontorio del sacro y a los lados por la cresta pectínea.
B A los lados limitado por las alas del sacro y adelante por la cresta pectínea.
C A los lados limitado por promontorio del sacro y adelante por la sínfisis del pubis.
D Atrás limitado por la cresta pectínea y adelante por la sínfisis del pubis.

309. Indique la afirmación correcta en relación al manejo de conjuntivitis.

A La conjuntivitis viral responde a los agentes antibacterianos.


B El uso de corticoides puede empeorar una infección por herpes virus.
C La conjuntivitis por molusco contagioso, responde adecuademente a antivirales.
D La conjuntivitis bacteriana leve debe ser tratada con antibióticos pues no se autolimita.

310. Las siguientes patologías son parte del diagnóstico diferencial de las
demencias. EXCEPTO:
La característica esencial del trastorno delirante es la presencia de uno o más
delirios, que persisten durante al menos un mes. Aparte del impacto directo
producido por los delirios, el deterioro del funcionamiento psicosocial puede estar
más circunscrito que en otros trastornos y el comportamiento no es
manifiestamente extravagante o extraño. Las funciones ejecutivas del paciente
están conservadas.
311. Los siguientes enunciados en relación a Herpes Zóster. EXCEPTO:

Si la función inmunitaria disminuye luego de la infección primaria, el virus se reactiva en los ganglios
A
sensitivos, desciende por los nervios y se multiplica.
Se caracteriza por disestesias unilaterales, erupción vesicular en la piel, en el trayecto del dermatoma
B
o los dermatomas inervados por el ganglio sensitivo correspondiente.
Durante la infección primaria, el virus de varicela-zóster infecta los ganglios sensitivos y persiste en
C
éstos, en forma latente, durante el resto de la vida.
D La administración de la vacuna de virus vivo atenuado contra el virus Herpes zóster (VZV)

312. Paciente de 30 años, refiere disuria y secreción uretral purulenta de 3 días de


evolución. En el examen físico se objetiva abundante secreción uretral purulenta e
inflamación del meato uretral. La tinción de Gram muestra abundantes leucocitos
polimorfonucleares y diplococos Gram-negativos. Señale el tratamiento empírico de
elección para este paciente:

A Ceftriaxona 250 mg intramuscular; Azitromicina 1 g vía oral, ambas en dosis única.


Ceftriaxona 250 mg intramuscular una sola dosis &#43; Metronidazol 500 mg vía oral por 14
B
días.
Ceftriaxona 250 mg intramuscular en una sola dosis &#43; Clindamicina 300 mg vía oral por
C
7 días.
D Gentamicina 160 mg intramuscular una sola dosis &#43; Azitromicina 500 mg vía oral por tres dias
313. Un paciente de 9 meses de edad presenta hace 3 días fiebre muy alta,
rinorrea y tos no productiva. Hoy acude a consulta por presentar un exantema que
inició en el pecho. Al examen físico: consciente, temperatura 37oC, FR: 36 rpm
(respiraciones por minuto), FC: 90 lpm (latidos por minuto), exantema macular
rosado, no confluente en el tórax y cuello, ¿Cuál es el diagnóstico más probable?

A Roseola.
B Sarampión.
C Varicela.
D Rubeola.

314. Un niño de 10 años de edad presentó hace 2 días rinorrea y tos. Hoy presenta
fiebre alta, dolor torácico, tos húmeda. Al examen presenta dificultad respiratoria y
taquipnea, ¿Cuál de los siguientes NO es un criterio para la selección de la
antibioticoterapia empírica en este caso?

A Grupo de edad.
B Análisis del esputo.
C Epidemiología local.
D Hallazgos radiográficos.
315. Usted se encuentra en un hospital de tercer nivel de atención y recibe una
paciente de 24 años con un embarazo de 33 semanas más ruptura prematura de
membranas confirmada, sin dinámica uterina. ¿Cuál es el protocolo a seguir?

A Profilaxis antibiótica, reposo en cama, maduración pulmonar y enviar a su casa.


B Hospitalización, reposo en cama, profilaxis antibiótica y maduración pulmonar.
C Uso de antibiótico, maduración pulmonar, uso de tocolíticos y referir a otro centro asistencial.
D Hospitalización, reposo en cama, uso de tocolíticos y uso de antibióticos.

316. ¿Cúal grupo farmacológico se usa como primera opción para el manejo
sintomático de un episodio de agitación psicomotriz de origen orgánico, una vez
identificada la causa de base?

A Anticonvulsivantes.
B Antipsicóticos.
C Antidepresivos.
D Benzodiacepinas.

317. Paciente mujer de 30 años de edad, sin antecedentes de importancia, que


luego de discusión familiar, presenta de manera brusca cefalea intensa y náusea
persistente. Tres horas más tarde acude a consulta y lo relevante en el examen
físico es que tiene presión arterial de 150/100 mm Hg, frecuencia cardiaca: 86 latidos
por minuto, algo somnolienta y dolor intenso al intentar flexionar el cuello. La
conducta más apropiada será:

A Administrar analgesia, prescribir reposo absoluto y realizar tomografía.


B Tranquilizar a la paciente, administrar antihipertensivos y realizar tomografía.
C Tranquilizar a la paciente, solicitar apoyo psicológico y reevaluar en 48 horas.
D Administrar antiagregantes plaquetarios más analgesia y tratar con antihipertensivo.

318. Paciente de 29 años sin antecedentes de interés, refiere que desde hace 3
días presenta dolor en zona vulvar y vaginal. No síntomas miccionales. En el examen
físico se objetivan en pubis y vagina lesiones vesiculares múltiples de 1 a 2 mm de
diámetro de bordes eritematosos y adenopatías inguinales bilaterales dolorosas a la
palpación. No se observa secreción vaginal ni uretral ¿Cuál es el tratamiento
indicado?

A Aciclovir 400 mg vía oral tres veces por 10 a 14 días.


B Azitromicina 1 gramo vía oral por una sola dosis.
C Doxiciclina 100 mg vía oral dos veces al día por 21 días.
D Penicilina benzatínica 1.2 millones de unidades intramuscular unidosis.

319. Se trata de una paciente de 61 años de edad con obesidad grado II. Acude
porque presenta fiebre y dolor en su región inguinal derecha. Al examen físico tiene
temperatura bucal de 38,8 grados y se aprecia una zona eritematosa, tumefacta,
caliente y dolorosa, a nivel de su región inguinal derecha que se extiende hasta el
flanco del mismo lado. Para determinar si tiene fasceitis necrotizante, de entre las
siguientes pruebas que pueden alterarse, ¿Cuál es la prueba más específica?

A Creatinfosfokinasa.
B Interleukina 6.
C Procalcitonina.
D Proteína C reactiva.

320. Carlitos es un niño de 5 años que hace una semana estuvo con resfriado
común. Hace 24 horas presenta primer evento de dolor en el oído derecho, está muy
molesto, no ha hecho fiebre, come bien, pero en algunos momentos ha llorado por el
dolor. Al examen físico se evidencia la membrana timpánica derecha ligeramente
eritematosa y abombada, la membrana izquierda ligeramente eritematosa. Escoja la
opción correcta para el manejo clínico de Carlitos.

A Dar azitromicina a 10 mg/kg/día el primer día y luego 5mg/kg/día por 4 días más.
B Dar amoxicilina a altas dosis 80-90mg/kg/día cada 12 horas por 10 días más acetaminofén.
Dar ibuprofeno a 10mg/kg/dosis cada 8 horas y esperar 4 días para reevaluar si la molestia no
C
ha cedido.
D Dar amoxicilina más ácido clavulánico 80-90 mg/9-15 mg /kg/día cada 8 horas más acetaminofén cada

321. Una mujer de 17 años llega a la consulta preocupada ya que todas sus
amigas han menstruado menos ella. Durante la anamnesis se queja también de no
percibir olores. El examen físico demuestra una clara falta de desarrollo de
caracteres sexuales secundarios. ¿Cuál es el diagnóstico?

A Síndrome de Kallman.
B Síndrome de Turner.
C Retraso fisiológico.
D Disgenesia gonadal pura.

322. Llega a la consulta una mujer de 27 años con un retraso en su menstruación


de 2 meses. La anamnesis revela un aumento de 5 kg durante 6 meses anteriores
(IMC 27), sequedad de piel, intolerancia al frío e irritabilidad. Durante el examen
físico no se evidencias signos alarmantes. Una prueba de BhCG es negativa. Con su
posible diagnóstico según la clínica. ¿Cuál es el tratamiento?

A Bromocriptina.
B Pastillas anticonceptivas orales.
C Metformina.
D Levotiroxina.

323. La colangitis aguda se caracteriza por una triada clásica (Triada de Charcot).
Indique sus componentes: a. Ictericia. b. Alteración mental. c. Dolor. d. Fiebre. e.
Choque séptico.

A c, d, e.
B a, c, d.
C b, c, d.
D a, c, e.

324. Relacione los tipos de demencias con sus características:


Tipos de Demencias Caracteristicas
1. Demencias Corticales a.Amnesia, afasia, apraxia, agnosia
2. Demencias Subcorticales b.Apatía, trastornos ejecutivos
c.Daño en Sustancia Blanca profunda
d.Déficit en la neurotransmisión colinérgica

A 1cd, 2ab
B 1ac, 2bd
C 1bd, 2ac
D 1ad, 2bc

325. Paciente de 38 años de edad, sin antecedentes médicos de importancia.


Desde hace seis días presenta estornudos y rinorrea inicialmente acuosa,
bilateral, que hace 24 horas se torna amarillenta. Se acompaña de hiposmia,
goteo retrofaringeo, tos irritativa y moderado dolor facial. Al examen físico, lo
relevante es la temperatura bucal de 37,8 grados. Según los lineamientos
generales recomendados, a más de un analgésico adecuado, ¿Cuál es la
conducta a seguir?
Mantener una conducta expectante con respecto a los antibióticos.

326. El tratamiento de la Esporotricosislinfocutánea es:

Literal Opción
A La terbinafina.
B Itraconazol.
C Anfotericina B.
D Fluocitosina.

327. De acuerdo a la valoración con ESCALA DE SARNAT Y SARNAT, cual es la


conducta y estadio correcto.
I. EHI leve
II. EHI moderada
III. EHI grave

1. Hipervigilia, Respuesta exagerada a los estímulos


2. Letargo Estupor, Respuesta tardía e incompleta a los estímulos sensoriales
3. Coma, Respuesta sólo a estímulos fuertes

A. Succion Débil
B. Succion Débil o ausente
C. Succion Ausente

A c) I,3, C - II,1, A - III,2, B


B d) I,1, B - II,3, B - III,3, A
C b) I,2, B - II,3, C - III,1,
D a) I,1, A - II,2, B - III,3,

328. Un escolar de 8 años, pesa 21 kg y vive en el subtrópico. Desde hace 1


semana presenta diarreas que contienen moco y estrías sanguinolentas. La cantidad
es muy variable y se repiten de 4 a 5 veces al día. Se acompañan de dolor abdominal
moderado. Hace un año presentó una sintomatología similar, aunque en aquella
ocasión tuvo prolapso rectal. El coproparasitario reporta presencia de huevos de
Trichuristrichiura. Seleccione el tratamiento adecuado:

A Albendazol, 400 mg diarios por 2 días.


B Albendazol, 400 mg diarios por 1 día.
C Albendazol, 400 mg diarios por 3 días.
D Mebendazol, 100 mg diarios por 3 días.

329. Un lactante mayor de 30 meses de edad acude a consulta con su madre; por
presentar desde hace 5 días rinorrea, inicialmente serosa luego amarilla y espesa.
Desde hace 2 días tiene fiebre y tos productiva que ha aumentado cada día. Hoy
amaneció con secreción ocular amarilla y densa. El niño está irritable, come poco,
tiene astenia. No tiene antecedentes quirúrgicos ni de hospitalizaciones. Peso= 13
kg; temperatura axilar: 38.3 °C; FC= 100 x 1''; FR= 32 x 1''. Presenta congestión
rinofaríngea y de membranas timpánicas, con ausencia de reflejo a la luz. Campos
pulmonares limpios. La biometría hemática presenta 11 000 leucocitos con
predominio de segmentados (78%). Los frotis de secreción ocular y nasal
demuestran la presencia de cocobacilos gramnegativos pleomórficos. ¿Cuál será el
germen involucrado más probable?

A Streptococcuspyogenes.
B Haemophilusinfluenzae.
C Pseudomonaaeruginosa.
D Staphylococcusaureus.

330. Paciente gestante a término con reporte ecográfico de placenta adyacente al


orificio cervical interno sin sobrepasarlo. ¿Qué tipo de anomalía de inserción
placentaria presenta esta paciente?

A Placenta previa parcial.


B Placenta previa oclusiva total.
C Implantación baja de la placenta.
D Placenta previa marginal.

331. Varón de 55 años de edad. Consulta por presentar disuria, dolor supra
púbico, pélvico, perineal, acompañado de fiebre escalofríos, mialgias y malestar
general. Usted sospecha prostatitis y para apoyar su diagnóstico solicita exámenes,
¿Cuál de los siguientes exámenes y/o procedimientos NO se recomienda?

A Ultrasonido pélvico.
B Tacto rectal.
C Urocultivo y hemocultivo.
D Antígeno prostático específico.

332. Señale qué principio de la bioética permite al paciente rechazar un


tratamiento

A No maledicencia.
B Justicia.
C Autodeterminación.
D Beneficencia.

333. Paciente de 7 años diagnosticado con faringoamigdalitis aguda, el médico


prescribe un antibiótico intramuscular para su infección. La madre solicita un
antibiótico por vía oral para su hijo, pero el médico desecha esa posibilidad,
argumentando que es mejor para el niño recibir el tratamiento en un solo pinchazo.
Escoja el modelo de relación médico-paciente correspondiente a este caso clínico.

A Informativo.
B Autonomista.
C Paternalista.
D Interpretativo.

334. Una mujer de 18 años de edad, con epilepsia, tiene un cuadro de acné
moderado a severo que afecta cara, torso y espalda, ¿Cuál de los siguientes
anticonvulsivantes, con mayor probabilidad, podría agravar su padecimiento
dermatológico?

A Fenobarbital.
B Gabapentina.
C Carbamazepina.
D Difenilhidantoina.

335. Relacione las causas de bocio endémico con sus características: Causas
Características 1. Alteraciones del metabolismo del yodo. 2. Bociógenos. a)
Interfieren en la captación del yodo. b) Aclaramiento renal de yodo aumentado. c)
Déficit en el aporte de yodo. d) Interfieren en la producción hormonal.

A 1ac, 2bd.
B 1ab, 2cd.
C 1bd, 2ac.
D 1bc, 2ad.

336. Las estrategias terapéuticas más eficaces para el tratamiento del VIH
tomadas en consideración son. EXCEPTO:

A Combinaciones de antirretrovirales eficaces con los que hayan sido tratados previamente.
B Combinaciones de antirretrovirales eficaces con los no presenten interacciones.
C Combinaciones de antirretrovirales eficaces con los que no hayan sido tratados previamente.
D Supresión máxima de la replicación viral.
337. Una paciente de 38 años de edad, sin antecedentes clínicos relevantes.
Debido a una reacción anafiláctica la paciente presenta parada cardiorrespiratoria y
tiene secuela neurológica. La glicemia y los electrolitos séricos están normales.
Luego de la reanimación se intentará mejorar su pronóstico, evitando daños
secundarios, ¿Qué hidratación intravenosa es la de elección, en estas condiciones?

A Lactato de Ringer.
B Solución salina al medio.
C Dextrosa al 5% en agua.
D Solución salina isotónica.

338. Usted atiende a un paciente varón de 64 años de edad con antecedentes


desde hace 2 años de varios episodios de pérdida súbita y transitoria de
consciencia, con recuperación espontánea y completa a los pocos segundos, en
cada ocasión. Acude a su consulta por un nuevo episodio ocurrido 1 hora antes,
¿Cuál de los siguientes síntomas o signos sugiere fuertemente insuficiencia
vértebro basilar como causa de su sintomatología?

A Amaurosis.
B Parosmia.
C Hemiparesia.
D Diplopia.

339. Una adolescente de 16 años presenta algunas manifestaciones con relación a


su alimentación. Se diagnostica bulimia nerviosa. ¿Cuál de los siguientes
enunciados es correcto con relación a este dignóstico?

A Se presenta con mayor frecuencia en hombres, en una relación de 10:1.


Episodios de atracón en los que se ingiere un cantidad de alimentos superior a lo que la mayoría
B
de personas ingiere en un periodo similar.
C Se acompaña de comportamientos compensatorios para evitar subir de peso al menos tres veces al día.
D Se produce exclusivamente como síntoma acompañante de anorexia nerviosa.

340. Niño de 8 años sin antecedentes patológicos, acude a emergencia con el


siguiente cuadro clínico: fiebre 38,1 grados centígrados, tos seca, estertores
crepitantes y sibilancias en base pulmonar derecha, saturación de oxígeno de 92%
con FiO2 ambiente, frecuencia respiratoria 22 por minuto. La biometría hemática
reporta: leucocitos 12000 (segmentados 75%, linfocitos 22%, eosinófilos 3%).
Radiografía de tórax: imagen condensante en base pulmonar derecha. Indique el
manejo médico más adecuado para el paciente:
A Hospitalizado. Dicloxacilina
B Ambulatorio. Cefuroxima
C Ambulatorio. Claritromicina
D Hospitalizado. Amoxicilina

341. Un paciente de 2 años de edad tiene desde hace 5 días vómito y heces
acuosas, sin moco ni sangre, con una frecuencia de 4 a 5 por día. Al examen físico
se lo encuentra irritable, con ojos hundidos, sediento y con un signo del pliegue
negativo. Llenado capilar 2 segundos, ¿Cuál de estas opciones sería la mejor
alternativa terapéutica?

A SRO 100-200 cc luego de cada diarrea.


B Solución Salina IV 100 cc/Kg en 3 horas.
C Dieta sin lácteos y líquidos caseros.
D SRO 50-100 cc/Kg en 4 horas.

342. Una niña de 4 meses de edad sin antecedentes de importancia, es traída por
su madre a la emergencia debido a que el día de hoy presenta convulsiones y
vómito. Al examen se la encuentra irritable, inconsciente y letárgica. El fondo de ojo
revela hemorragia retiniana derecha. Los exámenes de laboratorio son normales
excepto por la presencia de anemia, ¿Cuál es el diagnóstico más probable?

A Encefalitis herpética
B Enfermedad de von Willebrand.
C Hemofilia tipo A.
D Síndrome del niño sacudido.

343. Carolina de 24 años de edad presenta fiebre de 39°C tres días después de
haber sido sometida a cesárea. Su hijo nació en buenas condiciones, sin signos de
enfermedad y se encuentra en buen estado de salud. Se palpa el fondo uterino
blando y doloroso y se aprecian loquios de mal olor. Al tacto vaginal presenta dolor
en cuello que se incrementa con los movimientos. Se solicita biometría hemática
que reporta: hemoglobina 11 g/dl, leucocitos 12500 (segmentados 75%, linfocitos
25%); análisis de orina normal. ¿Cuál de los siguientes esquemas antibióticos es el
más adecuado para el tratamiento de la endometritis?

A Ampicilina - Sulbactam
B Metronidazol – Clindamicina
C Clindamicina - Gentamicina
D Gentamicina - Ampicilina
344. En el manejo de una Clave Roja obstétrica en una paciente con sangrado
genital que presenta choque severo. Señale el criterio correcto:

Transfundir 1 concentrado de glóbulos rojos previa realización de pruebas cruzadas, y en caso de no


A
cruzadas se colocará 1 unidad de ORh Negativo.
Transfundir 2 concentrados de glóbulos rojos previa realización de pruebas cruzadas, y en caso de no
B
cruzadas se colocará 2 unidades de ORh Negativo.
Transfundir 1 concentrado de glóbulos rojos previa realización de pruebas cruzadas, y en caso de no
C
cruzadas se colocará 1 unidad de plasma ORh Positivo.
Transfundir 2 concentrados de glóbulos rojos previa realización de pruebas cruzadas, y en
D
caso de no disponer pruebas cruzadas se colocará 2 unidades de plasma ORh Positivo.

345. Uno de los principales objetivos del manejo nutricional y el soporte de


ejercicio moderado en pacientes con DM gestacional es:

A Controlar la cetonuria.
B Reducir la glucemia posprandial.
C Optimizar el control glicémico.
D Controlar la hemoglobina glicosilada.

346. Un paciente varón de 62 años de edad, fumador crónico, sin enfermedad


cardiaca sintomática. De manera brusca presenta palpitaciones, a las que siguen
intensa disnea y tos con expectoración rosada espumosa, la presión arterial es
80/50 mm Hg y en el electrocardiograma se confirma fibrilación auricular, sin signos
de crecimiento auricular, isquemia ni trombos endocavitarios. En estas condiciones,
¿Cuál es la mejor opción terapéutica

A Desfibrilación eléctrica.
B Amiodarona intravenosa.
C Lidocaína intravenosa.
D Cardioversión eléctrica.

347. Un paciente de 18 años, previamente sano. Acude a su consulta por presentar


desde hace 2 días, cefalea moderada a intensa, fiebre, malestar general, dolor ocular
bilateral, nausea y vómito. Al examen físico lo relevante es una moderada rigidez nucal.
Sin alteración del estado de conciencia. La tomografía de encéfalo fue normal y en la
punción lumbar se encontró pleocitosis de 100 células, todas linfocitos, tinción Gram con
ausencia de bacterias, proteínas 48 mg/dL, glucosa del 60% de la glicemia. VIH negativo.
De entre los siguientes, ¿Cuál es el germen más probablemente implicado?

A Mycobcterium tuberculosum.
B Citomegalovirus.
C Neisseria meningitidis.
D Enterovirus.

348. Niño de 5 años de edad acude a emergencias con fiebre de 48 horas de


evolución. El examen físico es normal. El residente solicita exámenes y prescribe
acetaminofén, ¿Cuál sería la dosis adecuada para este paciente?

A 15 miligramos cada 6 horas.


B 30 miligramos/ kilo/ día.
C 5 mg/kg dosis cada 4 horas.
D 15 g/kg dosis cada 6 horas.

349. Respecto a las claves obstétricas, la toma de muestra de lactato sérico


corresponde a:

A CLAVE ROJA para manejo de hemorragia obstétrica.


B CLAVE NARANJA para manejo de traumatismo obstétrico.
C CLAVE AMARILLA para manejo de choque séptico obstétrico.
D CLAVE AZUL para manejo trastornos hipertensivos obstétricos.

350. ¿Cuál es el enunciado correcto en la infección urinaria en el embarazo?

A Embarazada sin síntomas de infección urinaria, no es necesario la realización de urocultivo.


B Las infecciones urinarias en el embarazo se clasifican en: cistitis y pielonefritis.
C Es de elección el trimetropin sulfametoxazol por sus mínimos efectos adversos.
D Los cambios anatómicos y fisiológicos en la gestación predisponen a infecciones urinarias.

351. Un niño es diagnosticado de faringoamigdalitis aguda, ¿Cuál de los


siguientes criterios clínicos le ayudaría a orientar el origen viral de esta patología?

A La presencia de tos.
B Exudado amigdalar en placas.
C Una edad mayor a 3 años.
D El inicio brusco del cuadro.

352. Una mujer de 52 años de edad, con diabetes mellitus en tratamiento oral
durante varios años. Desde hace diez días presenta rinorrea bilateral, inicialmente
fluida y después amarillenta. Desde hace tres días tiene fiebre y cefalea frontal.
Acude a consulta por dolor periocular izquierdo. Al examen físico, temperatura bucal
39°C, FC: 110/min, zona periorbitaria izquierda con edema, eritema y dolor. La
conducta más apropiada será administrar tratamiento

A Sintomático, iniciar tratamiento antibiótico intravenoso y hospitalizar.


B Ambulatorio sintomático, colocar compresas frías y reevaluar en 48 horas.
C Ambulatorio sintomático más ampicilina vía oral y reevaluar en 48 horas.
D Sintomático, realizar radiografía de senos de la cara e iniciar antibiótico.

353. Los siguientes son factores de riesgo para pancreatitis aguda, excepto:

A Diabetes mellitus tipo II


B Consumo exagerado de alcohol
C Hipertrigliceridemia familiar
D Colangiopancreatografía endoscópica

354. Varón de 38 años de edad, relata que hace 3 días, inmediatamente después
de cambiar el neumático de su vehículo, presentó de forma brusca dolor localizado
en la parte posterior de la cintura que fue incrementando su intensidad y a la
mañana siguiente se extendía por la cara posterior del muslo derecho hasta el talón,
y a momentos hasta el dedo gordo del pie, acompañándose de sensación de
amortiguamiento en el mismo trayecto. Al examen físico se encuentra signo de
Lasègue positivo, ¿Cuál sería el tratamiento inmediato?

A AINES y reposo durante 7 días.


B Paracetamol y Tramadol.
C Calor local y reposo durante 7 días.
D Ibuprofeno y Ciclobenzaprina.

355. Niño de 9 años, acude a consulta con tos leve con expectoración mucosa,
disnea espiratoria, el cuadro se ha presentado por algunas ocasiones. Al examen
físico se encuentra algo cianótico, se auscultan sibilancias e hipertimpanismo. Una
de las sospechas diagnósticas es el asma, con la finalidad de confirmar o descartar
el diagnóstico usted solicita algunos estudios funcionales pulmonares. Los
siguientes son resultados de pruebas funcionales pulmonares normales. EXCEPTO:

A FEF 25-75% = 86%.


B FVC menor a 68%.
C FEV 1 89%.
D FEV 1/ FVC = 90%.
356. Señale el signo que se presenta en el cáncer pulmonar:

A Resoplador rosado.
B Signo de Hoover.
C Tórax en tonel.
D Hipocratismo digital.

357. Paciente de 19 años desarrolla un cuadro de hematuria y proteinuria


microscópica asociado a hipertensión arterial y uremia. La sospecha diagnostica es
síndrome nefrítico. Todos los siguientes exámenes son útiles para diagnostico de
síndrome nefrítico, EXCEPTO:

A Proteínas en suero: relación albumina/globulina.


B Aclaramiento de creatinina en orina 24 horas.
C Proteinuria/creatinuria en muestra de orina.
D Complemento sérico: C3 – C4 – CH50.

358. Una paciente de 66 años de edad acude por dolor pleurítico. Luego de verificar
la existencia de derrame pleural izquierdo, usted decide realizar un estudio del
líquido. Los exámenes de sangre de la paciente indican proteínas totales 6 g/dL,
LDH: 200 UI/L. Los exámenes del líquido reportan proteínas de 1,5 g/dL, LDH: 100
UI/L. Como posibles causas de derrame pleural, de acuerdo a los resultados
obtenidos, ¿Cuál de los siguientes diagnósticos es MENOS probable?

A Nefrosis.
B Embolismo pulmonar.
C Insuficiencia cardiaca congestiva.
D Cirrosis.

359. Un preescolar presenta una diarrea aguda viral. Se decide iniciar terapia de
rehidratación oral. Se dispone de soluciones de distintas concentraciones en sodio.
Seleccione la solución más adecuada de acuerdo con las normas de la OMS:

A Solución de rehidratación oral, disponible en el mercado, con 30 mEq/l.


B Sobres de sales para rehidratación oral, preparar para obtener 90 mEq/l.
C Solución de rehidratación oral, disponible en el mercado, con 45 mEq/l.
D Sobres de sales para rehidratación oral, preparar para obtener 75 mEq/l.
360. Algunas parasitosis intestinales pueden causar la pérdida constante de sangre
dentro del huésped. Seleccione cual parasitosis intestinal causa anemia ferropénica
en casos de infestación moderada a severa:

A Balantidiasis.
B Giardiasis
C Uncinariasis.
D Amibiasis.

361. Según las curvas de crecimiento aceptadas en el Ecuador, para un varón de 7


años de edad, los percentiles 10, 50 y 95, para su antropometría, son los siguientes:
Peso: 16, 22 y 27 kg; Talla: 108,119 y 128 cm; IMC: 13.7; 15 y 18 kg/ m2; Un niño de 7
años de edad presenta los siguientes parámetros antropométricos: Peso: 30 kg;
Talla: 118 cm; IMC: 21.5 kg/m2. De acuerdo con estos datos, seleccione el
diagnóstico nutricional:

A Obesidad grado I.
B Obesidad mórbida.
C Sobrepeso.
D Obesidad Grado II.

362. Paciente de 51 años, acude a la consulta por presentar durante los últimos dos
años episodios de sensación de calor intenso seguidos de sudoración profusa,
localizados en cara, cuello y tórax. Son de corta duración (minutos). Se asocian a
palpitaciones y angustia. Tienen una frecuencia de 10 al día y en la noche le impiden
el sueño. También refiere sequedad vaginal que provoca dispareunia. La fecha de la
última menstruación es hace 7 meses y sus ciclos menstruales han sido irregulares
desde hace 18 meses. Si usted sospecha de síndrome premenopáusico, en el que
hay insuficiencia ovárica, ¿Cuál de los siguientes exámenes evidenciaría mejor el
feed back (retroalimentación) negativo que se produce?

A Determinación de estrógenos.
B Hormona folículo estimulante.
C Determinación de progesterona.
D Hormona liberadora de gonadotropinas.

363. Paciente con embarazo de 36 semanas. Acude por salida, desde hace 12 horas,
de líquido lechoso por vagina, compatible con líquido amniótico. El cérvix es
posterior, de 1 cm de dilatación y 0% de borramiento. Presentación cefálica.
Adicionalmente a la antibióticoterapia, ¿Cuál de los siguientes manejos aplicaría?

A Oxitocina 2 mUI/minuto.
B Misoprostol 200 microgramos.
C Misoprostol 50 microgramos.
D Oxitocina 50 mUI/minuto.

364. Un varón de 19 años presenta dolor abdominal de 12 horas de evolución. Al


momento el dolor se localiza en fosa ilíaca derecha, se acompaña de náusea, escaso
vómito y no ha realizado la deposición. Al examen físico el abdomen es suave,
doloroso a la palpación superficial en fosa ilíaca derecha, con signo de rebote
positivo. La apendicectomía inmediata reveló apendicitis grado II (dos). Recibe
ampicilina sulbactam por vía intravenosa, dosis única transoperatoria. ¿Cuál es la
mejor conducta a seguir?

A No requiere otro tratamiento antibiótico adicional.


B Asociación de gentamicina y metronidazol por 7 días.
C Metronidazol intravenoso cada 8 horas por 3 días.
D Asociación de ciprofloxacina y clindamicina por 7 días.

365. Un hombre de 29 años presenta alteración de la conducta sin causa aparente.


Su familiar refiere que lo encuentran con marcada inquietud, con elevado estado de
nerviosismo, muy irritable con su entorno. Increpa a transeúntes sin motivo
aparente. Además no concilia el sueño, deambula de forma errática por el domicilio,
presenta soliloquios y conductas extravagantes (anota mensajes en papeles que
coloca en las paredes). Expresa miedos respecto a su seguridad. A la exploración se
muestra lábil, con afecto incongruente, con risas inmotivadas y suspicaz. No
refieren antecedentes de consumo de sustancias psicoactivas. Indique el
diagnóstico más probable:

A Delirio.
B Pseudodemencia depresiva.
C Esquizofrenia.
D Agitación Psicomotriz.

366. Las personas que tienen derecho a solicitar copia certificada de la Historia
Clínica en un establecimiento de salud público o privado son los siguientes,
EXCEPTO:

A Los representantes legales o apoderado(a


B Los usuarios de los servicios del establecimiento de salud público o privado.
C Familiares de segundo y tercer grado de consanguinidad para fallecidos.
D Las instituciones que representan las autoridades judiciales.
367. Una mujer de 42 años de edad, desde hace 2 días luego de ingerir alimentos,
presenta dolor epigástrico moderado, que se ha intensificado de manera paulatina,
es continuo y se irradia hacia la espalda. Se acompaña de esporádico vómito de
contenido alimentario que no calma el dolor. Al examen físico, al tomar la presión
arterial, se provoca la contractura espástica de la musculatura extensora y aductora
de los dedos de la mano correspondiente. La amilasa se encuentra elevada 7 veces
y la lipasa 3 veces, con respecto a sus valores normales. El reporte ecográfico
indica páncreas incrementado de volumen e hipoecogénico. ¿Qué hallazgo de
laboratorio podría explicar la contractura espástica descrita?

A Hipocalcemia.
B Hiponatremia.
C Hipocloremia.
D Hipopotasemia.

368. Se trata de una paciente de 61 años de edad con obesidad grado II. Acude porque
presenta fiebre y dolor en su región inguinal derecha. Al examen físico tiene temperatura
bucal de 38,8 grados y se aprecia una zona eritematosa, tumefacta, caliente y dolorosa, a
nivel de su región inguinal derecha que se extiende hasta el flanco del mismo lado. Para
determinar si tiene fasceitis necrotizante, de entre las siguientes pruebas que pueden
alterarse, ¿Cuál es la prueba más específica

369. Con respecto a la urticaria en pacientes pediátricos es correcto que:

A La urticaria puede ser causada por varios factores, siendo lo más común los estímulos físicos.
B En la urticaria crónica las lesiones duran más de dos días a la semana durante 3 semanas.
El habón característico es una roncha elevada, pruriginosa y eritematosa que palidece a la
C
presión.
D En la urticaria aguda las lesiones suelen durar más de 24 horas, con tendencia a confluir.

370. Una niña de 5 años de edad, presenta desde hace 2 días dolor abdominal,
fiebre de 39,5 0C y deposiciones blandas con sangre con una frecuencia de 5 por
día. ¿Adicionalmente a la hidratación oral cuál es la mejor alternativa terapéutica?

A Amoxicilina.
B Cotrimoxazol.
C Dicloxacilina.
D Cefalexina.
371. Un niño de 5 años de edad presenta desde hace 12 horas diarreas con: moco,
pus y sangre por 6 ocasiones. En este tiempo, ha presentado 2 convulsiones tónico
clónicas generalizadas de 20 minutos cada una. El niño se encuentra aletargado,
aunque responde a estímulos táctiles; su temperatura axilar es de 39.3°C, abdomen
doloroso en forma difusa, no hay signo de pliegue en abdomen, mucosas orales
semi-húmedas. Su condición se considera grave e ingresa para tratamiento
hospitalario. Seleccione el posible germen causante y el tratamiento indicado para
este caso.

A Salmonella thypi, Cotrimoxazol oral por 3 días.


B Staphilococcus aureus, Oxacilina parenteral por 5 días.
C Escherichia coli, tratamiento Penicilina vía oral por 5 días.
D Shigella dysenteriae, Ceftriaxona parenteral por 5 días.

372. Un varón de 66 años de edad, sin antecedentes relevantes presenta molestias


urinarias desde hace varios meses, que consisten en sentir que su vejiga no se
vacía completamente luego de terminar la micción, en ocasiones requiere volver a
orinar luego de una a dos horas después de haberlo hecho, la micción a veces es
entrecortada y el chorro es débil. El examen físico general es normal para su edad y
el examen de orina no tiene alteraciones. Hasta obtener una evaluación
especializada, en caso de requerir alivio de síntomas, ¿Cuál medicamento
prescribiría usted?

A Biperiden.
B Finasteride.
C Dimenhidrinato.
D Amitriptilina.

373. Un paciente varón de 30 años de edad previamente sano, cursa con un


cuadro de neumonía comunitaria no complicada y usted prescribe reposo en cama e
instaura el tratamiento pertinente. A las 48 horas evalúa nuevamente al paciente, la
evolución clínica es estacionaria y se auscultan ruidos anormales adicionales y
contralaterales a los encontrados inicialmente. En estas condiciones, ¿Cuál es la
conducta más apropiada?

A Hospitalizar e iniciar terapéutica intravenosa.


B Cuidado domiciliario y añadir otro antibiótico vía oral.
C Solicitar una radiografía y decidir con resultados.
D Mantener terapéutica y esperar 48 horas adicionales.

374. Se trata de un paciente de 38 años de edad. Tiene antecedentes de diarrea


ocasionada por un proceso inflamatorio intestinal que afecta a la porción terminal
del íleon. Acude a su consulta para consejo nutricional. De entre los siguientes
nutrientes, ¿Cuál está probablemente más afectado?

A Cobalamina.
B Hierro.
C Aminoácidos.
D Ácido fólico.

375. Paciente femenina de 5 años de edad, con disuria, polaquiuria y dolor


suprapúbico. Presenta también hematuria visible en ropa interior. ¿Cuál es el
diagnóstico clínico?

A Cólico renal.
B Pielonefritis.
C Bacteriuria asintomática.
D Cistitis.

376. Paciente adulto con fiebre, dolor por primera ocasión, sensibilidad dolorosa a
la palpación y defensa muscular en el cuadrante superior derecho. En los examenes
realizados se encuentran leucocitosis y ligeras elevaciones de la fosfatasa alcalina,
bilirrubinas y transaminasas. Señale el diagnóstico:

A Colangitis aguda.
B Colecistitis aguda.
C Coledocolitiasis.
D Colecistitis crónica.

377. Mujer de 50 años que presenta desde hace 12 horas dolor tipo cólico en
cuadrante superior derecho, 30 minutos después de ingesta de alimentos grasos. El
dolor se irradia hacia la región subescapular derecha. Se acompaña de síntomas
neurovegetativos. Desde hace 8 horas se nota coloración amarillenta de ojos y
sensación de alza térmica y escalofríos. Al examen físico presenta: FR 16/min, FC
90/min, TA 120/80 mmHg, T 39.3° C. Facies: álgica, sudorosa, fría, con palidez
generalizada. Ojos: escleras ictéricas. Abdomen: doloroso a la palpación superficial
y profunda en epigastrio e hipocondrio derecho. Exámenes: leucocitos 18000/µL,
neutrófilos 80%, bilirrubina total 8 mg/dL, bilirrubina directa 6.5 mg/dL, bilirrubina
indirecta 1.5 mg/dL, AST 248 UI, ALT 285 UI, fosfatasa alcalina 280 UI/dL, amilasa de
440 U/dL. El reporte de ecosonografía indica: vesícula con múltiples litos en su
interior y de diferente tamaño, paredes de 3 mm de espesor; colédoco de 16 mm de
diámetro y no se visualiza cálculos. De acuerdo al cuadro clínico, indique el
diagnóstico y el tratamiento más acertados.
Colecistitis aguda más colangitis aguda; terapia hidroelectrolítica, antibióticos, colecistectomía
A
con exploración de vías biliares.
Coledocolitiasis más colangitis aguda; antibióticos y colecistectomía con exploración de vías
B
biliares.
C Colecistitis crónica reagudizada; terapia hidroelectrolítica, antibióticos y colecistectomía.
Coledocolitiasis más colangitis aguda; terapia hidroelectrolítica, antibióticos y colecistectomía
D
con exploración de vías biliares.

378. Paciente puerperal posterior a cesárea con diagnóstico de infección pélvica.


¿Cúal es el esquema de tratamiento estandar de oro?

A Vancomicina y Metronidazol.
B Secnidazol y Ciprofloxacina.
C Clindamicina y Gentamicina.
D Vancomicina y Ciprofloxacina.

379. Un paciente varón de 72 años de edad con antecedentes de alcoholismo


crónico y cirrosis. Acude a emergencias por decaimiento, fiebre y dolor abdominal
de 2 días de evolución. Al examen físico, lo relevante es la temperatura de 38,8
grados centígrados y frecuencia cardiaca 100 lpm (latidos por minuto) y presencia
de ascitis, con dolor difuso a la palpación abdominal y peritonismo. La tomografía
descarta compromiso visceral abdominal. La punción peritoneal reporta líquido con
300 polimorfonucleares por microlitro. Entre los siguientes gérmenes, ¿Cuál NO está
implicado habitualmente en la etiología de este cuadro?

A Enterobacterias
B Anaerobios.
C Estafilococo.
D Estreptococo.

380. Las siguientes complicaciones de los neonatos con retardo del crecimiento
intrauterino (RCIU) son correctas. EXCEPTO:

A Policitemia.
B Hiponatremia.
C Hipertensión pulmonar.
D Asfixia.

381. ¿Cuáles son complicaciones de la hemorragia subaracnoidea?


A Vasoespasmo, encefalopatía hipertensiva e isquemia secundaria.
B Hematoma intraparenquimatoso, hipernatremia y ceguera.
C Hidrocefalia, vasoespasmo y meningitis.
D Hidrocefalia, vasoespasmo y resangrado.

382. ¿Cuál de las siguientes es una característica de la ataxia cerebelosa?

A Rigidez.
B Déficit propioceptivo.
C Dismetría.
D Signo de Romberg positivo.

383. Señale los exámenes normales de control glucémico (objetivo terapéutico) en


un paciente diabético:

A Hb A1c 2.0% ; glucosa postprandial;15 mmol/ L.


B Hb A1c 7.0 %; glucosa postprandial; 10 mmol/L.
C Hb A1c 11.0% ; glucosa postprandial ;10 mmol/ L.
D Hb A1c;10.0% ; glucosa postprandial 10 mmol/ L.

384. Un recién nacido a término, de 18 horas de vida, presenta ictericia en zona I y


hepatoesplenomegalia. Los exámenes complementarios arrojan los siguientes
resultados: Hematocrito 48%, Bilirrubina Total 7 mg/dl, Bilirrubina directa 0.5 mg/dl.
¿Cuál sería la causa más probable de esta hiperbilirrubinemia?

A Atresia de vías biliares.


B Incompatibilidad sanguínea.
C Síndrome de Dubin Johnson.
D Ictericia por leche materna.

385. Las siguientes son las reacciones adversas más comunes asociadas a la
vacunación por rotavirus:

A Irritabilidad, diarrea, vómito, flatulencia y dolor abdominal.


B Convulsiones, alteración del estado de conciencia.
C Ictericia, elevación de enzimas hepáticas y acolia.
D Síncope, hipotensión ortostática.
386. Mujer de 28 años, cursa con embarazo de 30 semanas. Acude a emergencia
por presentar desde hace 2 días alteración del estado general, náusea, vómito,
escalofrío, fiebre de 39,5°C. Tiene como antecedentes: Gestas 3, Para 2. El examen
físico revela signos vitales normales, puño percusión lumbar derecha dolorosa. Las
maniobras de Leopold coinciden con la amenorrea, frecuencia cardíaca fetal 140 lpm
(latidos por minuto), ¿Cuál de las siguientes opciones sería la explicación más
probable para el cuadro que presenta la paciente?

A Colecistitis aguda.
B Absceso tubo ovárico.
C Apendicitis aguda.
D Pielonefritis aguda.

387. ¿Cuál de los siguientes signos ecográficos corresponde a colecistitis aguda?

A Murphy ecográfico.
B Vesícula biliar con un volumen de 30 ml.
C Pared vesicular de 2 mm de diámetro.
D Presencia de litos al interior de la vesícula.

388. Paciente diagnosticado con depresión severa, ha recibido tratamiento de


forma intermitente por dos años, es traído por sus familiares al primer nivel de
atención por emergencia, en contra de su voluntad, presenta un corte profundo en la
región anterior de la muñeca, está muy agitado, ha golpeado a su hermano e insulta
a todos. Escoja la opción terapéutica adecuada para este paciente:

Valoración e ingreso a emergencia, seguimiento conductual semanal e inicio de terapia


A sistémica
familiar de 30 a 40 sesiones.
Valoración e ingreso a emergencia, tratamiento con ansiolíticos e inicio de terapia cognitivo
B
conductual de 6 a 8 sesiones.
C Ingreso a hospitalización, cierre de la herida y olanzapina oral.
Valoración y referir para su ingreso a hospitalización, cierre de la herida y terapia electro
D
convulsiva.

389. El principal síntoma o signo predictor de prescripción antibiótica en


Bronquitis aguda es:
A Disnea.
B Esputo blanquecino.
C Tos.
D Fiebre.
390. Una mujer de 35 años de edad, sin antecedentes personales ni familiares de
importancia, acude a consulta externa por presentar de manera insidiosa: fatiga y
dificultad para la concentración. Refiere además piel seca, cabello seco y de fácil
caída, intolerancia al frío y estreñimiento. Su peso se ha incrementado en 5 libras en
los últimos meses. Al examen físico se encuentra una tiroides palpable, no visible,
algo dolorosa. ¿Cuál es el diagnóstico mas probable
A Enfermedad de Hashimoto.
B Enfermedad granulomatosa tiroidea.
C Deficiencia en ingestión de yodo.
D Síndrome de Sheehan.

391. Usted atiende a una paciente con cuadro compatible con colecistitis aguda.
Al examen físico tiene fiebre, y Murphy positivo. El utrasonido demuestra una
vesícula grande, de paredes tensas, e inmóviles, sin cálculos en su interior. Su
diagnóstico es colecistitis acalculosa, ¿Cuál de las siguientes condiciones estaría
asociada con esta enfermedad?
A Recurrentes dietas muy hipocalóricas.
B Trauma y quemadura extensa.
C Consumo habitual de fibratos.
D Estados hemolíticos crónicos.

392. Un bebé de 21 días de vida, es llevado a un control de niño sano. En el


Examen Físico se encuentra una maniobra de Barlow positiva. ¿Cuál de los
siguientes factores de riesgo podría encontrarse en la anamnesis?
A Madre multípara.
B Presentación podálica.
C Polihidramnios.
D Sexo masculino.

393. Una niña de 5 años de edad ha sido diagnosticada de emaciación grave,


¿Cómo esperaría encontrar su IMC en las curvas de crecimiento de la OMS?
A Bajo el score zeta -1.
B Bajo el score zeta -4.
C Bajo el score zeta -3.
D Bajo el score zeta -2.

394. La velocidad de ganancia de peso, promedio, por semana (kg / sem), en una
embarazada durante el segundo trimestre con un IMC pregestacional normal es de:
A 0.51Kg / sem.
B 0.42 Kg / sem.
C 0.28 Kg / sem.
D 0.22 Kg / sem
395. Paciente de 25 años con ciclos menstruales regulares (intervalo de 30 días y
duración 5 días) desde su menarquia a los 13 años. Consulta por amenorrea de 4
meses de duración, sin otra sintomatología. El examen físico y genital es normal. En
el estudio de la amenorrea secundaria, ¿Cuál de las siguientes pruebas NO debe
incluir usted en el inicio del proceso diagnóstico?
A Gonadotropina coriónica.
B Gonadotropinas hipofisarias (LH-FSH).
C Hormona estimuladora del tiroides.
D Prueba a la progesterona.

396. Paciente de 49 años, con antecedentes de hipertensión arterial y diabetes


gestacional previa, en su único embarazo. Menstruaciones regulares y normales
hasta hace 3 años, en que se hicieron frecuentes y prolongadas, (3-4 semanas de
sangrado). Índice de masa corporal de 29,4, presión arterial de 146/92 mm Hg. Al
examen genital presenta sangrado uterino, el resto del aparato genital es normal,
¿Cuál de los siguientes procedimientos usaría inicialmente para manejar el caso?
A Papanicolaou.
B Legrado instrumental.
C Histeroscopia.
D Ecografía transvaginal.

397. Una mujer de 70 años, diabética e hipertensa requiere bajo criterio médico
una cirugía para colocar prótesis de cadera derecha. La señora esta lúcida y su
salud se encuentra estable. Los hijos se reúnen y deciden no autorizar la
intervención quirúrgica porque han leído en internet y familiares médicos les han
informado que la convalecencia es dolorosa y dificultosa. ¿Cuál de las siguientes
debe ser la conducta médica recomendada?
A Convocar al comité de ética para decidir si se opera o no a la paciente
B El galeno decide en base a evidencia médica si opera o no a la paciente.
C No operar a la señora conforme a lo decidido en forma consensuada por los hijos.
D Conversar con la paciente y operar en base a la decisión de ella.

398. Una paciente de 58 años de edad, sin antecedentes de importancia. Está


asintomática, pero durante los controles médicos subsecuentes, usted verifica que
las cifras de presión arterial de la paciente están en 140/98 mm Hg, y decide iniciar
terapia farmacológica. De entre las siguientes alternativas farmacológicas, ¿Cuál
tiene MENOR impacto sobre la mortalidad, morbilidad y el riesgo de insuficiencia
cardiaca?
A Calcioantagonistas.
B Diuréticos.
C Alfabloqueantes.
D Betabloqueantes.

399. El tratamiento de cetoacidosis diabética es:


A Tratar la alcalosis metabólica.
B Venoclisis con solución salina.
C Colocar insulina de acción prolongada.
D Administración de bicarbonato.

400. En la mujer con diabetes gestacional es importante la identificación de los


factores de riesgo. Los siguientes son factores de riesgo alto. EXCEPTO:
A Síndrome de ovario poliquístico (SOP
B Trastorno del metabolismo de los carbohidratos.
C Partos con productos macrosómicos de más de 4 kilos.
D Sobrepeso (IMC mayor a 25 kg/m2

401. Paciente asintomático, que al examen físico presenta un aumento del tiroides
grado II, sin nódulos palpables. Los exámenes reportan: TSH y T3 normales, T4 total
baja, colesterol alto, anticuerpos contra TPO negativos, la gamagrafía tiroidea pone
de manifiesto un aumento en la captación. Señale el diagnóstico:
A Bocio multinodular tóxico.
B Bocio difuso no tóxico.
C Tiroiditis de Hashimoto.
D Bocio multinodular no tóxico.

402. Indique que enfermedad se relaciona con la hipertrigliceridemia Grave.


A Hipertiroidismo.
B Glomerulonefritis Aguda.
C Diabetes Mellitus 2.
D Hepatitis Aguda.

403. Para el tratamiento de una mucositis candidiásica por metotrexate, en un


niño con tratamiento oncológico, se debe utilizar?
A Ketoconazol.
B Secnidazol.
C Fluconazol.
D Miconazol.

404. Se trata de una paciente de 35 años de edad, con antecedentes de


convulsiones pero con tratamiento irregular. Desde hace una hora presenta
tres episodios de crisis convulsivas generalizadas, con pérdida de la
conciencia durante cada convulsión. Usted la recibe en emergencia en un
nuevo episodio convulsivo de similares características. A más de una
benzodiacepina, decide administrar fenitoina, ¿Cuál es el tiempo en el que
debe pasar la dosis de carga de este fármaco?
A Máximo en 2 minutos.
B Entre 5 y 10 minutos.
C Entre 1 y 2 horas.
D Entre 20 y 30 minutos.

405. Hombre de 69 años se presenta a consulta quejándose de dolor de pecho que


ha empeorado en las últimas semanas, el dolor se acompaña de episodios de
síncope, especialmente cuando realiza algún tipo de esfuerzo; también refiere
dificultad respiratoria durante las noches a la hora de acostarse. Al examen físico
presenta: P: 89/min, TA: 135/89 mmHg, FR: 16/min, T: 37 °C. La auscultación
cardiaca revela un soplo de tipo sistólico crescendo-decrescendo en foco aórtico
con un desdoblamiento paradójico de S2. ¿Cuál sería el diagnóstico?
A Insuficiencia Mitral
B Insuficiencia Tricuspídea
C Estenosis Pulmonar
D Estenosis Aórtica

406. Se trata de una paciente de 38 años de edad con enfermedad ácido péptica
sintomática, recurrente, sin sangrado digestivo. Tiene biopsia positiva para
Helicobacter pylori. De entre las siguientes alternativas, ¿Cuál es la más adecuada?
Se trata de un paciente de 35 años de edad, previamente sano, que se dedica a la venta
informal de quesos artesanales. Acude porque desde hace un par de semanas tiene
sensación de fiebre, decaimiento, sudoración nocturna, dolor corporal y algo de pérdida de
peso. No tiene tos. Al examen físico lo relevante es la temperatura de 37,9 grados
centígrados. En los exámenes de laboratorio las muestras de BAAR en orina son negativas,
tiene leucocitos de 7.000/mm3, con 50% de linfocitos y 45% de neutrófilos de características
normales. Aglutinaciones para brucella son positivas 1:640. Hasta que los resultados de los
cultivos estén listos, ¿Qué antibióticos debería prescribir

A Vancomicina y gentamicina.
B Dicloxacilina y amikacina.
C Ampicilina y claritromicina.
D Estreptomicina y doxiciclina.

407. Un lactante mayor de 24 meses de edad tiene un peso de 10kg, una talla de
70 cm y un perímetro cefálico de 43 cm. El peso está bajo el percentil 5, la talla en el
percentil 5 y el perímetro cefálico en el percentil 25. Al examen físico presenta
astenia, reducción del tejido celular subcutáneo y de las masas musculares.
Abdomen algo prominente. Piel seca, pelo escaso, descolorido, sin brillo y
fácilmente desprendible. No presenta visceromegalias. Si el percentil 50 del peso
para esta edad es de 12 kg, su pérdida de peso es de 16.6%. Seleccione el
diagnóstico más probable para este niño:
A Desnutrición calórica proteica de segundo grado.
B Desnutrición calórica proteica de primer grado.
C Desnutrición proteica calórica o kwashiorkor.
D Desnutrición calórica proteica de tercer grado.
408. Una paciente nuligesta presenta ciclos menstruales regulares con FUM hace
7 días, al examen físico tiene dolor anexial bilateral, la movilización del cuello uterino
y palpación directa de abdomen son dolorosas, adicionalmente tiene 38.5 grados C y
leucocitosis de 12 000 / ml. ¿Cuál es el diagnóstico?
A Endometriosis pélvica.
B Ruptura de quiste del cuerpo lúteo.
C Enfermedad pélvica inflamatoria.
D Infección de vías urinarias.

409. Usted se encuentra en un hospital de tercer nivel de atención y recibe una


paciente de 24 años con un embarazo de 33 semanas más ruptura prematura de
membranas confirmada, sin dinámica uterina. ¿Cuál es el protocolo a seguir?
A Profilaxis antibiótica, reposo en cama, maduración pulmonar y enviar a su casa.
B Hospitalización, reposo en cama, profilaxis antibiótica y maduración pulmonar.
C Uso de antibiótico, maduración pulmonar, uso de tocolíticos y referir a otro centro asistencial.
D Hospitalización, reposo en cama, uso de tocolíticos y uso de antibióticos.

410. ¿Cuáles son complicaciones de la hemorragia subaracnoidea?

A Vasoespasmo, encefalopatía hipertensiva e isquemia secundaria.


B Hematoma intraparenquimatoso, hipernatremia y ceguera.
C Hidrocefalia, vasoespasmo y meningitis.
D Hidrocefalia, vasoespasmo y resangrado.

411. Paciente mujer de 30 años de edad, sin antecedentes de importancia, que luego de
discusión familiar, presenta de manera brusca cefalea intensa y náusea persistente. Tres
horas más tarde acude a consulta y lo relevante en el examen físico es que tiene presión
arterial de 150/100 mm Hg, frecuencia cardiaca: 86 latidos por minuto, algo somnolienta y
dolor intenso al intentar flexionar el cuello. La conducta más apropiada será:

A Administrar analgesia, prescribir reposo absoluto y realizar tomografía.


B Tranquilizar a la paciente, administrar antihipertensivos y realizar tomografía.
C Tranquilizar a la paciente, solicitar apoyo psicológico y reevaluar en 48 horas.
D Administrar antiagregantes plaquetarios más analgesia y tratar con antihipertensivo.

412. ¿Cuál de las siguientes es una característica de la ataxia cerebelosa?

A Rigidez.
B Déficit propioceptivo.
C Dismetría.
D Signo de Romberg positivo.

413. Un recién nacido a término, de 18 horas de vida, presenta ictericia en zona I y


hepatoesplenomegalia. Los exámenes complementarios arrojan los siguientes
resultados: Hematocrito 48%, Bilirrubina Total 7 mg/dl, Bilirrubina directa 0.5 mg/dl.
¿Cuál sería la causa más probable de esta hiperbilirrubinemia?

A Atresia de vías biliares.


B Incompatibilidad sanguínea.
C Síndrome de Dubin Johnson.
D Ictericia por leche materna.

414. Mujer de 28 años, cursa con embarazo de 30 semanas. Acude a emergencia


por presentar desde hace 2 días alteración del estado general, náusea, vómito,
escalofrío, fiebre de 39,5°C. Tiene como antecedentes: Gestas 3, Para 2. El examen
físico revela signos vitales normales, puño percusión lumbar derecha dolorosa. Las
maniobras de Leopold coinciden con la amenorrea, frecuencia cardíaca fetal 140 lpm
(latidos por minuto), ¿Cuál de las siguientes opciones sería la explicación más
probable para el cuadro que presenta la paciente?

A Colecistitis aguda.
B Absceso tubo ovárico.
C Apendicitis aguda.
D Pielonefritis aguda.

415. ¿Cuál de los siguientes signos ecográficos corresponde a colecistitis aguda?


A Murphy ecográfico.
B Vesícula biliar con un volumen de 30 ml.
C Pared vesicular de 2 mm de diámetro.
D Presencia de litos al interior de la vesícula.

416. Paciente diagnosticado con depresión severa, ha recibido tratamiento de


forma intermitente por dos años, es traído por sus familiares al primer nivel de
atención por emergencia, en contra de su voluntad, presenta un corte profundo en la
región anterior de la muñeca, está muy agitado, ha golpeado a su hermano e insulta
a todos. Escoja la opción terapéutica adecuada para este paciente:
Valoración e ingreso a emergencia, seguimiento conductual semanal e inicio de terapia
A sistémica
familiar de 30 a 40 sesiones.
Valoración e ingreso a emergencia, tratamiento con ansiolíticos e inicio de terapia cognitivo
B
conductual de 6 a 8 sesiones.
C Ingreso a hospitalización, cierre de la herida y olanzapina oral.
Valoración y referir para su ingreso a hospitalización, cierre de la herida y terapia electro
D
convulsiva.

417. El principal síntoma o signo predictor de prescripción antibiótica en


Bronquitis aguda es:
A Disnea.
B Esputo blanquecino.
C Tos.
D Fiebre.

418. Una mujer de 35 años de edad, sin antecedentes personales ni familiares de


importancia, acude a consulta externa por presentar de manera insidiosa: fatiga y
dificultad para la concentración. Refiere además piel seca, cabello seco y de fácil
caída, intolerancia al frío y estreñimiento. Su peso se ha incrementado en 5 libras en
los últimos meses. Al examen físico se encuentra una tiroides palpable, no visible,
algo dolorosa. ¿Cuál es el diagnóstico mas probable?
A Enfermedad de Hashimoto.
B Enfermedad granulomatosa tiroidea.
C Deficiencia en ingestión de yodo.
D Síndrome de Sheehan.

419. Usted atiende a una paciente con cuadro compatible con colecistitis aguda.
Al examen físico tiene fiebre, y Murphy positivo. El utrasonido demuestra una
vesícula grande, de paredes tensas, e inmóviles, sin cálculos en su interior. Su
diagnóstico es colecistitis acalculosa, ¿Cuál de las siguientes condiciones estaría
asociada con esta enfermedad?
A Recurrentes dietas muy hipocalóricas.
B Trauma y quemadura extensa.
C Consumo habitual de fibratos.
D Estados hemolíticos crónicos.

420. Un bebé de 21 días de vida, es llevado a un control de niño sano. En el


Examen Físico se encuentra una maniobra de Barlow positiva. ¿Cuál de los
siguientes factores de riesgo podría encontrarse en la anamnesis?
A Madre multípara.
B Presentación podálica.
C Polihidramnios.
D Sexo masculino.

421. Una niña de 5 años de edad ha sido diagnosticada de emaciación grave,


¿Cómo esperaría encontrar su IMC en las curvas de crecimiento de la OMS?
A Bajo el score zeta -1.
B Bajo el score zeta -4.
C Bajo el score zeta -3.
D Bajo el score zeta -2.

422. La velocidad de ganancia de peso, promedio, por semana (kg / sem), en una
embarazada durante el segundo trimestre con un IMC pregestacional normal es de:
A 0.51Kg / sem.
B 0.42 Kg / sem.
C 0.28 Kg / sem.
D 0.22 Kg / sem

423. Paciente de 25 años con ciclos menstruales regulares (intervalo de 30 días y


duración 5 días) desde su menarquia a los 13 años. Consulta por amenorrea de 4
meses de duración, sin otra sintomatología. El examen físico y genital es normal. En
el estudio de la amenorrea secundaria, ¿Cuál de las siguientes pruebas NO debe
incluir usted en el inicio del proceso diagnóstico?
A Gonadotropina coriónica.
B Gonadotropinas hipofisarias (LH-FSH).
C Hormona estimuladora del tiroides.
D Prueba a la progesterona.

424. Una mujer de 70 años, diabética e hipertensa requiere bajo criterio médico
una cirugía para colocar prótesis de cadera derecha. La señora esta lúcida y su
salud se encuentra estable. Los hijos se reúnen y deciden no autorizar la
intervención quirúrgica porque han leído en internet y familiares médicos les han
informado que la convalecencia es dolorosa y dificultosa. ¿Cuál de las siguientes
debe ser la conducta médica recomendada?
A Convocar al comité de ética para decidir si se opera o no a la paciente
B El galeno decide en base a evidencia médica si opera o no a la paciente.
C No operar a la señora conforme a lo decidido en forma consensuada por los hijos.
D Conversar con la paciente y operar en base a la decisión de ella.

425. Una paciente de 58 años de edad, sin antecedentes de importancia. Está


asintomática, pero durante los controles médicos subsecuentes, usted verifica que
las cifras de presión arterial de la paciente están en 140/98 mm Hg, y decide iniciar
terapia farmacológica. De entre las siguientes alternativas farmacológicas, ¿Cuál
tiene MENOR impacto sobre la mortalidad, morbilidad y el riesgo de insuficiencia
cardiaca?
A Calcioantagonistas.
B Diuréticos.
C Alfabloqueantes.
D Betabloqueantes.
426. El tratamiento de cetoacidosis diabética es:
A Tratar la alcalosis metabólica.
B Venoclisis con solución salina.
C Colocar insulina de acción prolongada.
D Administración de bicarbonato.

427. En la mujer con diabetes gestacional es importante la identificación de los


factores de riesgo. Los siguientes son factores de riesgo alto. EXCEPTO:
A Síndrome de ovario poliquístico (SOP
B Trastorno del metabolismo de los carbohidratos.
C Partos con productos macrosómicos de más de 4 kilos.
D Sobrepeso (IMC mayor a 25 kg/m2

428. Paciente asintomático, que al examen físico presenta un aumento del tiroides
grado II, sin nódulos palpables. Los exámenes reportan: TSH y T3 normales, T4 total
baja, colesterol alto, anticuerpos contra TPO negativos, la gamagrafía tiroidea pone
de manifiesto un aumento en la captación. Señale el diagnóstico:
A Bocio multinodular tóxico.
B Bocio difuso no tóxico.
C Tiroiditis de Hashimoto.
D Bocio multinodular no tóxico.

429. Indique que enfermedad se relaciona con la hipertrigliceridemia Grave.


A Hipertiroidismo.
B Glomerulonefritis Aguda.
C Diabetes Mellitus 2.
D Hepatitis Aguda.

430. Para el tratamiento de una mucositiscandidiásica por metotrexate, en un niño


con tratamiento oncológico, se debe utilizar?
A Ketoconazol.
B Secnidazol.
C Fluconazol.
D Miconazol.

431. Se trata de una paciente de 35 años de edad, con antecedentes de


convulsiones pero con tratamiento irregular. Desde hace una hora presenta
tres episodios de crisis convulsivas generalizadas, con pérdida de la
conciencia durante cada convulsión. Usted la recibe en emergencia en un
nuevo episodio convulsivo de similares características. A más de una
benzodiacepina, decide administrar fenitoina, ¿Cuál es el tiempo en el que
debe pasar la dosis de carga de este fármaco?
A Máximo en 2 minutos.
B Entre 5 y 10 minutos.
C Entre 1 y 2 horas.
D Entre 20 y 30 minutos.

432. Hombre de 69 años se presenta a consulta quejándose de dolor de pecho que


ha empeorado en las últimas semanas, el dolor se acompaña de episodios de
síncope, especialmente cuando realiza algún tipo de esfuerzo; también refiere
dificultad respiratoria durante las noches a la hora de acostarse. Al examen físico
presenta: P: 89/min, TA: 135/89 mmHg, FR: 16/min, T: 37 °C. La auscultación
cardiaca revela un soplo de tipo sistólico crescendo-decrescendo en foco aórtico
con un desdoblamiento paradójico de S2. ¿Cuál sería el diagnóstico?
A Insuficiencia Mitral
B Insuficiencia Tricuspídea
C Estenosis Pulmonar
D Estenosis Aórtica

433. Se trata de una paciente de 38 años de edad con enfermedad ácido péptica
sintomática, recurrente, sin sangrado digestivo. Tiene biopsia positiva para
Helicobacter pylori. De entre las siguientes alternativas, ¿Cuál es la más adecuada?
Se trata de un paciente de 35 años de edad, previamente sano, que se dedica a la venta
informal de quesos artesanales. Acude porque desde hace un par de semanas tiene
sensación de fiebre, decaimiento, sudoración nocturna, dolor corporal y algo de pérdida de
peso. No tiene tos. Al examen físico lo relevante es la temperatura de 37,9 grados
centígrados. En los exámenes de laboratorio las muestras de BAAR en orina son negativas,
tiene leucocitos de 7.000/mm3, con 50% de linfocitos y 45% de neutrófilos de características
normales. Aglutinaciones para brucella son positivas 1:640. Hasta que los resultados de los
cultivos estén listos, ¿Qué antibióticos debería prescribir

A Vancomicina y gentamicina.
B Dicloxacilina y amikacina.
C Ampicilina y claritromicina.
D Estreptomicina y doxiciclina.

434. Un lactante mayor de 24 meses de edad tiene un peso de 10kg, una talla de
70 cm y un perímetro cefálico de 43 cm. El peso está bajo el percentil 5, la talla en el
percentil 5 y el perímetro cefálico en el percentil 25. Al examen físico presenta
astenia, reducción del tejido celular subcutáneo y de las masas musculares.
Abdomen algo prominente. Piel seca, pelo escaso, descolorido, sin brillo y
fácilmente desprendible. No presenta visceromegalias. Si el percentil 50 del peso
para esta edad es de 12 kg, su pérdida de peso es de 16.6%. Seleccione el
diagnóstico más probable para este niño:
A Desnutrición calórica proteica de segundo grado.
B Desnutrición calórica proteica de primer grado.
C Desnutrición proteica calórica o kwashiorkor.
D Desnutrición calórica proteica de tercer grado.

435. Una paciente nuligesta presenta ciclos menstruales regulares con FUM hace
7 días, al examen físico tiene dolor anexial bilateral, la movilización del cuello uterino
y palpación directa de abdomen son dolorosas, adicionalmente tiene 38.5 grados C y
leucocitosis de 12 000 / ml. ¿Cuál es el diagnóstico?
A Endometriosis pélvica.
B Ruptura de quiste del cuerpo lúteo.
C Enfermedad pélvica inflamatoria.
D Infección de vías urinarias.

436. Usted se encuentra en un hospital de tercer nivel de atención y recibe una


paciente de 24 años con un embarazo de 33 semanas más ruptura prematura de
membranas confirmada, sin dinámica uterina. ¿Cuál es el protocolo a seguir?
A Profilaxis antibiótica, reposo en cama, maduración pulmonar y enviar a su casa.
B Hospitalización, reposo en cama, profilaxis antibiótica y maduración pulmonar.
C Uso de antibiótico, maduración pulmonar, uso de tocolíticos y referir a otro centro asistencial.
D Hospitalización, reposo en cama, uso de tocolíticos y uso de antibióticos.

437. Una adolescente de 16 años presenta algunas manifestaciones con relación a


su alimentación. Se diagnostica bulimia nerviosa. ¿Cuál de los siguientes
enunciados es correcto con relación a este dignóstico?

A Se presenta con mayor frecuencia en hombres, en una relación de 10:1.


Episodios de atracón en los que se ingiere un cantidad de alimentos superior a lo que la mayoría
B
de personas ingiere en un periodo similar.
C Se acompaña de comportamientos compensatorios para evitar subir de peso al menos tres veces al día.
D Se produce exclusivamente como síntoma acompañante de anorexia nerviosa.

438. Niño de 8 años sin antecedentes patológicos, acude a emergencia con el


siguiente cuadro clínico: fiebre 38,1 grados centígrados, tos seca, estertores
crepitantes y sibilancias en base pulmonar derecha, saturación de oxígeno de 92%
con FiO2 ambiente, frecuencia respiratoria 22 por minuto. La biometría hemática
reporta: leucocitos 12000 (segmentados 75%, linfocitos 22%, eosinófilos 3%).
Radiografía de tórax: imagen condensante en base pulmonar derecha. Indique el
manejo médico más adecuado para el paciente:

A hospitalizado. Dicloxacilina
B ambulatorio. Cefuroxima
C ambulatorio. Claritromicina
D hospitalizado. Amoxicilina
439. Paciente de 19 años desarrolla un cuadro de hematuria y proteinuria
microscópica asociado a hipertensión arterial y uremia. La sospecha diagnostica es
síndrome nefrítico. Todos los siguientes exámenes son útiles para diagnostico de
síndrome nefrítico, EXCEPTO:

A Proteínas en suero: relación albumina/globulina.


B Aclaramiento de creatinina en orina 24 horas.
C Proteinuria/creatinuria en muestra de orina.
D Complemento sérico: C3 – C4 – CH50.

440. Una paciente de 66 años de edad acude por dolor pleurítico. Luego de verificar
la existencia de derrame pleural izquierdo, usted decide realizar un estudio del
líquido. Los exámenes de sangre de la paciente indican proteínas totales 6 g/dL,
LDH: 200 UI/L. Los exámenes del líquido reportan proteínas de 1,5 g/dL, LDH: 100
UI/L. Como posibles causas de derrame pleural, de acuerdo a los resultados
obtenidos, ¿Cuál de los siguientes diagnósticos es MENOS probable?

A Nefrosis.
B Embolismo pulmonar.
C Insuficiencia cardiaca congestiva.
D Cirrosis.

441. Paciente con embarazo de 36 semanas. Acude por salida, desde hace 12 horas,
de líquido lechoso por vagina, compatible con líquido amniótico. El cérvix es
posterior, de 1 cm de dilatación y 0% de borramiento. Presentación cefálica.
Adicionalmente a la antibióticoterapia, ¿Cuál de los siguientes manejos aplicaría?

A Oxitocina 2 mUI/minuto.
B Misoprostol 200 microgramos.
C Misoprostol 50 microgramos.
D Oxitocina 50 mUI/minuto.

442. Las personas que tienen derecho a solicitar copia certificada de la Historia
Clínica en un establecimiento de salud público o privado son los siguientes,
EXCEPTO:

A Los representantes legales o apoderado(a


B Los usuarios de los servicios del establecimiento de salud público o privado.
C Familiares de segundo y tercer grado de consanguinidad para fallecidos.
D Las instituciones que representan las autoridades judiciales.
443. Con respecto a la urticaria en pacientes pediátricos es correcto que:

A La urticaria puede ser causada por varios factores, siendo lo más común los estímulos físicos.
B En la urticaria crónica las lesiones duran más de dos días a la semana durante 3 semanas.
El habón característico es una roncha elevada, pruriginosa y eritematosa que palidece a la
C
presión.
D En la urticaria aguda las lesiones suelen durar más de 24 horas, con tendencia a confluir.

1. Las siguientes son factores relacionados al desarrollo de dermatitis seborreica.


EXCEPTO:

A . Infeccion por virus de inmunodeficiencia humana (VIH).

B. Enfermedades neurológicas como Parkinson.

C. Infección por Staphylococcus aureus.

D. Infección por malassezia furfur.

D. No debe recibir levotiroxina.


2. En un paciente asintomático se ausculta un desdoblamiento amplio del segundo
ruido cardiaco sobre la línea paraesternal izquierda, a nivel del segundo espacio
intercostal. En estas condiciones, ¿Cuál de los siguientes bloqueos puede explicar el
hallazgo encontrado?

A. Aurículo ventricular de primer grado.

B. Completo de la rama derecha.


C. De la subdivisión anterior izquierda.
D. Aurículo ventricular de tercer grado.

3. Seleccione el enunciado correcto para un paciente pediátrico con fiebre:


A. F fiebre mayor a 41 º C es producto de bacterias muy agresivas.
B. Un pico febril aislado se asocia a infección bacteriana.
C. La temperatura se mide inmediatamente despues de retirar el exceso de ropa.

D. La evaluación inicial se centra en la búsqueda de una causa infecciosa.


4. Un escolar de 4 años de edad, eutrófico, acude a consulta porque desde hace 10 días
presenta tos intensa, inicialmente y luego de un resfriado, era una tos seca.
Progresivamente se ha transformado en una tos productiva, con esputo claro y espeso.
Ha tomado varios medicamentos para eliminar la flema, sin embargo, la tos persiste y
le impide dormir normalmente. Al examen físico se encuentra afebril, con buen estado
general. Tose en forma repetitiva y a la auscultación presenta estertores roncantes
difusos, especialmente en la espiración. La Rx de tórax es normal, con esta
información. Seleccione el diagnóstico:
A. Adenoiditis.
B. Laringitis aguda.

C. Bronquitis aguda.
D. Bronquiolitis.

5 . Un niño de 18 meses de edad acude a emergencias con fiebre de 24 horas de


evolución. El peso es de 10 Kg y el examen físico es normal. El residente prescribe un
antitérmico y lo llama para una reevaluación en 48 horas. ¿Cuál sería la dosis de
acetaminofen adecuada?

A. 50 Mg cada 6 horas.

B 75 mg cada 6 horas.

C 250 mg cada 4 horas.

D 100 mg cada 4 horas


6 . Gimnasta de 17 años, nuligesta, acude a la consulta por presentar falta de periodo
menstrual. Con prueba de embarazo negativa. En la anamnesis revela prácticas físicas
extenuantes de 6 horas diarias, al examén físico IMC (indice de masa corporal) 17
¿Cuál es la causa de la amenorrea secundaria?
A. SIndrome de Asherman.
B Defecto de los conductos de Muller.

C Función de la FSH (hormona folículo estimulante) reducida.


D Hiperplasia suprarrenal congénita.
7. Mujer de 24 años de edad, primigesta con 28 semanas de gestación, acude con una
presión arterial de 160/120 mm Hg y proteinuria 3+, con visión borrosa, la
exploración a las 6 horas arroja los mismos resultados. Indique cuál es su diagnóstico:
A.- Preeclampsia grave.
B Preeclampsia leve.
C Hipertensión Gestacional.
D Eclampsia.

8 . Paciente varón de 70 años, portador de fibrilación auricular crónica, acude al


servicio de emergencias. Ha presentado de manera súbita, la pérdida de movimientos
y de la sensibilidad en las extremidades superior e inferior del lado izquierdo y
desviación de la mirada hacia el lado derecho. ¿Cuál de las siguientes arterias está
probablemente involucrada?
A.- Cerebral media.
B. Cerebral anterior.
C. Cerebral posterior.
D. Vértebro basilar
9. . En el traumatismo cráneo encefálico es muy importante mantener una correcta
presión de perfusión cerebral (PPC), que resulta de la diferencia de una presión
sanguínea, menos la presión intracraneal (PIC), ¿Cómo se calcula la PPC?
a. Presión venosa central menos PIC.
B Presión arterial diastólica menos PIC.
C Presión arterial sistólica menos PIC.

D Presión arterial media menos PIC.


10 .- Una mujer de 65 años de edad fue gastrectomizada por cáncer gástrico hace 2
meses. Acude porque de manera brusca presenta disnea de reposo, dolor torácico
derecho y cianosis. Al examen físico presión arterial 100/60 mmHg, frecuencia
cardiaca 150 lpm (latidos por minuto), frecuencia respiratoria 30 rpm (respiraciones
por minuto), saturación de 84%, respirando aire ambiente, cianosis central. En
radiografía de tórax no hay alteraciones y la gammagrafía pulmonar reveló defectos
en el segmento lateral basal del lóbulo inferior del pulmón derecho y en el ángulo
costofrénico posterobasal derecho. El electrocardiograma muestra taquicardia sinusal
de 150 por minuto. ¿Cuál de los siguientes fármacos debería administrar en estas
condiciones?
A. Amiodarona.
B. Estreptoquinasa.
C. Digoxina.

D. Heparina.
11 .- Un recién nacido de 32 semanas de edad gestacional, sin antecedentes prenatales
de importancia, nace por parto céfalo vaginal luego de amniorexis instrumental.
Presenta desde el nacimiento quejido, aleteo nasal, retracciones y cianosis. En el
examen físico se encuentra una frecuencia respiratoria de 80 lpm (latidos por minuto),
crepitantes bilaterales y mala entrada de aire. La Radiografía de tórax revela un
volumen pulmonar disminuido, infiltrado retículo granular bilateral, ¿Cuál de los
siguientes factores mejoraría el pronóstico?

A. Realizar cesárea electiva.

B. Paciente de sexo masculino.

C. Aplicar corticoides prenatales.

D. Prescribir prostaglandina E2.

12 .- Un niño de 3 y medio años de edad, cuyo crecimiento y desarrollo psicomotor son


normales, tiene frecuentemente problemas con sus padres porque no desea comer.
Seleccionar la acción adecuada frente a este problema:
A. Tranquilizar y guiar a los padres.
B. Ofrecerle alimentos constantemente, entre comidas, para que pique.
C. Darle estimulantes del apetito para que pueda comer mejor.
D. Usar premios y juegos para distraerlo mientras se le ayuda a comer.
13.- Un niño de 5 años de edad acude por primera vez a su consulta, para control de
niño sano. Es un niño activo, comunicativo. Su peso y talla se encuentran en el
percentil 50. Juega fútbol y es amiguero. Sin embargo, a la auscultación cardíaca
presenta soplo sistólico mesocárdico, grado II/VI, sin una irradiación definida y que
con los cambios de posición del niño, disminuye notablemente de intensidad.
Seleccione el literal correspondiente a esta descripción:
A. Soplo funcional o inocente.
B. Comunicación interventricular (CIV
C. Comunicación interauricular (CIA
D. Persistencia del conducto arterioso (PCA).
14 .- Un paciente de 12 meses de edad presentó hace 2 días rinorrea y tos “perruna”.
Hoy presenta cianosis y disfonía. Al examen físico se encuentra estridor en reposo y
retracciones subcostales. ¿Cuál de las siguientes opciones escogería como la conducta
más adecuada a seguir?
A. Realizar nebulizaciones con adrenalina racémica y usar corticoides sistémicos.
B Pedir una biometría hemática y administrar nebulizaciones con salbutamol.
C Administrar antibióticos parenterales y corticoide inhalado.
D Solicitar radiografía de cuello y según el resultado iniciar antibioticoterapia.

15 .- La forma de prescripción de dosis deimpregnación de Sulfato de Magnesio, como


tratamiento para prevenir la eclampsia es:
A. 4 Gramos de Sulfato de Magnesio, vía intravenosa a pasar en 20 minutos.
B 8 gramos de Sulfato de Magnesio, vía intravenosa a pasar en 10 minutos.
C 6 gramos de Sulfato de Magnesio, vía intravenosa a pasar en 20 minutos.
D 3 gramos de Sulfato de Magnesio, vía intravenosa a pasar en 20 minutos.

PREGUNTAS SIMULACRO

1- ¿CUAL DE LOS MEDICAMENTOS DE ELECCIÓN PARA EL TRATAMIENTO


DE LA TIÑA CAPITIS EN ADULTOS?
GRISEOFULVINA.
2- SEÑALE QUE TASA DE FILTRACIÓN GLOMERULAR QUE INDICA FALLA
RENAL EN EL PACIENTE DIABÉTICO TIPO 2:
14-15 ML/MIN.
3- UN VARÓN DE 20 AÑOS DE EDAD RESIDENTE EN RIOBAMBA CON
ANTECEDENTE DE DIABETES MELLITUS TIPO 1, DESDE LOS 14 AÑOS TUVO
AMIGDALITIS AGUDA HACE 3 DÍAS, HACE 48 HORAS NAUSEAS, VÓMITOS,
POLIURIA, SED INTENSA Y DOLOR ABDOMINAL. AL EXAMEN FÍSICO TA
110/70. FC 120, FR 18 CON RESPIRACIONES PROFUNDAS LABORATORIO
PH 7.16, PCO2 18, HCO3 8, SODIO 144, POTASIO 6, EN ORINA CUERPOS
CETÓNICOS ++++.¿CUAL ES LA ENZIMA CUYA ACTIVIDAD INCREMENTADA
DIRECTAMENTE DERIVA EN LA PRODUCCIÓN DE CUERPOS CETÓNICOS?
PALMITOTRANSFERASA DE CARNITINA I
4- LA DEFICIENCIA DE FOLATO EN LA ANEMIA MEGALOBLASTICA CAUSA:
ABORTOS RECIDIVANTES.

5- PACIENTE JOVEN, SIN PATOLOGÍAS PREVIAS, SUFRE UN TRAUMATISMO


CRANEAL Y LO RECIBE EN EL SERVICIO DE EMERGENCIA. LOS SIGNOS
VITALES SON NORMALES, LA ESCALA DE GLASGOW ES DE 9 Y EN EL OJO
DERECHO SE APRECIA PUPILA 5 MM, SIN RESPUESTA A LA LUZ, PERO EL
REFLEJO CONSENSUAL PRESENTE EN EL LADO IZQUIERDO, CUYA PUPILA
MIDE 3MM, ADEMAS, EXISTE DESVIACION EXTERNA DEL OJO DERECHO.
¿QUE PAR CRANEAL ESTA COMPROMETIDO EN ESE LADO?
III PAR.
6- UN VARÓN DE 37 AÑOS DE EDAD, TRABAJO POR VARIAS SEMANAS
COMO ALBAÑIL EN LA DEMOLICION Y REACONDICIONAMIENTO DE
GALPONES PARA GALLINAS HASTA HACE 1 MES. DESDE HACE 3 SEMANAS
PRESENTA MALESTAR GENERAL, DECAIMIENTO, TOS CON
EXPECTORACION Y FIEBRE. HA RECIBIDO LEVOFLOXACINA 5 DÍAS, PERO
LOS SINTOMAS NO CEDEN. LA EXPECTORACION HA AUMENTADO Y DESDE
HACE 2 DIAS PRESENTA DISNEA. EN LA RADIOGRAFIA DE TORAX SE
EVIENDICA PEQUEÑA MASA PARAHILIAR DERECHA. SI USTED ES EL
MÉDICO QUE LO RECIBE, COMO PRIMERA APROXIMACIÓN. ¿CUAL ES LA
SOLICITUD MÁS PERTINENTE?
BACILOSCOPIA SERIADA DE ESPUTO

7- UN RECIEN NACIDO. PRODUCTO DE LA PRIMERA GESTA DE UNA MADRE


DE 23 AÑOS. TUVO UNA LABOR DE PARTO DE 12 HORAS, RUPTURA DE
MEMBRANAS INTRA PARTO, LÍQUIDO AMNIÓTICO CLARO Y GRUMOSO.
NACIÓ DE PARTO NORMAL EN UN SERVICIO HOSPITALARIO. AL NACER
PRESENTO PESO DE 2.9 KG,TALLA DE 49, PC 34.5, APGAR 7/9. A LAS 24
HORAS DE EDAD PRESENTA ICTERICIA EN CARA Y TRONCO. LA
BILIRRUBINA INDIRECTA ES DE 7, LA DIRECTA DE 0.9. SE ENCUENTRA
ACTIVO, TIENE BUENA REACTIVIDAD AL MANEJO,TOMA SENO MATERNO
EXCLUSIVO, CON SUCCION ADECUADA, PRESENTO REGURGITACION POR
2 OCASIONES, EL GRUPO DE SU MADRE SE A+, EL RN O+ Y COOMBS
DIRECTO ES NEGATIVO ¿CUAL ES LA CAUSA DE LA ICTERICIA Y
CONDUCTA?
ICTERICIA FISIOLÓGICA: OBSERVAR EVOLUCIÓN Y REALIZAR
CONTROL DE BILIRRUBINAS
8- CAMILA DE 2 AÑOS 2 MESES,USTED REALIZA EL TEST DE DENVER Y
EVIDENCIA QUE DICE SU NOMBRE Y APELLIDOS, ARMA TORRE DE 6
CUBOS, SE PUEDE PONER Y SACAR LA CHOMPA, SALTA Y SE SEPARA DE
LA MADRE CON FACILIDAD. ESCOJA EL LITERAL CORRECTO
RELACIONADO CON LA CONSEJERÍA QUE DARÁ A LOS PADRES DE
CAMILA.
ELOGIA A LOS PADRES Y COMUNICA QUE CAMILA TIENE UN
DESARROLLO PSICOMOTOR ADECUADO Y BRINDA
INFORMACION SOBRE MAS ACTIVIDADES DE ESTIMULACIÓN

9- ALGUNAS PARASITOSIS INTESTINALES PUEDEN CAUSAR LA PÉRDIDA


CONSTANTE DE SANGRE DENTRO DEL HUESPED. SELECIONA CUAL
PARASITOSIS CAUSA ANEMIA FERROPENICA EN CASOS DE INFESTACION
MODERADA A SEVERA:
UNCINARIASIS (ANCILOSTOMA).
10- UN NIÑO DE 3 AÑOS DE EDAD ACUDE A EMERGENCIA POR
PRESENTAR DESDE HACE 2 DÍAS FIEBRE, VOMITO E IRRITABILIDAD. EN EL
EX FISICO SE ENCUENTRA LETARGIA, SIGNO DE KERNIG Y BRUDZINSKI
POSITIVOS, EL GRAM DEL LÍQUIDO ARROJA COMO RESULTADO UN COCO
BACILO GRAM NEGATIVO. ¿CUAL DE LAS SIGUIENTES OPCIONES
CORRESPONDE AL AGENTE ETIOLÓGICO MAS PROBABLE?
HAEMOPHILUS INFLUENZAE

11- UNA VEZ QUE LA HEMORRAGIA POSTPARTO INMEDIATA HA SIDO


IDENTIFICADA. INDIQUE EL TRATAMIENTO FARMACOLOGICO DE PRIMERA
OPCIÓN Y LA DOSIS CORRECTA.
OXITOCINA 10 UI/ML IM O 5 UI. IV LENTO O
20-40 UI EN 1.000 ML DE SOLUCION CRISTALOIDE EN INFUSIÓN
IV A 250 ML/H
12- EN CLAVE ROJA OBSTETRICA EN RELACIÓN A LA ADMINISTRACIÓN DE
UTEROTÓNICOS. ¿CUAL ES EL CRITERIO CORRECTO?
OXITOCINA 10UI/ML. IM Y 20-40 UI EN 1000ML DE SOLUCION
CRISTALOIDE EN INFUSION IV A 250ML/H
13- VARON DE 37 AÑOS LLEGA A LA EMERGENCIA DEL HOSPITAL AL POCO
TIEMPO DE UN ACCIDENTE DE TRANSITO. EL EX FISICO CONSTATA: TA
88/42, FC118, ABDOMEN LIGERAMENTE DISTENDIDO Y DOLOROSO. USTED
SOSPECHA DE TRAUMA ABDOMINAL CERRADO ¿CUAL DE LOS
SIGUIENTES PROCEDIMIENTOS EFECTUARIA PRIMERO?
ECOGRAFÍA AL PIE DE LA CAMA

14- LAS SIGUIENTES SON ANOMALIAS DE LA PARED ABDOMINAL


EXCEPTO:
QUISTE EPIPLOICO
15- A UN PACIENTE CON UN PRIMER EPISODIO PSICÓTICO. ¿QUE SE
APLICA UNA VEZ INSTAURADO EL TRATAMIENTO FARMACOLOGICO DE
APOYO TERAPEUTICO?
PLAN DE ACTIVIDAD FÍSICA Y NUTRICIONAL

16- VARON DE 23 AÑOS DE EDAD, PADECE DE OBESIDAD HACE 6 AÑOS Y


NO TIENE ANTECEDENTES DE ALTERACIONES MENTALES. DESDE HACE 2
SEMANAS NO SALE DE SU HABITACIÓN, MANIFIESTA ESTAR TRISTE Y
PREOCUPADO QUE SUS VECINOS PLANIFICAN MATARLO. REFIERE QUE
ESPIAN SUS MOVIMIENTOS Y ADEMAS SABEN LO QUE EL PIENSA. ¿CUAL
SERIA SU TRATAMIENTO DE ELECCION?
RISPERIDONA 6 MG/DIA

17- PACIENTE DE 29 AÑOS DE EDAD SIN ANTECEDENTES PATOLOGICOS,


DESDE HACE 5 DIAS PRESENTA FIEBRE, MALESTAR GENERAL,
DECAIMIENTO, TOS NO PRODUCTIVA Y DOLOR EN PUNTA DE COSTADO
DEL LADO DERECHO, ACUDE A CONSULTA Y AL EX FISICO TIENE TA
120/80, FC 98, FR 22, TEMPERATURA 39°C. TIENE INCREMENTO DEL
FREMITO Y BRONCOFONÍA EN REGIÓN SUBESCAPULAR DERECHA, EL
HEMITORAX IZQUIERDO ES NORMAL, RUIDOS CARDIACOS NORMALES,
ABDOMEN NORMAL, EN LA RADIOGRAFIA DE TORAX SE APRECIA IMAGEN
RADIO OPACA CON BRONCOGRAMA AEREO LIMITADO AL LOBULO
INFERIOR DERECHO, LA OXIMETRIA DE PULSO DE 91% AL AIRE AMBIENTE.
EN LA BIOMETRIA LEUCO 8500, SEG 70 Y LINF 30. LA CONDUCTA
APROPIADA SERA:
REPOSO EN CASA ADMINISTRAR AMOXACILINA CLAVULANATO
Y REEVALUAR A LAS 48 HORAS

18- UN HOMBRE DE 45 AÑOS, ASINTOMÁTICO ACUDE A CONTROL MÉDICO


DE RUTINA. AL EX FISICO LO RELEVANTE ES SU TA 130/84, IMC 26,
EXAMEN DE LABORATORIO TRIGLICERIDOS 150, COLESTEROL 240, LDL
180. EL PACIENTE DESEA CONOCER SI ESOS RESULTADOS SON
ANORMALES .SEGUN LA CLASIFICACION DE FREDRICKSON. ¿QUE TIPO DE
DISLIPIDEMIA TIENE ESTE PACIENTE?
II a

19- PACIENTE DE 29 AÑOS SIN ANTECEDENTES DE INTERES, REFIERE


QUE DESDE HACE 3 DÍAS PRESENTA DOLOR EN ZONA VULVAR Y VAGINA
NO SINTOMAS MICCIONALES. EN EL EXAMEN FISICO SE OBJETAN EN
PUBIS Y VAGINA LESIONES VESICULARES MÚLTIPLES DE 1 A 2 MM DE
DIAMETRO DE BORDES ERITEMATOSOS Y ADENOPATIAS INGUINALES
BILATERALES DOLOROSAS A LA PALPACION. NO SE OBSERVA SECRECION
VAGINAL NI URETRAL. ¿CUAL ES EL TRATAMIENTO INDICADO?
ACICLOVIR 400MG VO 3 VECES AL DÍA POR 14 DÍAS

20- EN RELACIÓN AL DENGUE CON SIGNOS DE ALARMA EN NIÑOS.


SELECCIONE LA AFIRMACIÓN CORRECTA
LA HEPATOMEGALIA DEBE CONSIDERARSE COMO UN FACTOR
DE RIESGO PARA CHOQUE POR DENGUE EN NIÑOS

21- UNA MUJER SOLTERA DE 25 AÑOS DE EDAD. ACUDE AL SERVICIO DE


EMERGENCIA POR PRESENTAR DESDE HACE APROXIMADAMENTE 24
HORAS, DOLOR DE CABEZA DE GRAN INTENSIDAD, CONTINUO, TIPO
CONTRICCION, LOCALIZADO EN REGION PARIETAL Y OCCIPITAL, QUE SE
INCREMENTA CON LA PALPACION Y CON LOS CAMBIOS BRUSCOS DE
POSICION. COMO ANTECEDENTES REFIERE CEFALEAS FRECUENTES, DE
MENOR INTENSIDAD QUE LA ACTUAL, POR LO QUE SE AUTO MEDICA
IBUPROFENO 400MG, HASTA 3 VECES AL DIA. AL EX. FISICO TA 100/60, FC
72, NO HAY EVIDENCIA DE PATOLOGIA EN OJOS, OIDOS O SENOS DE LA
CARA. DOLOR CERVICAL A LA DIGITO PRESION. DE ACUERDO A ESTOS
DATOS. ¿CUAL SERIA LA CAUSA MAS PROBABLE DE ESTE DOLOR?
DOLOR POR CONTRACTURA MUSCULAR CRÓNICA

22- UNA PACIENTE CAMPESINA DE 22 AÑOS DE EDAD PRESENTA UN


CUADRO COMPATIBLE CON PANCREATITIS AGUDA. EN LA
ECOSONOGRAFIA SE EVIDENCIA PANCREAS INCREMENTADO DE TAMAÑO,
VESICULA BILIAR LIBRE DE CALCULOS, PERO DENTRO DEL COLEDOCO SE
VISUALIZA UNA IMAGEN ALARGADA, SIN SOMBRA ACUSTICA DE UN CM DE
LONGITUD. EN ESTE CONTEXTO, ENTRE LAS SIGUIENTES ALTERNATIVAS.
¿CUAL ES EL PARASITO MAS PROBABLEMENTE IMPLICADO?
ASCARIS LUMBRICOIDES
23- RECIEN NACIDO DE 38 SEMANAS DE EDAD GESTACIONAL Y 3.450 GR
DE PESO AL NACIMIENTO. A LAS 8 HORAS DE VIDA SE LA ENCUENTRA
CON UNA FC 160, FR 68, SAT DE O2 92% RESPIRANDO AIRE AMBIENTAL,
TEMPERATURA 36.5, HIPOACTIVO, TEMBLOROSO Y CON SUCCION LENTA.
LOS EXAMENES REVELAN HTO 67% , GLICEMIA 35.
¿CUAL SERIA LA CONDUCTA MAS ADECUADA EN ESTE CASO?
CORREGIR LA POLIGLOBULIA

24- PACIENTE DE 51 AÑOS. ACUDE A CONSULTA POR PRESENTAR


DURANTE LOS ULTIMOS 2 AÑOS EPISODIOS DE SENSACION DE CALOR
INTENSO SEGUIDOS DE SUDORACION PROFUSA LOCALIZADOS EN CARA,
CUELLO Y TORAX. SON DE CORTA DURACION (MINUTOS), SE ASOCIAN A
PALPITACIONES Y ANGUSTIA.TIENEN UNA FRECUENCIA A 10 POR DIA Y EN
LA NOCHE LE IMPIDEN EL SUEÑO. TAMBIEN REFIERE SEQUEDAD VAGINAL
QUE PROVOCA DISPAREUNIA. LA FECHA DE LA ULTIMA MENSTRUACION
ES HACE 7 MESES Y SUS CICLOS MENSTRUALES HAN SIDO IRREGULARES
DESDE HACE 18 MESES. SI USTED SOSPECHA DE SINDROME
PREMENOPAUSICO EN EL QUE HAY INSUFICIENCIA OVÁRICA. ¿CUAL DE
LOS SIGUIENTES EXMENES EVIDENCIARIA MEJOR EL FEED BACK
(RETROALIMENTACION) NEGATIVO QUE SE PRODUCE?
HORMONA FOLICULO ESTIMULANTE

25- ¿CUAL ES EL MANEJO INICIAL DE UN VARON CON RETENCION AGUDA


DE ORINA CAUSADA POR HEMATURIA CON COAGULOS?
COLOCAR UN CATETER DE TRES VÍAS

26- LAS SIGUIENTES INDICACIONES DE LAPAROTOMIA EN PACIENTES CON


LESIÓN PENETRANTE DE ABDOMEN.EXCEPTO
DISTENCION ABDOMINAL
27- PACIENTE DE 60 AÑOS, ACUDE A LA CONSULTA POR PRESENTAR
SANGRADO VAGINAL DESPUES DE 5 AÑOS DE MENOPAUSIA, TIENE HTA Y
DM TIPO 2 DESDE HACE 8 AÑOS, GESTA 0, IMC 31, TA 155/95, RESTO DEL
EX FISICO ES NORMAL, AL EX GENITAL REVELA MUCOSA VAGINAL
TROFICA Y EL SANGRADO PROVENIENTE DEL ORIFICIO EXTERNO DEL
CUELLO DEL UTERO,CERVIX SANO, UTERO Y ANEXOS NORMALES.¿ EN EL
MANEJO DE ESTA PACIENTE EL SIGUIENTE PASO ES?
ECOGRAFIA PELVICA

28- ESCOJA LA RECOMENDACIÓN CORRECTA EN RELACION A LA


ALIMENTACIÓN DE UN LACTANTE MENOR DE 6 MESES DE EDAD CON
LECHE MATERNA SEGUN MSP, UNICEF Y OMS.
A LIBRE DEMANDA

29- PACIENTE MASCULINO DE UN AÑO DE EDAD, ACUDE A LA CONSULTA


PORQUE SUS PADRES INDICAN QUE EN OCASIONES NO SIENTEN EL
TESTICULO DEL LADO DERECHO, SOBRE TODO CUANDO LO BAÑAN. EL
EX FISICO REVELA QUE EL TESTICULO DERECHO SE ENCUENTRA UN
POCO ELEVADO, PERO CON FACILIDAD ES LLEVADOA LA BOLSA
ESCROTAL Y PERMANECE EN EL. EL MANEJO APROPIADO DEL PACIENTE
ES:
EVALUAR CADA AÑO LA EVOLUCION DEL CUADRO
30- MUJER DE 19 AÑOS, SIN ANTECEDENTES PATOLOGICOS DE
IMPORTANCIA, NO TIENE HIJOS Y FUM HACE 15 DIAS, VIVE EN ZONA
RURAL DE ESMERALDAS DESDE HACE 1 SEMANA PRESENTA FIEBRE
INTERMITENTE, ESCALOFRIOS, MALESTAR GENERAL, ANOREXIA Y
NAUSEA. AL EX FISICO: TA 100/60, FC 88, FR 16, ESTA CONSCIENTE
ORIENTADA , MUCOSAS HUMEDAS , LOS EXAMENES HTO 42, CREAT 0.9 Y
GOTA GRUESA CONFIRMA PLASMODIUM VIVAX. ¿CUAL ES EL ESQUEMA
DE TRATAMIENTO DE PRIMERA ELECCION SEGUN LOS PROTOCOLOS DEL
CONSEJO NACIONAL DE SALUD DEL ECUADOR?
CLOROQUINA MAS PRIMAQUINA LOS 3 PRIMEROS DIAS
SEGUIDOS DE PRIMAQUINA LOS CUATRO DIAS SIGUIENTES.

31- SEÑALE EL OBJETIVO TERAPEUTICO PARA CONTROL DE


HIPERLIPIDEMIA QUE DEBE TENER UN PACIENTE ADULTO CON DIABETES:
TRIGLICÉRIDOS 150MG/DL (1.7MMOL/L)

32.- EL MEDICAMENTO QUE DISMINUYE LA HEMORRAGIA EN EL


SANGRADO DIGESTIVO ALTO DEL PACIENTE CIRRÓTICO ES:
OCTEOTRIDE
33.- UNO DE LOS TIPOS DE DEMENCIA SENIL ES UN TRASTORNO
NEUROCOGNITIVO MAYOR, CON CARGA GENÉTICA, QUE EVOLUCIONA
PROGRESIVAMENTE EN UN PERIODO DE 20 AÑOS. ES MAS FRECUENTE
EN MUJERES DESPUES DE LOS 60 AÑOS, ENTRE SUS SINTOMAS SE
INCLUYE ALTERACIONES EN LA MEMORIA A LARGO PLAZO, AFASIA,
APRAXIA, DESORIENTACION, DEPRESION, CAMBIOS EN LA CONDUCTA Y
COMPORTAMIENTO. SEÑALE EL DIAGNÓSTICO CORRESPONDIENTE:
DEMENCIA ALZHEIMER
34- ¿QUE GRUPO FARMACOLÓGICO SE RECOMIENDA EN PACIENTES
ADULTOS CON DEPRESIÓN MAYOR QUE PRESENTAN IDEACIÓN SUICIDA?
INHIBIDORES SELECTIVOS DE LA RECAPTACIÓN DE
SEROTONINA
35- ADOLESCENTE DE 14 AÑOS DE EDAD, REFIERE QUE DESDE HACE 3
DIAS PRESENTA DOLOR EN SU OIDO DERECHO, EL CUAL HA IDO
AUMENTANDO PROGRESIVAMENTE DE INTENSIDAD. ADEMAS, DESDE LA
NOCHE DE AYER PRESENTA ALZA TREMICA A 40°C, SU MEDICO REALIZA
EX. OTOSCOPICO Y ENCUENTRA UNA MODERADA SUPURACIÓN DE
COLOR AMARRILLO. EN UN PRINCIPIO EL PRESCRIBE PARACETAMOL
500MG Y AMOXACILINA 500MG CADA 8 HORAS DURANTE 3 DIAS, ANTE LA
PERSISTENCIA DE LA SINTOMATOLOGIA DESPUES DE 48 HORAS.
¿CUAL SERIA EL ANTIBIOTICO DE ELECCCION?
CEFUROXIMA.

36- ¿CUAL ES LA INDICACION PARA REFERIR A UN PACIENTE A UN


CENTRO ESPECIALIZADO PARA LA ATENCION DE QUEMADOS?
QUEMADURAS QUIMICAS, DE CUALQUIER MECANISMO DE
ACCION

37- UN NIÑO DE 18 MESES DE EDAD, TUVO UN RESFRIADO COMUN HACE 5


DIAS Y DESDE HACE 48 HORAS PRESENTA FIEBRE DE 38°C E
IRRITABILIDAD, AL REALIZAR LA OTOSCOPIA SE ENCUENTRA EL TIMPANO
ERITEMATOSO, ABOMBADO Y NO SE VISUALIZA EL TRIANGULO
LUMINOSO. ¿ADEMAS DE RECIBIR TRATAMIENTO SINTOMÁTICO.¿CUAL
SERIA LA MEJOR ALTERNATIVA ANTIMOCROBIANA?
AMOXACILINA
38- UN GUIA DE TURISMO DE SELVA. ACUDE POR PRESENTAR UNA
LESION EN LA PIEL DEL DORSO DE LA MUÑECA PROVOCADA POR UNA
PICADURA DE INSECTO, DESDE HACE UN PAR DE MESES, LA PAPULA
TOMO UN COLOR VIOLETA, TIENDE A CRECER Y A ULCERARSE. AL EX.
FISICO VEMOS UNA LESION PAPULAR INDURADA , COLOR ROJO VINOSO,
CON HALO ERITEMATOSO DE UNOS 6 CM DE DIAMETRO, CUBIERTA DE
UNA CAPA ESCAMOSA Y EN EL CENTRO TIENE UNA COSTRA
REDONDEADA DE 4 CM DE DIAMETRO. EL RESULTADO DEL FROTIS
ENCUENTRAN MACROFAGOS Y ESCASOS MICROORGANISMOS EN
FORMAS AMASTIGOTES
¿CUAL DE LOS SIGUIENTES FARMACOS ESTA INDICADO COMO TTO. DE
PRIMERA LINEA PARA ESTA LESION?
ANTIMONIATO DE MEGLUMINA

39- ¿CUAL DE LAS SIGUIENTES SITUACIONES DETERMINA UN RESULTADO


FALSO POSITIVO EN LAS PRUEBAS DE TAMIZAJE PARA VIH?
VACUNACIÓN CONTRA LA GRIPE
40- EL SIGUIENTE ESTUDIO DE LABORATORIO CLINICO ES DIAGNOSTICO
DE DESNUTRICION AGUDA:
CONCENTRACION SERICAS DE ALBUMINA MENORES A 2.8
G/100ML
41- DE LAS SIGUIENTES OPCIONES: ¿CUAL ES LA COMPLICACION
METABÓLICA DE LA PANCREATITIS AGUDA?.
HIPERGLUCEMIA

42- LAS BENZODIAZEPINAS SON UTILIZADAS EN EL TTO SINTOMÁTICO DE


LOS TRASTORNOS DE PANICO. ¿CUAL ES EL MECANISMO
FARMACOLOGICO POR EL QUE LOGRA SU EFECTO ANSIOLITICO?
LA ACCION DEL FARMACO SOBRE EL RECEPTOR GABA-A
80- UNA NIÑA DE 5 MESES DE EDAD ES TRAIDA A CONTROL DE SALUD, EL
MEDICO DESCUBRE QUE LA NIÑA HA INICIADO LA ABLACTACION DESDE
HACE 1 MES, DEBIDO QUE LA MADRE DICE TENER POCA LECHE MATERNA
Y YA COMENZO A TRABAJAR, ¿A QUE EDAD DEBIO HABERSE INICIADO LA
ABLACTACION DE ACUERDO A LAS RECOMENDACIONES DE LA OMS?
6 MESES
43- UN MADRE PRIMIGESTA, SIN FACTORES DE RIESGO ACUDE A
EMERGENCIA CON ACTIVIDAD UTERINA IRREGULAR, SE DETECTA
DURANTE LA LABOR BRADICARDIA FETAL Y LA AMNIOCENTESIS REVELA
LIQUIDO MECONIAL, EL RECIEN NACIDO A TERMINO NACE HIPOTÓNICO,
CON FC100 Y APNEICO, RECIBE PRESION POSITIVA DURANTE 30
SEGUNDOS, SU APGAR FUE DE 5/7/8 (AL PRIMER MINUTO, A LOS 5
MINUTOS Y A LOS 10 MINUTOS RESPECTIVAMENTE) A LAS 2 HORAS DE
VIDA SE LO ENCUENTRA IRRITABLE, HIPOTONICO Y QUEJUMBROSO.
¿CUAL ES EL DIAGNOSTICO MAS PROBABLE?
ENFERMEDAD HIPOXICO-ISQUEMICA.
44- UNA JOVEN DE 17 AÑOS TIENE ABUNDANTE CASPA Y ESCASAS
LESIONES SEBORREICAS EN CEJAS Y CONDUCTO AUDITIVO EXTERNO,
LOS PARPADOS, PLIEGUES NASOFARINGEOS Y EL RESTO DEL CUERPO
ESTAN RESPETADOS. ¿CUAL DE LOS SIGUIENTES FARMACOS TOPICOS
ESTA INDICADO EN ESTA CONDICION?
KETOCONAZOL

45- UNA PACIENTE DE 35 AÑOS DE EDAD SIN ANTECEDENTE PATOLOGICO


DE IMPORTANCIA , HACE 7 DIAS TUVO UNA INFECCION BACTERINA AGUDA
DE LAS VIAS RESPIRATORIAS ALTAS Y ESTA RECIBIENDO TTO CON
AMPICILINA 500MG CADA 6 HORAS, PARACETAMOL 1GR CADA 8 H.
ACTUALMENTE ACUDE A CONSULTA POR PRESNTAR ANOREXIA Y
NAUSEAS, AUNQUE SIN VOMITO. ADEMAS, HA NOTADO UN TINTE ROJIZO
EN LA ORINA. EX. FISICO TA 120/80, FC 80, FR16, TEMP 36.8°C Y EDEMA
PALPEBRAL BILATERAL. EX DE SANGRE LEUCO 7500, LINFO 20, EOSINO
14, NEUTROFILO 60 Y MONOCITOS 6, GLICEMIA 80, CREATININA 2.6 Y UREA
DE 26, SOLICITA UN EXAMEN DE ORINA QUE REPORTA: DENSIDAD 1.015,
PH 5.5, COLOR ROJIZO, LIGERAMENTE TURBIO, PROTEINAS 30, HB
POSITIVO, PIOCITOS 20, PRESENCIA DE CILINDROS GRANULOSOS,
ERITROCITARIOS Y LEUCOCITARIOS. ADEMAS SE OBSERVAN
LEUCOCITOS POLIMORFONUCLEARES EOSINOFILOS, LA FRACCION
EXCRETADA DE SODIO ES MAYOR A 1%, EL VALOR DE COMPLEMENTO C3
ES NORMAL
¿CUAL ES EL DIAGNOSTICO MAS PROBABLE?
NEFRITIS INTERSTICIAL
46- SEÑALE UN SIGNO QUE REPRESENTE DÉFICIT AGUDO DE VOLUMEN
EXTRACELULAR:
HIPERAZOEMIA
47- PACIENTE DE 49 AÑOS CON ANTECEDENTE DE HTA Y DIABETES
GESTACIONAL PREVIA, EN SU UNICO EMBARAZO, MENSTRUACIONES
REGULARES NORMALES HASTA 3 AÑOS, EN QUE SE HICIERON MAS
FRECUENTES Y PROLONGADAS, 3-4 SEMANAS SANGRANDO, IMC 29.4. TA
146/92. EX. GENITAL PRESENTA SANGRADO UTERINO,EL RESTO DEL
APARATO ES NORMAL ¿ CUAL DE LOS SIGUIENTES PROCEDIMIENTOS
USARIA INICIALMENTE PARA MANEJAR EL CASO?
ECOGRAFIA TRANSVAGINAL

48- CON RESPECTO AL USO DEL DISPOSITIVO INTRAUTERINO DE COBRE


COMO METODO DE PLANIFICACION FAMILIAR. SEÑALE CUAL ES SU
CONTRAINDICACION DE USO
METRORRAGIA DE ORIGEN DESCONOCIDO

49- MUJER DE 19 AÑOS, SOLTERA. ACUDE A CONSULTA POR PRESENTAR


AUMENTO DE LA SECRECION VAGINAL DESDE HACE 24 HORAS, ASOCIADA
A LEVE SINTOMATOLOGIA URINARIA, AGO G0, FUM HACE 20 DIAS,
RELACION SEXUAL HACE 6 DIAS CON PAREJA OCASIONAL. EX. ESPECULO
REVELA VULVA Y VAGINA NORMALES, CERVIX POSTERIOR CON LIGERO
ERITEMA, POR CUYO ORIFICIO EXTERNO SE EXTERIORIZA SECRECION
AMARRILLO PURULENTA, EL T.V DOLOROSO A LA MOVILIZACION Y
ANEXOS NO PALPABLES PERO SENSIBLES. ¿CUAL DE LAS SIGUIENTES
OPCIONES ES EL DIAGNOSTICO MAS PROBABLE?
GONORREA.
50- UN PACIENTE DE 3 AÑOS DE EDAD TUVO HACE 6 DIAS UN RESFRIO
QUE RESOLVIO ESPONTANEAMENTE, DESDE HACE 2 DIAS PRESENTA
UNAS LESIONES VESICULARES NO DOLOROSAS NI PRURIGINOSAS
ALREDEDOR DE LAS FOSAS NASALES, LAS CUALES SE ENCUENTRAN
CUBIERTAS DE UN EXUDADO AMARILLENTO DE ASPECTO PARECIDO A LA
MIEL. ¿CUAL SERIA LA CONDUCTA MAS ADECUADA?
ADMINISTRAR DICLOXACILINA.

51- NIÑA DE 1 AÑO 6 MESES QUE DURANTE SU CONTROL DE SALUD


PRESENTA LA SIGUIENTE CURVA DE CRECIMIENTO. ¿COMO SE
INTERPRETA LA SIGUIENTE CURVA DE CRECIMIENTO?
LA NIÑA NO ESTA SUBIENDO DE PESO Y TIENE UNA SEÑAL DE
PELIGRO.

52- UNA MUJER DE 25 AÑOS, SIN ANTECEDENTE PSIQUIATRICOS. ES


INGRESADA AL SERVICIO DE EMERGENCIA, INFORMA PERDIDA DE
MEMORIA FRECUENTEMENTE, EXPLICANDO QUE A MENUDO ENCUENTRA
EVIDENCIA DE REALIZAR ACCIONES QUE NO RECUERDA, ADEMAS
EXPLICA QUE DURANTE ESTAS PERDIDAS DE MEMORIA, FAMILIARES Y
AMIGOS HAN NOTADO QUE SU PERSONALIDAD CAMBIA CLARAMENTE. LA
MADRE DE LA PACIENTE AFIRMA SU FAMILIA TIENE UNA SÓLIDA BASE
RELIGIOSA Y QUE LOS SÍNTOMAS DE LA PACIENTE SON DE POSESIÓN
DIABÓLICA, EN LA EVALUACIÓN SE DEMUESTRA QUE LA PACIENTE NO
TIENE MEMORIA DE NINGUNO DE LOS INCIDENTES Y SUS FUNCIONES
MENTALES SUPERIORES ESTÁN NORMALES, SALVO UNA LEVE
HIPERTIMIA DISPLACENTERA. EL EX. TOXICOLOGICO NEGATIVO AL IGUAL
QUE TAC. ¿CUAL ES LA PATOLOGÍA DE LA PACIENTE?
TRASTORNO DE IDENTIDAD DISOCIATIVO.

53- DE ACUERDO A LA OMS: ¿CUAL DE LAS SIGUIENTES OPCIONES


REPRESENTA LA PRINCIPAL CAUSA DE MUERTE MATERNA?
HEMORRAGIA POST PARTO. (CAUSAS: 2. INFECCIONES, 3. HT
GESTACIONAL. 4. COMPLICACIONES EN EL PARTO, 5. ABORTOS
PELIGROSOS).

53- UNA PACIENTE TIENE UNA TENSIÓN ARTERIAL DE 160/95 AL MOMENTO


DEL PARTO Y A LAS 12 SEMANAS POSTERIORES 150/90, EL DIAGNOSTICO
ES:
HIPERTENSIÓN CRÓNICA.
54- UN NIÑO DE 3 AÑOS DE EDAD ACUDE A CONSULTA PORQUE DESDE
HACE 5 DÍAS PRESENTA DEPOSICIONES BLANDAS, CON MOCO, SIN
SANGRE, EN UNA FRECUENCIA DE 4 POR DIA. ADEMAS, A PRESENTADO
FIEBRE Y VOMITOS EN DOS OCASIONES. AL EX. FISICO NO SE LO
ENCUENTRA DESHIDRATADO. ¿CUAL ES LA MEJOR CONDUCTA EN ESTE
CASO?
UTILIZAR EL PLAN A DE LA OMS. (TRATAMIENTO EN EL HOGAR
PARA PREVENIR LA DESHIDRATACION Y LA DESNUTRICION).

55- ANA ES UNA MUJER QUE TIENE UNA HIJA DE 4 SEMANAS, ESTA
CANSADA Y EN LA NOCHE LE DA EL SENO ACOSTADA, PERO HACE 72
HORAS PRESENTA FIEBRE ACOMPAÑADA DE ERITEMA DE 2 CM DE
DIAMETRO EN LA MAMA DERECHA, DOLOR DE MODERADA INTENSIDAD Y
SIENTE COMO UN BULTO DE MAS O MENOS 1CM DE DIAMETRO EN EL
SENO DERECHO. ¿CUAL ES LA OPCCION TERAPEUTICA CORRECTA?
LE PRESCRIBE DICLOXACILINA, ANALGÉSICO Y LE EXPLICA LA
POSICIÓN CORRECTA DE LACTAR, INCENTIVANDO LA
LACTANCIA MATERNA.
56- UN NIÑO DE 4 AÑOS DE EDAD PRESENTA DESDE HACE 24 HORAS
DEPOSICIONES DIARREICAS LIQUIDAS, EN MODERADA CANTIDAD, SIN
MOCO NI SANGRE, HA REALIZADO 6 DEPOSICIONES EN 24 HORAS, NO
PRESENTA VOMITOS NI FIEBRE, EN ESTE TIEMPO HA COMIDO POCO Y HA
RECIBIDO POCA CANTIDAD DE AGUAS AROMÁTICAS, SE DIAGNOSTICA
DESHIDRATACION ISOTONICA LEVE (1er GRADO) Y SE DECIDE INICIAR
TRO (TERAPIA DE REHIDRATACION ORAL), EN EL MERCADO EXISTEN
DISTINTAS SOLUCIONES COMERCIALES PARA REHIDRATACION ORAL.
¿ CUAL DE LAS SIGUIENTES SOLUCIONES CORRESPONDE A LA
PROPUESTA POR LA OMS PARA TRO?
SODIO 75 MMOL/L. OSMOLARIDAD TOTAL 245 MOSM/L.

57- UN PERSONA CON DM QUE SE CONTROLA CON ANTIDIABETICOS


ORALES E INSULINA POR DECISIÓN PROPIA DEJO DE ADMINISTRARSE LA
INSULINA DESDE HACE 2 SEMANAS, USTED RECIBE A LA PACIENTE CON
CETOACIDOSIS DIABÉTICA Y LA RESPUESTA VENTILATORIA A LA
ACIDOSIS METABÓLICA ES LA ESPERADO. ¿CUAL ES EL LIMITE INFERIOR
DEL PH SANGUÍNEO ANTE EL CUAL SE PODRIA ADMINISTRAR
BICARBONALTO DE SODIO POR VIA ENDOVENOSA?
6.99
58 - UN PACIENTE DE 62 AÑOS SIN COMORBILIDADES, ACUDE A SU
CONSULTA PORQUE DESDE HACE 4 HORAS PRESENTA UN DOLOR
ABDOMINAL TIPO RETORTIJON DE INTENSIDAD 6/10 Y NAUSEA SIN
LLEGAR AL VOMITO. HA REALIZADO UNAS 8 DEPOSICIONES LIQUIDAS,
ABUNDANTES, SIN TENESMO NI MOCO. COMO PARTE DEL TRATAMIENTO
USTED PRESCRIBE SUBSALICILATO DE BISMUTO EN DOSIS TERAPEUTICA.
¿CUAL DE LAS SIGUIENTES COMPLICACIONES PODRIA ATRIBUIRSE AL
FARMACO PRESCRITO EN ESTE CASO?
AUMENTA LA PROBABILIDAD DE HIPOGLICEMIA.
59- SEÑALE EN QUE CASO SE RESPETA LA OBJECIÓN DE CONCIENCIA DEL
MEDICO:
SUICIDIO MEDICAMENTE ASISTIDO.
60- MUJER DE 63 AÑOS QUE ACUDE A EMERGENCIA POR PRESENTAR
DOLOR ABDOMINAL AGUDO TIPO COLICO E INCAPACIDAD PARA ELIMINAR
FLATOS, LA PACIENTE SUFRE DE CONSTIPACION CRONICA Y TOMA
LAXANTES DIARIAMENTE. AL EX. FISICO FC 97, TA 140/95, TEMP 38.1,
DESHIDRATADA, ABDOMEN DISTENDIDO, DOLOROSO Y TIMPANICO,
AUSENCIA DE PERISTALTISMO INTESTINAL, EL MEDICO A CARGO DEL
PACIENTE SE PLANTEA DIAGNOSTICO DE VOLVULO DE COLON. ¿CUAL DE
LOS SIGUIENTES HALLAZGOS ES MAS SUGERENTE DE NECESIDAD DE
INTERVENCIÓN QUIRÚRGICA?
PROCTORRAGIA CON ELIMINACIÓN DE MUCOSA COLONICA
OSCURA.
61- SON CRITERIOS DECENTOR PARA EL DIAGNOSTICO DE AMIGDALITIS.
EXCEPTO:
PRESENCIA DE TOS
62- PACIENTE MASCULINO DE 60 AÑOS CON ANTECEDENTE DE
ESTREÑIMIENTO Y OBESIDAD. ACUDE POR CUADRO DE DOLOR
ABDOMINAL EN EL CUADRANTE INFERIOR IZQUIERDO QUE SE ACOMPAÑA
DE ASTENIA Y TAC DE ABDOMEN EN LA QUE SE EVIDENCIA UNA
DIVERTICULITIS DEL COLON SIGMOIDEO ASOCIADO A UN ABSCESO
MENOR A 2 CM. EL TRATAMIENTO ES:
ANTIBIOTICOTERAPIA

63- EL INCREMENTO TOTAL DE PESO RECOMENDADO (KG) AL FINAL DEL


EMBARAZO GEMELAR Y CON UN IMC NORMAL PREGESTACIONAL ES DE:
17-25KG.

64- UN PACIENTE DE 21 AÑOS DE EDAD SIN ANTECEDENTE CLINICOS DE


IMPORTANCIA PRESENTA UN CUADRO COMPATIBLE CON NEUMONIA
COMUNITARIA Y DECIDE HOSPITALIZACION, 3 DIAS MAS TARDE CON
ADECUADA EVOLUCION CLINICA, USTED SOLICITA UNA RADIOGRAFIA DE
TORAX PARA CONTROL PREVIO AL ALTA, PERO TODAVIA PERSISTE EL
INFILTADO RADIOLOGICO, EN ESTAS CONDICIONES Y EXPRESADO EN
SEMANAS, USUALMENTE ¿ CUAL ES EL MINIMO TIEMPO PROMEDIO QUE
DEMORA EN DESAPARECER LAS ALTERACIONES RADIOLOGICAS?
4 SEMANAS.

65- UN ADULTO DE 55 AÑOS DE EDAD PREVIAMENTE SANO. ACUDE A SU


CONSULTA POR TOS SECA DE 8 DIAS DE EVOLUCION , QUE DESDE HACE 4
DIAS SE ACOMPAÑA DE FIEBRE Y POCA EXPECTORACION, DESDE HACE
24 HORAS REFIERE SENTIR DOLOR TORACICO DERECHO. AL EX. FISICO
EL PACIENTE ESTA CONSCIENTE, ORIENTADO, TIENE UNA TA 100/60, FC
110, FR 18, SE AUSCULTAN CREPITANTES SUBESCAPULARES DERECHOS,
EN SANGRE LOS LEUCOCITOS ESTAN EN 8500, SEG 66, LINF 28,
CREATININA 0.9, UREA 20. LA RADIOGRAFIA DE TORAX MUESTRA UN
PEQUEÑO INFILTRADO INTERSTICAL EN BASE DERECHA.¿CUAL ES EL
PUNTAJE DEL CURB 65?
0 PUNTOS
66- SE TRATA DE UN PACIENTE JOVEN, SIN PATOLOGIA PREVIA, QUE 1
HORA ATRAS, FUE HERIDO CON UN OBJETO PUNZANTE EN LA CARA
LATERAL DE SU HEMITORAX DERECHO, AL MOMENTO, LOS SIGNOS
VITALES SON ESTABLES, PERO EN LA RADIOGRAFIA DE TORAX SE
APRECIA CLARAMENTE UN NEUMOTORAX DEL 20%. ¿CUAL ES LA
CONDUCTA RECOMENDADA?
MANTENER EN OBSERVACIÓN AL PACIENTE
67- UN PACIENTE DE 64 AÑOS DE EDAD , ACUDE POR PRESENTAR DE
MANERA BRUSCA UN CUADRO DE DOLOR PRECORDIAL OPRESIVO E
INTENSO DE 4 HORAS DE EVOLUCION, DIAFORESIS Y ORTOPNEA, SU TA
110/70, EL EKG FC 100, DEPRESION DEL SEGMENTO ST EN LAS
DERIVACIONES V2,V3 Y V4. ¿CUAL DE LOS SIGUIENTES FARMACOS ESTA
CONTRAINDICADO ADMINISTRAR EN ESTE CASO PARTICULAR?
FIBRINOLITICO.

68- EN UN PACIENTE DE 65 AÑOS DE EDAD FUMADOR DE 20 CIGARRILLOS


DIARIOS HASTA HACE 5 AÑOS, CON DIAGNOSTICO DE EPOC GRADO LEVE.
ACUDE POR UNA EXCERBACION DE SU ENFERMEDAD, COMO PARTE DEL
MANEJO ACTUAL LE ADMINISTRAN CORTICOIDES SITÉMICOS. ¿CUANTO
TIEMPO DEBE MANTENER ESTE TRATAMIENTO?
ENTRE 10 Y 14 DIAS.

69- UN VARON DE 18 AÑOS DE EDAD PREVIAMENTE SANO, CON HABITUAL


COMPORTAMIENTO SEXUAL DE RIESGO ULTIMO HACE 10 DIAS, INDICA
QUE HACE 5 DIAS PRESENTO ENROJECIMIENTO DE LOS 2 OJOS CON
SECRECION PURULENTA, SE HA AUTOMEDICADO CON CIPROFLOXACINA
TOPICA CADA 4 HORAS DURANTE 5 DIAS CONSECUTIVOS SIN ALIVIO,
ACUDE A CONSULTA POR PRESENTAR EDEMA PALPEBRAL SEVERO
BILATERAL, OJOS ENROJECIDOS, ABUNDANTE SECRECIÓN PURULENTA Y
VISIÓN BORROSA BILATERAL AL EXAMEN FÍSICO SE CORROBORA LOS
HALLAZGOS OCULARES, CON ABUNDANTE SECRECIÓN VERDOSA.
RESTO DEL EXAMEN FISICO NORMAL. USTED SOSPECHA EN
CONJUNTIVITIS POR GONOCOCO, ADEMAS DE LAVADOS PROFUSOS CON
SOLUCIÓN SALINA ESTÉRIL Y ANTIBIÓTICOS TÓPICOS. ¿CUAL ES EL
TRATAMIENTO SISTÉMICO DE PRIMERA ELECCIÓN EN ESTE CASO?
CEFTRIAXONA SÓDICA 1 GRAMO CADA 12 HORAS VIA
MUSCULAR DURANTE 5 DIAS.
70- SE TRATA DE UN PACIENTE DE 33 AÑOS CON ANTECEDENTE DE BULLA
PULMONAR DERECHA, MIENTRAS REALIZABA UNA ACTIVIDAD FISICA
PRESENTO DOLOR INTENSO EN HEMITORAX DERECHO, ACOMPAÑADO DE
DISNEA DE REPOSO Y SIGNOS FISICOS DE NEUMOTORAX SIMPLE, LA PO2
ESTA REDUCIDA, LA PCO2 ELEVADA, PERO SU SATURACIÓN DE O2 SUBE
SIGNIFICATIVAMENTE ANTE LA ADMINISTRACIÓN DE O2 SUPLEMENTARIO.
¿CUAL ES EL PRINCIPAL MECANISMO DE HIPOXEMIA EN ESTE PACIENTE?
REDUCCION DE LA SUPERFICIE DE INTERCAMBIO.
71- VARON DE 19 AÑOS, PRESENTA DESDE HACE 2 DIAS FIEBRE,
MIALGIAS, DISFONIA, ODINOFAGIA Y TOS. LOS SINTOMAS SE
INCREMENTAN POR LO CUAL ACUDE A CONSULTA. EN EL EX FISICO TEMP
40°C, FC100, FR 18, RINORREA SEROSA, OROFARINGE CONGESTIVA CON
ADENOPATIAS RETROAURICULARES, AUSCULTACIÓN RESPIRATORIA
NORMAL. ¿CUAL SERIA LA MEJOR CONDUCTA EN ESTE CASO?
CULTIVO DE SECRECIÓN FARÍNGEA
72- UN ESCOLAR DE 10 AÑOS PRESENTA HACE 3 DIAS DIARREAS
LIQUIDAS ABUNDANTES, FETIDAS, CON MOCO Y UN POCO DE SANGRE,
CON UNA FRECUENCIA DE 3 A 4 VECES AL DIA, SE ACOMPAÑA DE DOLOR
ABDOMINAL TIPO COLICO DE MODERADA INTENSIDAD, SE REALIZA
COPROPARASITARIO QUE REPORTA LA PRESENCIA DE BALANTIDIUM
COLI. SELECCIONA EL FARMACO CORRECTO PARA TRATAR ESTA
INFESTACION:
METRONIDAZOL
73- UN MADRE PRIMIGESTA, SIN FACTORES DE RIESGO ACUDE A
EMERGENCIA CON ACTIVIDAD UTERINA REGULAR, SE DETECTA DURANTE
LA LABOR, BRADICARDIA FETAL Y AMNIOCENTESIS INSTRUMENTAL
REVELA LIQUIDO MECONIAL, EL RECIEN NACIDO A TERMINO NACE
HIPOTONICO, CON UNA FC 80 AL MINUTO Y APNEICO, RECIBE PRESION
POSITIVA DURANTE 90 SEGUNDOS, SU APGAR FUE DE 4/6/8 (AL PRIMER
MINUTO, A LOS 5 MIN Y A LOS 10MIN). ¿QUE COMPLICACION PODRIA
PRESENTARSE DURANTE LAS PRIMERAS HORAS?
HIPOCALCEMIA.
64- LA EVOLUCION NATURAL DE LA NEFROPATIA DIABETICA SE
CARACTERIZA POR LOS SIGUIENTES EVENTOS. EXCEPTO:
PROSTATITIS.
65- UN NIÑO DE 5 AÑOS DE EDAD PRESENTA DESDE HACE 12 HORAS
DIARREAS CON MOCO, PUS Y SANGRE POR 6 OCASIONES, EN ESTE
TIEMPO HA PRESENTADO 2 CONVULSIONES TONICO CLONICAS
GENERALIZADAS DE 20 MINUTOS CADA UNA. EL NIÑO SE ENCUENTRA
ALETARGADO, AUNQUE RESPONDE A ESTIMULOS TACTILES, SU TEMP
39.3. ABDOMEN DOLOROSO EN FORMA DIFUSA, NO HAY SIGNO DE
PLIEGUE EN ABDOMEN, MUCOSAS ORALES SEMIHUMEDAS, SU
CONDICION SE CONSIDERA GRAVE E INGRESA PARA TRATAMIENTO
HOSPITALARIO, SELECCIONE EL POSIBLE GERMEN CAUSANTE Y EL
TRATAMIENTO INDICADO PARA ESTE CASO:
SHIGELLA DYSENTERIAE.
CEFTRIAXONA PARENTERAL POR 5 DIAS
66- PACIENTE EN EL TERCER DIA POSTPARTO , QUE PRESENTA TEMP 38.4
Y MALESTAR GENERAL, LO OTROS SIGNOS VITALES SON NORMALES,
TIENE ABDOMEN SUAVE , FONDO UTERINO DOLOROSO,A NIVEL DEL
OMBLIGO, LOQUIOS ASALMONADOS DE MAL OLOR. ¿CUAL DE LOS
SIGUIENTES GÉRMENES ESTA COMÚNMENTE ASOCIADO A LA
ENDOMETRITIS POST PARTO?
ESCHERICHIA COLI

67- RELACIONE LAS SIGUIENTES SUSTANCIAS QUE PUEDEN SER


EMPLEADAS CON FINES SUICIDAS, CON EL ANTIDOTO CORRECTO:
1. BENZODIAZEPINAS d FLUMAZENIL
2. MONOXIDO DE CARBONO a OXIGENO
3. ORGANOS FOSFORADOS e ATROPINA
4. PARACETAMOL b N-ACETIL CISTEINA
5. OPIACEOS f NALOXONA
6. ANTIDEPRESIVOS c BICARBONATO DE SODIO
68- EN UN EXAMEN ELEMENTAL MICROSCOPICO DE ORINA REALIZADO EN
PACIENTES SIN SINTOMAS ATRIBUIBLES A INFECCION DE VIAS
URINARIAS, QUE HA SIDO RECOLECTADO Y PROCESADODE MANERA
ADECUADA, EL REPORTE ES EL SIGUIENTE: ORINA AMARILLO, ASPECTO
TRANSPARENTE, OLOR SUIGENERIS, DENSIDAD1020, PH5.5, NITRITOS,
GLUCOSA, CETONAS, PROTEINAS, SANGRE, BILIRRUBINA,
UROBILINOGENO Y LEUCOCITOS SON NEGATIVOS. AL MICROSCOPIO:
HEMATIES 1 POR CAMPO. PIOCITOS 2X CAMPO, INCONTABLES
BACTERIAS, NO CILINDROS.
¿EN CUAL DE LAS SIGUIENTES CONDICIONES ESPECIFICAS ESTA
INDICADO DE MANERA ABSOLUTA EL TRATAMIENTO ANTIBIOTICO?
INMEDIATAMENTE ANTES DE UNA URETEROCISTOSCOPIA
ELECTIVA
69- PACIENTE DE 65 AÑOS QUE ACUDE A CONSULTA MEDICA DE PRIMER
NIVEL, SEXO MASCULINO, NO ANTECEDENTES DE DM, SE LE TOMA LA TA
SISTOLICA 160, UTILIZANDO LA SIGUIENTE TABLA DE ESTIMACION DE
RIESGO CARDIOVASCULAR (OMS/ISH). DETERMINE CUAL ES EL RCV QUE
PRESENTA ESTE PACIENTE:
RCV DE 10% A MENOR A 20%
70- A UNA PACIENTE JOVEN CON UNA SEMANA DE EVOLUCION DE FIEBRE,
MALESTAR GENERAL, ANOREXIA Y NAUSEA E ICTERICIA LE
DIAGNOSTICAN HEPATITIS VIRAL AGUDA, EN ESTE CONTESTO. ¿CUAL ES
EL ORDEN CRONOLOGICO DE LOS ACONTECIMIENTOS QUE
GENERALMENTE SUCEDEN EN ESTA INFECCION VIRICA?
VIREMIA, EXCRECIÓN FECAL-ORAL, ELEVACIÓN DE
AMINOTRANSFERASAS, ICTERICIA.

71- EN LA CONSULTA EXTERNA DE UN CENTRO DE SALUD TIPO C UN


PACIENTE MASCULINO DE 10 MESES DE EDAD ES TRAIDO POR SU MADRE
POR PRESENTAR UN CUADRO DE TOS PERRUNA DESDE HACE 24 HORAS.
AL EX. FISICO EL NIÑO SE ENCUENTRA AFEBRIL, NO PRESENTA CIANOSIS,
SE EVIDENCIA RETRACCIONES TORACICAS LEVES, LA TOS PERUNA ES
EVIDENTE, ASI COMO ESTRIDOR INSPIRATORIO EN REPOSO. ¿CUAL ES EL
TRATAMIENTO?
ADMINISTRAR CORTICOIDE ORAL MÁS NEBULIZACION CON
ADRENALINA
72-¿QUE MEDICAMENTO DE USO TOPICO (DE PRIMERA ELECCION) SE
EMPLEA EN LA TERAPIA DE MANTENIMIENTO DEL ACNE LEVE A
MODERADO?
ADAPALENO EN GEL 0.1%.

73- UNA PACIENTE DE 29 AÑOS SIN ANTECEDENTES, DESDE HACE 5 DIAS


PRESENTA FIEBRE, MALESTAR GENERAL DECAIMIENTO, TOS NO
PRODUCTIVA Y DOLOR EN PUNTA DE COSTADO DEL LADO DERECHO,
TIENE TA 120/80, FC 98, FR 22, TEMP 39, TIENE INCREMENTO DEL FRÉMITO
Y BRONCOFONIA EN REGION SUBESCAPULAR DERECHA, EL HEMITORAX
IZQUIERDO ES NORMAL, EN LA RADIOGRAFIA SE APRECIA IMAGEN
RADIOPACA CON BRONCOGRAMA AEREO LIMITADO AL LOBULO INFERIOR
DERECHO, OXIMETRIA 91% AL AIRE AMBIENTE, BIOMETRIA LEUCO 8500,
SEG 73, LINF 3. LA CONDUCTA MAS APROPIADA SERA:
REPOSO EN CASA ADMINISTRAR AMOXACILINA CLAVULANATO
VIA ORAL Y REEVALUAR A LAS 48 HORAS.

74- PACIENTE MASCULINO DE 25 AÑOS, CON ANTECEDENTE FAMILIARES


DE ACNE QUISTICO GRAVE, PRESENTA LESIONES QUE INICIARON CON
COMEDONES ABIERTOS Y CERRADOS QUE SE TRANSFORMARON EN
LESIONES PAPULO-PUSTULOSAS, CON AUMENTO DE TAMAÑO Y
DIFUSION, LO CUAL ES CARACTERISTICO DEL ACNE NODULOQUISTICO,
LAS LESIONES SON MUY DOLOROSAS Y PRESENTA UNA DISTRIBUCION
SIMETRIA EN LA CARA. ¿CUAL ES LA GRAVEDAD DEL ACNE Y SU
TRATAMIENTO?
ACNE GRAVE. MANEJAR CON TRATAMIENTO TOPICO Y TTO
SITEMICO CON ISOTRETIONINA
75- PACIENTE CON DM TIPO 2 QUE INICIA TTO FARMACOLOGICO CON
METFORMINA Y QUE DURANTE LOS PRIMEROS DIAS DE TTO PRESENTA
DIARREA, NAUSEA Y HA VOMITADO POR 2 OCASIONES, ESTAS
MANIFESTACIONES CLÍNICAS SE DEBEN A:
HIPERSENSIBILIDAD A LA METFORMINA
76- PACIENTE DE 19 AÑOS DESARROLLA CUADRO DE HEMATURIA Y
PROTEINURIA MICROSCOPICA ASOCIADO A HTA Y UREMIA, LA SOSPECHA
DIAGNOSTICA ES SINDROME NEFRITICO, TODOS LOS EXAMENES SON
UTILES PARA DIAGNOSTICO DE SINDROME NEFRITICO. EXCEPTO:
PROTEINAS EN SUERO RELACION ALBUMINA/GLOBULINA

77- UN PACIENTE JOVEN PREVIAMENTE SANO, PRESENTA CEFALEA Y


FIEBRE, DESDE HACE 48 HORAS. AL EX FISICO LO LLAMATIVO ES QUE
ESTA SOMNOLIENTO Y DESORIENTADO. LOS SIGNOS MENINGEOS SON
POSITIVOS, LA TAC ES NORMAL Y SE REALIZA UN PUNCION LUMBAR, EN
ESTE CONTEXTO. ¿CUAL DE LOS SIGUIENTES DATOS DEL REPORTE DEL
LCR NO SUGIERE QUE LA INFECCION ES BACTERIANA?
GLUCORRAQUIA >60% DE LA GLICEMIA
78- SE TRATA DE UN PACIENTE JOVEN CON ANTECEDENTE DE ASMA
BRONQUIAL, ACUDE PORQUE PRESENTA UNA NUEVA CRISIS COMO
PARTE DEL TTO USTED ADMINISTRA DOSIS REPETIDAS DE BETA
AGONISTAS INHALADOS DE ENTRE LAS SIGUIENTES ALTERACIONES.
¿CUALNO ES ATRIBUIBLE A ESTE FARMACO?
HIPONATREMIA
79- PACIENTE DE 9 AÑOS CON SÍNDROME DE LARON QUE HA SIDO
SELECCIONADO PARA ENTRAR EN UN ENSAYO CLÍNICO MULTICENTRICO
PARA PROBAR UN NUEVO IGF -1 RECOMBINANTE HUMANO, EL NIÑO Y SUS
PADRES FIRMAN EL CONSENTIMIENTO INFORMADO ¿ QUE PRINCIPIO
BIOETICO ESTA EJERCIENDO EL PACIENTE?
AUTONOMIA

80- PACIENTE MASCULINO DE 1 AÑO ES LLEVADO POR PRESENTAR


FIEBRE DE 38.6, TOS PRODUCTIVA DE LARGA EVOLUCIÓN LA CUAL SE HA
MANTENIDO INTERMITENTE LOS ULTIMOS 7 MESES, ACOMPAÑADO AL
MOMENTO DE DISNEA, TIRAJE INTERCOSTAL BAJO ASI MISMO LA MADRE
REFIERE EVACUACIONES DIARREICAS DE APROXIMADEMENTE 5 MESES
DE EVOLUCION, RX DE TORAX MUESTRA HIPERINFLACION PULMONAR,
ENGROSAMIENTO PERIBRONQUIAL Y ATELECTASIA SEGMENTARIA DE
LOBULO APICAL DERECHO ASI COMO AREAS DE CONDENSACIÓN , EN LOS
ANTECEDENTES PRODUCTO OBTENIDO DE PARTO EUTOCICO DE UN
EMBARAZO NORMO EVOLUTIVO, CON AUSENCIA DE EXPULSION DE
MECONIO, TRAS 48 HORAS DE OBSERVACION QUE REMITIO CON ENEMA
AL MOMENTO ESQUEMA DE VACUNACION COMPLETO, EL DATO
ORIENTADOR ES:
ILEO MECONIAL
81- PACIENTE FEMENINO DE 16 AÑOS QUE LLEGA AL SERVICIO DE
EMERGENCIA POR INTENTO DE SUICIDIO AL INGERIR TABLETAS DE
ASPIRINA SE LE REALIZA UNA GASOMETRIA INDICANDO PO2 36, HCO3
CONSIDERANDO CON EL COEFICIENTE DE SOLUBILIDAD DEL CO2 ES 0.03.
CUAL ES EL ESTADO ACIDO BASICO DEL PACIENTE
ACIDOSIS METABÓLICA COMPENSADA

82- PACIENTE DE 38 AÑOS SIN ANTECEDENTE, REFIERE HACE 3 DIAS


PRESENTA SENSACIÓN DISTERMICA, TOS CON EXPECTORACIÓN
BLANQUECINA Y MALESTAR GENERAL. EL PACIENTE NO HA UTILIZADO
ANTIBIÓTICOS, DURANTE EL ÚLTIMO AÑO. AL EX. FISICO SE ENCUENTRA
ALERTA, FC 90, FR 25, TA 120/80. NO CIANOSIS NI SIGNOS DE DIFICULTAD
RESPIRATORIA, EN BASE PULMONAR DERECHA SE AUSCULTAN
ESTERTORES BASALES, RX DE TORAX MUESTRA OPACIDAD NO
SEGMENTARIA EN EL LOBULO INFERIOR DERECHO CON PRESENCIA DE
BRONCOGRAMA AEREO.
¿CUAL ES EL TRATAMIENTO ANTIBIOTICO EMPIRICO INICIAL PARA ESTE
PACIENTE ?
DOXICICLINA 100MG CADA 12 HORAS VO POR 5 DIAS
83- LOS SIGUIENTES PARAMENTROS SEGUN LAS TABLAS DE
FRAMINGHAM, SE UTILIZAN PARA CALCULAR EL RIESGO
CARDIOVASCULAR. EXCEPTO:
COLESTEROL LDL
84- LOS SIGUIENTES SON FARMACOS QUE INDUCEN DIABETES.
EXCEPTO:
BROMOCRIPTINA
85- UN PACIENTE DE 5 AÑOS PRESENTA HACE 3 DÍAS FIEBRE, TOS SECA,
RINORREA Y CONGESTIÓN OCULAR, HOY LA FIEBRE SE INCREMENTA A
40°C Y SE ACOMPAÑA DE UN EXANTEMA MACULO-PAPULAR ROJIZO,
CONFLUENTE QUE INICIA EN LA LINEA DE IMPLANTACIÓN DEL CABELLO.
¿CUAL ES EL DIAGNOSTICO MAS PROBABLE?
SARAMPIÓN
86- PACIENTE DE 4 AÑOS, QUIEN ESTA CONSCIENTE, AFEBRIL, PRESENTA
ESTRIDOR INSPIRATORIO Y TOS SECA INDIQUE EL DIAGNOSTICO
CORRECTO:
CRUP MODERADO

87- MARIA ES UNA NIÑA QUE AL EX PARA EVALUAR SU DESARROLLO


PSICOMOTOR DEMUESTRA QUE PUEDE COMER GALLETAS SOLA,IMITA
PALABRAS Y DICE DE FORMA INESPECÍFICA PAPA, MAMA, PASA UN CUBO
DE UNA MANO A OTRA, ESTA SENTADA CON APOYO Y ES TIMIDA CON
EXTRAÑOS, ESCOJA LA OPCCION QUE CORRESPONDE A LA EDAD
APROXIMADA DE LA NIÑA
6 MESES
88- SE TRATA DE UN PACIENTE VARON DE 66 AÑOS , FUMADOR CRONICO
E HTA, ACUDE A EMERGENCIA PORQUE PRESENTA DE MANERA SUBITA
PERDIDA DE MOVIMIENTOS Y DE LA SENSIBILIDAD DE EXTREMIDAD
SUPERIOR E INFERIOR DERECHO, DESVIACION DE LA MIRADA HACIA EL
LADO IZQUIERDO, EX FISICO ES MAS PROMINENTE LA DEBILIDAD Y
ESPESTICIDAD EN EL BRAZO, ADEMAS SE ENCUENTRA AFASICO. ¿ CUAL
DE LAS SIGUIENTES ARTERIAS ESTA CON MAYOR PROBABILIDAD
INVOLUCRADA ?
CEREBRAL MEDIA
89- ¿ CUAL DE LAS SIGUIENTES CAUSAS DE HIPOGLICEMIA SE RELACIONA
EN EL PACIENTE NO ENFERMO?
HIPERINSULINISMO ENDÓGENO
90- UNA PACIENTE DE 66 AÑOS ACUDE POR DOLOR PLEURITICO, LUEGO
DE VERIFICAR LA EXISTENCIA DE DERRAME PLEURAL IZQUIERDO, SE
REALIZA ESTUDIO DEL LIQUIDO, EN SANGRE PROTEÍNAS TOTALES 6
GR/DL, LDH 200, EN EL LIQUIDO PROTEINAS 1.5 GR/DL, LDH 100, COMO
POSIBLES CAUSAS DE DERRAME PLEURAL DE ACUERDOA LOS
RESULTADOS OBTENIDOS. ¿ CUAL DE LOS SIGUIENTES DIAGNOSTICO ES
MENOS PROBABLE?
EMBOLISMO PULMONAR
91- UNA MUJER DE 30 AÑOS , ANTECEDENTE DE HTA Y QUE POR UN
PROCESO AUTOINMUNE REQUIERE DE TERAPEUTICA CRONICA CON
PREDNISONA 30 MG/DIA. ¿ QUE SUSTANCIA DEBE SER RESTRINGIDA EN
LA ALIMENTACION DE ESTA PACIENTE, DEBIDO A LA TERAPEUTICA
ESTEROIDE E HTA?
SODIO

92- NIÑO DE 8 AÑOS, SANO, CUADRO CLINICO DE FIEBRE DE 38.1, TOS


SECA, ESTERTORES CREPITANTES Y SIBILANCIAS EN BASE DERECHA,
SAT O2 92%, FR 22, BIOMETRIA LEUCO 12000, SEG 75, LINF 22, EOS 3, RX
DE TORAX IMAGEN CONDENSANTE EN BASE DERECHA. INDIQUE EL
MANEJO MEDICO MAS ADECUADO PARA EL PACIENTE:
AMBULATORIO, CLARITROMICINA. ( AGENTE MAS FRECUENTE EN
ESE GRUPO ETARIO EN LA COMUNIDAD MICOPLASMA ).

93- UN NIÑO DE 18 MESES DE VIDA HA SIDO HOSPITALIZADO EN 2


OCASIONES POR NEUMONIA, SE CONSIDERA QUE EL ORIGEN DE ESTOS
CUADROS ES UNA ENFERMEDAD POR REFLUJO GASTROESOFAGICO NO
TRATADA. ¿CUAL SERIA LA PRUEBA MAS INDICADA EN ESTE CASO PARA
CONFIRMAR LA SOSPECHA DIAGNOSTICA?
PH METRIA ESOFÁGICA
94- LA DETENCION OPORTUNA DEL CANCER COLORECTAL SE HACE CON
DIFERENTES METODOS, COMO COLONOSCOPIA, SANGRE OCULTA EN
HECES Y OTROS. ¿ A QUE EDAD SE DEBE INICIAR ESTA INVESTIGACION
EN PACIENTES SIN FACTORES DE RIESGO?
50 AÑOS
95- MATIAS ES UN NIÑO QUE EN LA EVALUACION DEL DESARROLLO
PSICOMOTOR EN EL AREA SOCIAL JUEGA A LA PELOTA, BEBE DE UN
VASO, IMITA TREAS DOMESTICAS , SE QUITA ALGUNAS PRENDAS, DICE 3
PALABRAS. ADEMAS DE MAMA Y PAPA, SEÑALA ALGUNAS PARTES DE SU
CUERPO , HACE GARABATOS Y ARMA UNA TORRE CON DOS CUBOS,
ESCOJA LA OPCCION QUE CORRESPONDA A LA EDAD DE MATIAS SEGUN
LOS RESULTADOS DE SU DENVER.
15 MESES
96- ¿ QUE MECANISMO FISIOLÓGICO EN EL EMBARAZO PRODUCE
INCREMENTO DE LA EXCRECIÓN URINARIA DE PROTEÍNA?
DISMINUCIÓN DE LA REABSORCION TUBULAR DE LAS
PROTEÍNAS FILTRADAS
97- UNA PACIENTE DE 58 AÑOS SIN ANTECEDENTES. ESTA ASINTOMÁTICA
PERO DURANTE LOS CONTROLES MEDICOS SUBSECUENTES USTED
VERIFICA QUE LAS CIFRAS DE PRESION ARTERIAL DE LA PACIENTE
ESTAN 140/98 Y DECIDE INICIAR TTO¿CUAL NO DEBE USAR?.
ALFABLOQUEANTES
98- UN PACIENTE VARON DE 55 AÑOS, PRESENTA TA 150/90, PERIMETRO
ABDOMINAL 90, GLICEMIA 150, TRIGLICERIDOS 179, HDL 25 DE ACUERDO A
LOS DATOS. ¿ CUAL DE LAS SIGUIENTES OPCIONES CORRESPONDE AL
DIAGNOSTICO DE SINDROME METABÓLICO?
PRESENTA 4 DE LOS 5 CRITERIOS DIAGNÓSTICOS , POR
TANTO TIENE SINDROME METABÓLICO
99- USTED ATIENDE UN PACIENTE CON CUADRO COMPATIBLE CON
COLECISTITIS AGUDA, AL EX FISICO FIEBRE Y MURPHY POSITIVO, EL
ULTRASONIDO DEMUESTRA UNA VESICULA GRANDE , DE PAREDES
TENSAS E INMOVILES, SIN CALCULOS EN SU INTERIOR, SU DIAGNOSTICO
ES COLECISTITIS ACALCULOSA. ¿ CUAL DE LAS SIGUIENTES
CONDICIONES ESTARIA ASOCIADA CON ESTA ENFERMEDAD?
TRAUMA Y QUEMADURA EXTENSA
100- UNA MUJER DE 42 AÑOS DESDE HACE 2 DIAS LUEGO DE INGERIR
ALIMENTOS , PRESENTA DOLOR MODERADO EN EPIGASTRIO, SE HA
INTENSIFICADO DE MANERA PAULATINA, SE IRRADIA A LA ESPALDA, SE
ACOMPAÑA DE VOMITO DE CONTENIDO ALIMENTICIO, EX FISICO AL
TOMAR LA PRESION SE PROVOCA CONTRACTURA ESPESTICA DE LA
MUSCULATURA EXTENSORA Y ADUCTORA DE LOS DEDOS DE LA MANO,
ELEVADA AMILASA 7 VECES Y LIPASA 3 VECES SU VALOR NORMAL,
ECOGRAFIA PANCREAS INCREMENTADO DE VOLUMEN E
HIPOECOCOGENICO. ¿ QUE HALLAZGO DE LABORATORIO PODRIA
EXPLICAR LA CONTRACTURA ESPASTICA DESCRITA?
HIPOCALCEMIA
238- UNA NIÑA DE 5 AÑOS PRESENTA DESDE HACE 2 DIAS FIEBRE,
DISURIA Y POLAQUIURIA, NO TIENE ANTECEDENTES, EX DE ORINA
REVELA NITRITOS Y ESTEREASA LEUCOCITARIA POSITIVOS. ¿ CUAL
SERIA LA CONDUCTA MAS ADECUADA?
SOLICITAR CULTIVO E INICIAR TTO EMPIRICO
241- UN RECIEN NACIDO PREMATURO PRESENTA AL NACIMIENTO ALETEO
NASAL, QUEJIDO, RETRACCIONES SUBCOSTALES E INTERCOSTALES, ¿
CUAL ESCALA CLÍNICA PERMITIRÁ ESTABLECER DE MEJOR MANERA LA
SEVERIDAD DE LOS SÍNTOMAS?
SCORE DE SILVERMAN
242- UNA MADRE PRIMIGESTA CON CUADRO DE MASTITIS Y POR ESA
RAZON SE ENCUENTRA TOMANDO AMOXACILINA, DESEA CONOCER LA
CONDUCTA A SEGUIR CON RESPECTO A LA LM, ¿CUAL DE LAS
SIGUIENTES OPCIONES SERIA LA MAS ADECUADA?
DEBE CONTINUAR ALIMENTANDO A SU HIJO CON SENO
MATERNO
244- UNA NIÑA DE 5 AÑOS , HACE 2 DIAS DOLOR ABDOMINAL, FIEBRE 39.5,
DEPOSICIONES BLANDAS CON SANGRE, CON UNA FRECUENCIA DE 5 POR
DIA ¿CUAL DE LOS SIGUIENTES GERMENES SE ASOCIA
FRECUENTEMENTE CON ESTE CUADRO?
SHIGELLA
245- PACIENTE DE 32 AÑOS , HACE 15 DIAS PARTO VAGINAL, TIENE
MALESTAR GENERAL , ALZA TERMICA NO CUANTIFICADA Y DOLOR EN
MAMA DERECHA HACE 3 DIAS, EL EMBARAZO Y PARTO NORMALES. AL
MONENTO EN LACTANCIA, GESTA 3, PARA 3. SIGNOS VIRALES TEMP 38.1,
MAMAS HIPERTROFICAS, CON RED DE HALLER EVIDENTE, CON FISURA EN
PEZON DERECHO, EN EL CUADRANTE SUPEROEXT PIEL ERITEMATOSA
COMO NARANJA SE PALPA MASA DURA, DOLOROSA IRREGULAR GANGLIO
DE 2 CM EN AXILA DERECHA. ¿CUAL SERIA EL TTO A SEGUIR?
DICLOXACILNA 500MG VO CADA 6 HORAS POR 7 DÍAS
246- PACIENTE ADULTO CON FIEBRE, DOLOR POR PRIMERA VEZ,
SENSIBILIDAD DOLOROSA A LA PALPACION Y DEFENSA MUSCULAR EN EL
CUADRANTE SUPERIOR DERECHO, EXAMENES LEUCOCITOSIS Y LIGERAS
ELEVACIONES DE FA, BILIRRUBINAS Y TRANSAMINASA, SEÑALE EL
DIAGNOSTICO:
COLECISTITIS AGUDA
251- SEÑALE EL PATÓGENO QUE CAUSA NEUMONIA EXTRAHOSPITALARIA
(ADQUIRIDA EN LA COMUNIDAD) EN PACIENTES CON EPOC Y
TABAQUISMO:
HAEMOPHILUS INFLUENZAE
253- EN LOS PACIENTES CON DM TIPO 2 , SE RECOMIENDA LA SIGUIENTE
DISTRIBUCIÓN DE LOS TIPOS DE GRASA DIETARIA CONSIDERANDO EL
TOTAL DE CALORÍAS DIARIAS, EXCEPTO:
ÁCIDOS GRASOS SATURADOS 30-45%
254- PACIENTE DE 7 AÑOS PROCEDENTE DE MACHALA, DESDE HACE 10
DIAS PRESENTA FIEBRE CADA 72 HORAS, CUANTIFICADA EN 40°C, SE
ACOMPAÑA DE SUDORACIÓN , CEFALEA, DIARREA Y PALIDEZ CUTÁNEO
MUCOSA, ESTOS EPISODIOS ALTERNAN CON PERIODOS DE BUEN
ESTADO GENERAL, INDIQUE EL DIAGNOSTICO MAS PROBABLE:
MALARIA
255- SE TRATA DE UN PACIENTE DE 38 AÑOS CON ENFERMEDAD ÁCIDO
PÉPTICA SINTOMÁTICA RECURRENTE SIN SANGRADO DIGESTIVO, TIENE
BIOPSIA POSITIVA PARA H. PYLORI,DE ENTRE LAS SIGUIENTES
ALTERNATIVAS, ¿CUAL ES LA MAS ADECUADA?
OMEPRAZOL, SUBSALICILATO DE BISMUTO, METRONIDAZOL Y
TETRACICLINA

256- UN PACIENTE DE 21 AÑOS DE EDAD CON RECIENTE DIAGNOSTICO DE


DM TIPO 1, ACUDE A CONTROL Y SOLICITA INFORMACIÓN RESPECTO A
LOS SÍNTOMAS DE HIPOGLICEMIA, DE ENTRE LOS SIGUIENTES SÍNTOMAS
O SIGNOS , ¿ CUAL NO ES HABITUAL EN HIPOGLICEMIA?
BRADICARDIA
257- UN RECIÉN NACIDO A TERMINO DE 18 HORAS , PRESENTA ICTERICIA
EN ZONA I Y HEPATOESPLENOMEGALIA, LOS EX HTO 40, BT 7, BD0.5,
RETICULOCITOS 10, MICROESFEROCITOS EN EL FROTIS DE SANGRE
PERIFÉRICA, ¿CUAL SERIA LA CONDUCTA MAS ADECUADA EN ESTE
CASO?
INDICAR FOTOTERAPIA
258- MARÍA ES UNA ESTUDIANTE UNIVERSITARIA DE 2 CICLO QUE DESDE
HACE 4 MESES PRESENTA TRASTORNOS ALIMENTICIOS Y HACE 2 AÑOS
PRESENTO UN CUADRO SEMEJANTE , SELECCIONE LOS CRITERIOS
DIAGNÓSTICOS PARA LA BULIMIA:
1- INGESTIÓN DE COMIDAS PEQUEÑAS CANTIDADES TODO EL DIA
2- INGESTIÓN DE COMIDAS EN GRANDES CANTIDADES EN CORTO TIEMPO
3- INGESTIÓN DE COMIDAS DE PREFERENCIA RESTRINGIDA O GUARDADA
4- INDUCCIÓN DE VÓMITO POSTPRANDIAL TARDÍO
5- LA RELACIÓN DE BULIMIA ENTRE HOMBRES Y MUJERES ES DE 1 A 10
6- LAS ADOLESCENTES TEMPRANAS PRESENTAN BULIMIA EN EL 10%
CORRECTAS:A)2,3,5

260- ¿CUAL DE LAS SIGUIENTES CONDICIONES SE ASOCIA CON MENOR


INCIDENCIA DE ENFERMEDAD POR REFLUJO GASTRO ESOFÁGICO?
GASTRITIS CRÓNICA ATRÓFICA
262- PACIENTE DE 38 AÑOS , SIN APP, HACE 6 DIAS PRESENTA
ESTORNUDOS Y RINORREA, INICIALMENTE ACUOSA, BILATERAL, HACE 24
HORAS SE TORNA AMARILLA, SE ACOMPAÑA DE HIPOSMIA, GOTEO
NASOFARINGEO, TOS IRRITATIVA Y MODERADO DOLOR FACIAL, EX FÍSICO
TEMP 37.8, SEGUN LOS LINEAMIENTOS GENERALES RECOMENDADOS , A
MAS DE UN ANALGÉSICO ADECUADO. ¿ CUAL SERIA LA CONDUCTA A
SEGUIR?
MANTENER UNA CONDUCTA EXPECTANTE CON RESPECTO A
LOS ANTIBIÓTICOS

264- SELECCIONA LOS MICROORGANISMOS RELACIONADOS CON


DIARREAS AGUDAS INFLAMATORIAS:
2,3,5
1- E.COLI ENTEROTOXIGÉNICA
2- E.COLI ENTEROINVASIVA
3- SHIGELLA SP
4- CLOSTRIDIUM PERFRINGENS
5- CAMPYLOBACTER SP

266- UN CAMPESINO AGRICULTOR DE 48 AÑOS , COMO APP TUVO


TÉTANOS A LOS 16 AÑOS, CUMPLIENDO SUS ACTIVIDADES HABITUALES
HOY SUFRE UNA HERIDA PEQUEÑA PERO PROFUNDA EN SU PIE, A MAS
DE LAS MEDIDAS DE HIGIENE ¿ CUAL ES LA CONDUCTA APROPIADA EN
CUANTO A LA VACUNACIÓN CONTRA EL TÉTANOS?
TRES DOSIS, UNA AL MOMENTO Y 2 REFUERZOS
268- MUJER DE 27 AÑOS PRESENTA DESDE HACE 11 DIAS , RINORREA
PURULENTA ANTERIOR Y POSTERIOR, OBSTRUCCIÓN NASAL BILATERAL
,DOLOR FACIAL, HIPOSMIA Y FIEBRE ¿CUAL ES LA MEJOR CONDUCTA
TERAPÉUTICA?
COTRIMOXAZOL
269- PACIENTE DE 12 MESES DE EDAD PRESENTÓ HACE 2 DIAS RINORREA
Y TOS PERRUNA, HOY PRESENTA CIANOSIS Y DISFONIA, AL EX SE
ENCUENTRA ESTRIDOR EN REPOSO Y RETRACCIONES SUBCOSTALES ¿
CUAL ES EL DIAGNOSTICO EN ESTE CASO?
LARINGITIS
270- SELECCIONA CAUSA PRE-RENAL DE INSUFICIENCIA RENAL AGUDA:
DESHIDRATACIÓN AGUDA
271- UN PACIENTE CON APP DE REGURGITACION Y PIROSIS QUE
ACTUALMENTE SE QUEJA DE DISFAGIA Y DOLOR TORÁCICO
RETROESTERNAL ACUDE A SU CONSULTA, EX FÍSICO PESO 82 KG, TALLA
1.70, TA 110/70, FC 72, FR 16, TEMP 36.5, RESTO NORMAL,¿CUAL DE LOS
SIGUIENTES FÁRMACOS ES DE PRIMERA ELECCIÓN PARA ESTE TIPO DE
ALTERACIÓN?
OMEPRAZOL
273- A CONSULTA ACUDE PACIENTE DE 15 MESES MASCULINO CON SU
MADRE PARA VACUNACIÓN ,EL PACIENTE HA CUMPLIDO EL ESQUEMA.
¿CUAL VACUNA ES LA SIGUIENTE A ADMINISTRARSE SEGÚN EL MSP DEL
ECUADOR?
VACUNA CONTRA LA VARICELA
274- EL MSP REALIZA ANUALMENTE UNA CAMPAÑA DE VACUNACIÓN PARA
INFLUENZA ¿A PARTIR DE QUE EDAD ESTA INDICADA LA VACUNA DE
GRIPE?
6MESES HASTA LOS 4 AÑOS OBLIGATORIO
275- CON RESPECTO AL USO DEL DISPOSITIVO INTRAUTERINO T DE
COBRE COMO MÉTODO DE PLANIFICACIÓN FAMILIAR ,SEÑALE CUAL ES
SU CONTRAINDICACIÓN DE SU USO
METRORRAGIA DE ORIGEN DESCONOCIDO
276- PACIENTE DE 21 AÑOS , SOLTERA ACUDE POR PRESENTAR
SECRECIÓN DE MAL OLOR DESDE HACE 24 HORAS, ASOCIADA A LEVE
SINTOMATOLOGÍA URINARIA, FUM 15 DIAS ANTES, RELACIÓN SEXUAL
HACE 1 SEMANA CON PAREJA OCASIONAL, EX ESPECULAR VULVA CON
ESCURRIMIENTO POSTERIOR, EN MODERADA CANTIDAD, VAGINA SE
CONSTATA LEUCORREA AMARILLO-VERDOSA , DE LIGERO MAL OLOR CON
POCAS BURBUJAS, CERVIX POSTERIOR ERITEMATOSO Y PUNTEADO
ROJO, TACTO VAGINAL NORMAL, EX FRESCO Y GRAM DE SECRECIÓN
VAGINAL TEST DE KOH POSITIVO LEVEMENTE, POLIMORFOS
AUMENTADOS Y BACTERIAS. ¿CUAL ES EL MEJOR TTO PARA ESTA
PACIENTE?
MERONIDAZOL 500MG VO BID POR 7 DÍAS
278- PACIENTE CON UNA FUNCIÓN RENAL NORMAL ¿CUAL ES EL
INCREMENTO MÍNIMO DE LA CREATININA SÉRICA DENTRO DE LAS
PRIMERAS 48 HORAS PARA CONSIDERAR INSUFICIENCIA RENAL AGUDA?
0.4 MG/DL
279- EN UN PACIENTE ASINTOMÁTICO SE AUSCULTA UN
DESDOBLAMIENTO AMPLIO DEL SEGUNDO RUIDO CARDÍACO SOBRE LA
LINEA PARAESTERNAL IZQUIERDA A NIVEL DEL SEGUNDO ESPACIO
INTERCOSTAL ,EN ESTAS CONDICIONES ¿CUAL DE LOS SIGUIENTES
BLOQUEOS PUEDE EXPLICAR EL HALLAZGO ENCONTRADO?
BLOQUEO COMPLETO DE RAMA DERECHA
287- PACIENTE VARÓN DE 56 AÑOS CON APP DE HERNIA INGUINAL,
CONSULTA POR ARDOR Y MASA QUE HA CRECIDO EN LOS ÚLTIMOS DÍAS
EN ESA ZONA ,EX FÍSICO GENERAL NORMAL EN REGIÓN INGUINAL
DERECHA, BAJO EL LIGAMENTO INGUINAL, SE PALPA MASA DE 6-7 CM
DOLOROSA, QUE NO SE PUEDE REDUCIR Y QUE A LA MANIOBRA DE
VALSALVA EMPUJA EL DEDO EXPLORADOR ¿ QUE TIPO DE HERNIA ES LA
MAS PROBABLE?
HERNIA FEMORAL

292- EN UNA EVALUACIÓN NUTRICIONAL SE REPORTA QUE ROSARIO DE


20 AÑOS TIENE UNA INGESTA DIARIA DE 50 GR DE PROTEÍNA, 100 GR DE
GRASA Y 400 GR DE HIDRATOS DE CARBONO,¿CUAL SERÍA LA INGESTA
CALÓRICA TOTAL?
2700 CALORÍA/DÍA.
La energía que libera un gramo de hidrato de carbono tras su oxidación a dióxido
de carbono y agua es de 4 calorías, mientras que cada gramo de grasa aporta 9
calorías y cada gramo de proteína equivale a 4 calorías. Por lo tanto el cálculo de
los ingresos calóricos de la paciente serían: hidratos de carbono 400 x 4= 1600;
grasas 100 x 9= 900 y proteínas 50 x 4= 200 es decir 2700 calorías por día.

293- UN NIÑO DE 4 AÑOS CON 16 KG PRESENTA HACE 4 DÍAS UN


PROCESO RESPIRATORIO DE TOS PRODUCTIVA, CON ESPUTO CAFÉ
AMARILLENTO Y FIEBRE, EN LAS ÚLTIMAS 24 HORAS SE ENCUENTRA
ASTÉNICO, RESPIRACIÓN AGITADA Y ANOREXIA, TOMA MUY POCOS
LÍQUIDOS ,T 39,2, FC: 116 ; FR: 36 ; SUS LABIOS ESTÁN SECOS Y RESPIRA
CON LA BOCA ENTREABIERTA, SE OBSERVA TIRAJE INTERCOSTAL Y
SUBCOSTAL ,A LA AUSCULTACIÓN DISMINUCIÓN EN BASE DERECHA, BH
:LEUC 18 000; POLI: 78%; LINF 17%; EOS: 2%: MONO: 3%. RX
CONSOLIDACIÓN DEL PARÉNQUIMA PULMONAR EN BASE DERECHA,SE
DIAGNOSTICA NEUMONÍA , SELECCIONAR EL GERMEN PRODUCTOR DE LA
NEUMONÍA Y EL ANTIMICROBIANO MÁS ADECUADO PARA SU TTO :
STREPTOCOCUS PNEUMONIAE, EL TTO ES PENICILINA IV.
295- ELIJA LA REACCIÓN ADVERSA ASOCIADA A LA VACUNACIÓN POR
SARAMPIÓN,RUBEOLA Y PAPERAS(SRP):
ERUPCIÓN CUTÁNEA
296- UNA NIÑA DE 7 AÑOS , REFIERE EN EMERGENCIA CEFALEA Y
HEMATURIA, HACE 3 SEMANAS TUVO UNA PIODERMITIS TRATADA CON
ANTIBIOTICOTEREPIA DE FORMA SATISFACTORIA, EX FISICO TA 150/105,
FC 110, FR 26, Y EDEMA PRETIBIAL, EMO HEMATÍES CAMPO
LLENO,CILINDROS HEMÁTICOS, LEUCOCITOS 3-4 X CAMPO Y
PROTEINURIA, EL C3 SE ENCUENTRA BAJO ¿CUAL DE LAS SIGUIENTES
OPCIONES SERIA EL DIAGNOSTICO MAS PROBABLE?
GLOMERULONEFRITIS
297- ADOLESCENTE FEMENINA DE 14 AÑOS , REFIERE HACE 3 DÍAS
DOLOR EN SU OÍDO DERECHO, QUE HA IDO AUMENTANDO
PROGRESIVAMENTE DE INTENSIDAD, FIEBRE 40°C, SU MEDICO HACE EX
OTOSCOPICO Y ENCUENTRA MODERADA SUPURACIÓN DE COLOR
AMARILLENTO, EL MEDICO PRESCRIBE PARACETAMOL 500MG Y
AMOXACILINA 500MG CADA 8 HORAS POR 3 DÍAS , PERSISTEN LOS
SÍNTOMAS A LAS 48 HORAS,SUGIERE CAMBIO DE ESQUEMA ANTIBIOTICO.
¿CUAL SERIA EL ANTIBIÓTICO DE ELECCIÓN?
CEFUROXIMA
298- UNA NIÑA DE 5 AÑOS PRESNTA HACE 2 DIAS FIEBRE, DISURIA Y
POLAQUIURIA, NO TIENE APP,EL EMO REVELA NITIRTOS Y ESTEREASA
LEUCOCITARIA POSITIVOS ¿CUAL SERIA LA CONDUCTA MAS ADECUADA?
SOLICITAR UROCULTIVO E INICIAR TRATAMIENTO EMPÍRICO
299- ANA MUJER DE 18 AÑOS ,CON EPILEPSIA TIENE UN CUADRO DE ACNÉ
MODERADO A SEVERO QUE AFECTA CARA,TORSO Y ESPALDA ¿CUAL DE
LOS SIGUIENTES ANTICONVULSIVANTES CON MAYOR PROBABILIDAD
PODRÍA AGRAVAR SU PADECIMIENTO?
DIFENILHIDANTOINA
305-SELECCIONE TODOS LOS PARÁMETROS QUE EVALUA EL ÍNDICE DE
BISHOP:
1, 4, 6, 7, 8
1. Dilatación cervical.
2. Variabilidad de la frecuencia cardíaca fetal.
3. Altura uterina
4. Altura de la presentación fetal.
5. Intensidad de las contracciones uterinas.
6. Consistencia cervical.
7. Posición de la presentación fetal.
8. Borramiento cervical.

307- UNA PTE DE 29 AÑIS SIN APP, HACE 5 DIAS FIEBRE, MALESTAR
GENERAL, DECAIMIENTO, TOS NO PRODUCTIVA Y DOLOR EN PUNTA DE
COSTADO DERECHO, EX FISICO TA 120/80, FC 98, FR 22, TEMP 39, SAT 91%
,TIENE INCREMENTO DEL FRÉMITO Y BRONCOFONIA EN REGION
SUBESCAPULAR DERECHA , EL EREMITORIO IZQUIERDO RUIDOS
CARDÍACOS Y ABDOMEN NORMALES, RX IMAGEN RADIO OPACA , BH
LEUCO 8500, SEG 70, LINF 30 , LA CONDUCTA MAS APROPIADA SERA:
REPOSO EN CASA,ADMINISTRAR AMOXACILNA CLAVULANATO
VIA ORAL Y REEVALUAR A LAS 48 HORAS
308.- DURANTE EL CONTROL PRENATAL ESPECIFICO EN MUJERES CON
DM UNO DE LOS EXÁMENES A SOLICITARSE CON SU DEBIDA FRECUENCIA
ES LA ECOGRAFIA, DE LA TABLA EXPUESTA A CONTINUACIÓN ,RELACIONE
EL PERIODO DEL EMBARAZO CON EL OBJETIVO PARA EL CUAL SE
SOLICITA LA ECOGRAFIA SEMANA GESTACIONAL OBJETIVO DE LA
ECOGRAFIA:
1c, 2d, 3a, 4b.
1. SEMANAS 11 A 14.(c. Realizar ecografía para determinar marcadores
ecográficos de AC)
2. SEMANAS 20 A 22.(d. Realizar una ecografía morfológica fetal (detalle
anatómico)
3. SEMANAS 28 A 30.(a. Realizar ecografía para valorar crecimiento fetal,
perímetro abdominal fetal y determinación de percentil para descartar macrosomía
fetal incipiente.)
4. SEMANA 36 .(b. Realizar ecografía para valorar crecimiento fetal y el volumen
del líquido amniótico.)

309- UNA NIÑA DE 3 AÑOS , PESA 15 KG, HACE 12 HORAS PRESENTA


ERUPCIÓN DE APARICIÓN ABDOMINAL,CON VESÍCULAS Y PAPULAS
ERITEMATOSA, CON INTENSO PRURITO, QUE SE HAN EXTENDIDO HACIA
EL TORAX, NO HA TENIDO FIEBRE , SE MANTIENE ACTIVA Y COME BIEN,
TEMP 37.5, FC 70, FR 24, AL EX FISICO MÁCULAS, PÁPULAS Y VESÍCULAS
SE HAN EXTENDIDO AL TRONCO SUPERIOR Y POSTERIOR . SELECCIONE
EL DIAGNÓSTICO Y TRATAMIENTO CORRECTOS:
VARICELA, ADMINISTRAR ACICLOVIR 200MG/5ML,7.5ML CADA 6
HORAS DURANTE 5 DÍAS
310- EN RELACIÓN A NEUROCISTICERCOSIS, LAS SIGUIENTES
AFIRMACIONES SON CORRECTAS, EXCEPTO:
LA NEUROCISTICERCOSIS SE TRATA CON ANTIPARASITARIOS
COMO METRONIDAZOL O TINDAZOL SUMADO A AINES PARA
MANEJO DEL DOLOR.
311- UN ESCOLAR DE 10 AÑOS , HACE 3 DIAS DIARREAS LIQUIDAS,
ABUNDANTES, FÉTIDAS, CON MOCO Y UN POCO DE SANGRE, CON
FRECUENCIA DE 3 A 4 VECES AL DIA,CON DOLOR ABDOMINAL DE TIPO
CÓLICO, DE MODERADA INTENSIDAD,SE REALIZA COPROPARASITARIO
QUE REPORTA BALATIDIUM COLI. SELECCIONE EL FÁRMACO CORRECTO
PARA TRATAR ESTA INFESTACIÓN:
METRONIDAZOL
312- UNA MUJER DE 52 AÑOS ,CON DM EN TTO ORAL DURANTE VARIOS
AÑOS, HACE 10 DÍAS PRESENTA RINORREA BILATERAL,AL INICIO FLUIDA Y
DESPUÉS AMARILLENTA ,DESDE HACE 3 DIAS FIEBRE Y CEFALEA, ACUDE
POR DOLOR PERIOCULAR IZQUIERDO, AL EXAMEN TEMP 39 ,FC 110, ZONA
PERIORBITARIA IZQUIERDA CON EDEMA , ERITEMA Y DOLOR, LA
CONDUCTA MAS APROPIADA SERA ADMINISTRAR TRATAMIENTO :
SINTOMÁTICO, INICIAR TTO ANTIBIÓTICO INTRAVENOSOS Y
HOSPITALIZAR
313- PACIENTE DE 20 AÑOS ACUDE POR SECRECIÓN VAGINAL VERDOSA
DE LIGERO MAL OLOR DE 3 DIAS, TENIENDO COMO CAUSA APARENTE
RELACIÓN SEXUAL HACE 9 DÍAS, SE ASOCIA A SINTOMATOLOGÍA
URINARIA, LA SECRECIÓN DESCRITA SE OBSERVA EN ORIFICIO EXT DEL
CUELLO UTERINO, EX DE FRESCO Y GRAM PRESENCIA DE DIPLOCOCOS
GRAM NEGATIVOS INTRA Y EXTRACELULARES. ¿CUAL DE LOS
SIGUIENTES ESQUEMAS TTO ES DE PRIMERA ELECCIÓN?
CEFTRIAXONA MÁS AZITROMICINA.
314- PACIENTE DE 25 AÑOS CON CICLOS MENSTRUALES
REGULARES,(INTERVALO 30 Y DURACIÓN 5) DESDE SU MENARQUIA A LOS
13 AÑOS, CONSULTA POR AMENORREA DE 4 MESES DE DURACIÓN, SIN
OTRA SINTOMATOLOGÍA, EX FISICO Y GENITAL NORMAL. EN EL ESTUDIO
DE LA AMENORREA SECUNDARIA, ¿CUAL DE LA SIGUIENTES PRUEBAS
NO DEBE INCLUIR USTED EN EL INICIO DEL PROCESO DIAGNÓSTICO?
GONADOTROPINAS HIPOFISIARIAS(LH-FSH).
315- UN NIÑO DE 12 AÑOS , SIN APP HA SIDO DIAGNOSTICADO DE NAC ,
USTED DEBE ESCOGER UN ANTIBIÓTICO QUE CUBRA LOS AGENTES
ETIOLÓGICOS COMUNES A ESTA EDAD ,DE LOS SIGUIENTES GÉRMENES,
¿CUAL ES FRECUENTE EN ESTE GRUPO ETARIO?
MYCOPLASMA

323- William tiene 44 años, está desempleado, divorciado, consume bebidas


alcohólicas todos los fines de semana, se siente triste, duerme mucho, ha bajado
de peso, no puede concentrarse, siente que piensa lento, por dos ocasiones se ha
cortado las venas y ha decidido lanzarse de un puente. Escoja el plan de
tratamiento a seguir en el manejo clínico de William.
Hospitalización en el servicio de psiquiatría e inicio de
psicofármacos.
324- El medicamento de primera línea para realizar el tratamiento de tiña corporis
es:
Terbinafina tópica.
328- De acuerdo a la valoración con ESCALA DE SARNAT (ENCEFALOPATIA
HIPOXICA ISQUÉMICA) cual es la conducta y estadio correcto:
a) I,1, A - II,2, B – III,3,C
I.EHI leve -1 Hipervigilia, Respuesta exagerada a los estímulos- A Succion Débil
II.EHI moderada-2 Letargo Estupor,Respuesta tardía e incompleta a los estímulos
sensoriales -B Succion Débil o ausente
III.EHI GRAVE 3-Coma, Respuesta sólo a estímulos fuertes -C Succion Ausente
A c) I,3, C - II,1, A - III,2, B
B d) I,1, B - II,3, B - III,3, A
C b) I,2, B - II,3, C - III,1,A
D a) I,1, A - II,2, B - III,3,
C

ESTADÍO 1: EHI LEVE, NIVEL DE CONCIENCIA EN HIPERVIGILIA,


RESPUESTA EXAGERADA A LOS ESTÍMULOS, SUCCION DEBIL.
ESTADÍO 2: EHI MODERADA, NIVEL DE CONCIENCIA EN LETARGO
ESTUPOR, RESPUESTA TARDÍA E INCOMPLETA A LOS ESTÍMULOS
SENSORIALES, SUCCION DÉBIL O AUSENTE.
ESTADÍO 3: EHI GRAVE, NIVEL DE CONCIENCIA EN COMA, RESPUESTA
SÓLO A ESTÍMULOS FUERTES, ESTADO DE IRRITABILIDAD E
HIPERALERTA, SUCCION AUSENTE

329- Una niña de 5 años de edad, presenta desde hace 2 días dolor abdominal,
fiebre de 39,5 °C y deposiciones blandas con sangre con una frecuencia de 5 por
día. ¿Adicionalmente a la hidratación oral cuál es la mejor alternativa terapéutica?
Cotrimoxazol

330- En las curvas de crecimiento aceptadas en el Ecuador, para un varón de 7


años de edad, los percentiles 10, 50 y 95, para su antropometría, son los
siguientes: Peso: 16, 22 y 27 kg; Talla: 108,119 y 128 cm; IMC: 13.7; 15 y 18 kg;
Un niño de 7 años de edad presenta los siguientes parámetros antropométricos:
Peso: 30 kg; Talla: 118 cm; IMC: 21.5 kg/m2. De acuerdo con estos datos,
seleccione el diagnóstico nutricional:
Obesidad grado I.(El IMC por encima del percentil 95 para su edad, establece
sin duda el diagnóstico de obesidad, en un niño que tiene el peso sobre el
percentil 95 para su edad.)

334- Un niño de 5 años de edad presenta súbitamente fiebre alta, cefalea, vómito
y convulsiones tónico - clónico generalizadas. En el examen físico se encuentra
rigidez de nuca. El examen de líquido cefalorraquídeo revela hipoglucorraquia,
proteinorraquia y pleocitosis con predominio de polimorfonucleares, ¿Cuál sería la
mejor alternativa terapéutica en este caso?
Cefotaxima + Vancomicina.
335- Un varón de 21 años de edad, sin antecedentes de importancia acude a su
consulta por presentar desde hace 3 días anorexia, decaimiento, fiebre, malestar
general, náusea y vómito. En el examen físico se aprecia ictericia escleral, dolor
en hipocondrio derecho. Los resultados de las enzimas hepáticas son: Alanino
transferasa (ALAT/TGP) 1000 unidades internacionales y aspartato transferasa
(ASAT/TGO) 850 unidades internacionales. ¿Cuál es la mejor conducta a seguir?
Solicitar inmunoglobulinas M para hepatitis.
336- Una mujer de 60 años de edad. Tiene antecedentes de hematoquecia por
diverticulosis colónica. El día de hoy se repite un episodio de sangrado indoloro,
los signos vitales son estables. En este contexto, ¿Cuál de las siguientes medidas
NO está indicada?
Antibióticos por vía intravenosa.
339- Señale que examen determina una extensión extracapsular (local) de un
cáncer de próstata:
Ecografía transrectal.
340- Paciente de 7 años diagnosticado con faringoamigdalitis aguda, el médico
prescribe un antibiótico intramuscular para su infección. La madre solicita un
antibiótico por vía oral para su hijo, pero el médico desecha esa posibilidad,
argumentando que es mejor para el niño recibir el tratamiento en un solo pinchazo.
Escoja el modelo de relación médico-paciente correspondiente a este caso clínico.
Paternalista.
342- En que etapa de la sífilis se observa la perforación del paladar blando?
Sífilis prenatal (congénita)

343- Relacione los tipos de demencias con sus características:


1ad, 2bc
Tipos de Demencias Caracteristicas
1. Demencias Corticales a.Amnesia, afasia, apraxia, agnosia
2. Demencias b.Apatía, trastornos ejecutivos
Subcorticales
c.Daño en Sustancia Blanca profunda
d.Déficit en la neurotransmisión
colinérgica

345- Un varón de 65 años de edad, utiliza regularmente anti inflamatorios no


esteroidales por dolores articulares inespecíficos, además tiene antecedentes de
estreñimiento crónico. Presenta hematoquecia indolora en dos ocasiones
seguidas, en un periodo de 14 horas, la última fue hace una hora. Al momento del
examen tiene presión arterial: 140/90 mmHg, frecuencia cardiaca: 82 lpm (latidos
por minuto), frecuencia respiratoria: 18 rpm (respiraciones por minuto), algo pálido.
Exploración cardiopulmonar normal. Abdomen suave, depresible, sin
visceromegalias, ruidos hidroaéreos presentes, al tacto rectal no se palpan
hemorroides. Hematocrito 39%, hemoglobina 13 g/dL. Química sanguínea básica,
plaquetas y tiempos de coagulación normales. ¿Cuál es la conducta más
conveniente a seguir en este caso?
Realizar colonoscopia.
346- Paciente de 29 años sin antecedentes de interés, refiere que desde hace 3
días presenta dolor en zona vulvar y vaginal. No síntomas miccionales. En el
examen físico se objetivan en pubis y vagina lesiones vesiculares múltiples de 1 a
2 mm de diámetro de bordes eritematosos y adenopatías inguinales bilaterales
dolorosas a la palpación. No se observa secreción vaginal ni uretral ¿Cuál es el
tratamiento indicado?.
Aciclovir 400 mg vía oral tres veces por 10 a 14 días.
347- Un adulto de 72 años de edad fue diagnosticado de infección complicada de
las vías aéreas, estuvo hospitalizado 3 días y recibió tratamiento con
bloqueadores de la bomba de protones y una cefalosporina de segunda
generación durante 10 días consecutivos, con evolución satisfactoria. Hace 48
horas presenta fiebre 38,5oC, 6 deposiciones diarreicas al día y dolor abdominal
tipo retortijón. En el coproparasitario tiene 90% de polimorfonucleares, sin
parásitos. En los exámenes de sangre tiene leucocitos de 15.000 /mm3 con 80%
de neutrófilos, ¿Cuál es la prueba con mayor sensibilidad para confirmar el
germen que se sospecha?
Cultivo de heces para identificar al microorganismo.

349- Se trata de una paciente de 26 años, previamente sana y con vida sexual
activa. Refiere que desde hace doce horas siente escozor intenso en el ojo
izquierdo, que a momentos es ardor, y que se acompaña de enrojecimiento ocular.
Al examen físico lo relevante en el ojo afectado es el edema inflamatorio palpebral,
quemosis conjuntival, la presencia de abundante secreción espesa, amarillo
verdosa, que persiste, ¿Cuál de los siguientes gérmenes está implicado con
mayor probabilidad, en la etiología del presente caso?
Neisseria.

350 -Una niña de 5 años de edad presenta desde hace 2 días dolor abdominal,
fiebre de 39,5 °C y deposiciones blandas con sangre, con una frecuencia de 5 por
día, ¿Cuál de los siguientes gérmenes se asocia frecuentemente con este cuadro?
Shigella.
351- A su consulta es llevado un paciente masculino de 6 meses de edad por sus
padres indicando que no sienten el testículo del lado derecho del niño. Ellos
indican que hace 3 meses acudieron a un médico quien les indico que el testículo
descendería posteriormente. El examen físico revela que el testículo derecho se
encuentra en el canal inguinal. Señale cual es el manejo en este tipo de
paciente.
Referirlo a urología pediátrica.
352- Seleccione el enunciado correcto para un paciente pediátrico con fiebre:
La evaluación inicial se centra en la búsqueda de una causa
infecciosa.
353- Los siguientes son factores de riesgo, en mayor o menor grado, para
desarrollar enfermedad pulmonar obstructiva crónica (EPOC). EXCEPTO:
Neumonía o bronquitis.
355- Un paciente de 60 años, sin antecedentes de importancia. Durante el último
examen de control anual tuvo presencia de hematuria en la tira reactiva “dipstick”,
que fue confirmada con la presencia de 3 a 5 hematíes dismórficos, por campo de
alto poder. Se acompaña de proteinuria de más de 500 mg/dL y de cilindros con
eritrocitos. Con mayor probabilidad, ¿Cuál es la estructura afectada?
Glomérulo.
356- Niño de 8 años sin antecedentes patológicos, acude a emergencia con el
siguiente cuadro clínico: fiebre 38,1 grados centígrados, tos seca, estertores
crepitantes y sibilancias en base pulmonar derecha, saturación de oxígeno de 92%
con FiO2 ambiente, frecuencia respiratoria 22 por minuto. La biometría hemática
reporta: leucocitos 12.000 (segmentados 75%, linfocitos 22%, eosinófilos 3%).
Radiografía de tórax: imagen condensante en base pulmonar derecha. Indique el
manejo médico más adecuado para el paciente:
Ambulatorio. Claritromicina
357- Paciente de 9 meses de edad, con infecciones respiratorias agudas bajas a
repetición, ¿Cuál de las siguientes condiciones debe descartarse en el paciente?
Reflujo Gastroesofágico.
361- La siguiente es causa benigna de elevación del PSA (Antígeno prostático
Especifico), EXCEPTO:
Epididimitis.
362- De los siguientes criterios. ¿Cuál está asociado con la presencia de una
pancreatitis aguda grave?

A Escala de RANSON de 1.
B Escala de APACHE II mayor a 8.
C Escala de BISAP de 1.
D Presencia de 1 criterio de Ranson.

363- Usted atiende a una paciente con cuadro compatible con colecistitis aguda.
Al examen físico tiene fiebre, y Murphy positivo. El utrasonido demuestra una
vesícula grande, de paredes tensas, e inmóviles, sin cálculos en su interior. Su
diagnóstico es colecistitis acalculosa, ¿Cuál de las siguientes condiciones estaría
asociada con esta enfermedad?
A Recurrentes dietas muy hipocalóricas.
B Trauma y quemadura extensa.
C Consumo habitual de fibratos.
D Estados hemolíticos crónicos.

Entre las causas de colecistitis acalculosa segundaria puede ser


politraumatizados, grandes quemados, postquirúrgica (sobre todo en
cirugías de tipo cardiaco o vascular, por su relación con la isquemia arterial),
sepsis severa, postparto, nutrición parenteral total prolongada, diabetes y
ateromatosis.

366- Paciente femenino de 18 años, sin antecedentes patológicos de importancia,


sexualmente activa acude a la consulta por presentar secreción vaginal, al
examen físico se detecta cervicitis con secreción vaginal abundante que es
inodora, no irritante y de color amarillento. ¿Cuál es el diagnóstico y tratamiento
médico adecuado?
A Infección por hongos - Azitromicina.
B Vaginosis bacteriana - Clotrimazol.
C Infección por herpes - Valaciclovir.
D Infección por gonorrea - Doxiciclina.

368- Paciente que cursa 30 semanas de embarazo, acude por presentar disuria,
polaquiuria, tenesmo vesical desde hace dos días. La sospecha de infección de
vías urinarias se confirma con un elemental y microscópico de orina positivo,
¿Cuál de los siguientes esquemas terapéuticos será el más eficaz en este caso?
A Ciprofloxacina por 7 días.
B Nitrofurantoína por 7 días.
C Ampicilina por 7 días.
D Azitromicina por 3 días.

369- Una mujer de 52 años de edad, con diabetes mellitus en tratamiento oral
durante varios años. Desde hace diez días presenta rinorrea bilateral, inicialmente
fluida y después amarillenta. Desde hace tres días tiene fiebre y cefalea frontal.
Acude a consulta por dolor periocular izquierdo. Al examen físico, temperatura
bucal 39°C, FC: 110/min, zona periorbitaria izquierda con edema, eritema y dolor.
La conducta más apropiada será administrar tratamiento
A Sintomático, iniciar tratamiento antibiótico intravenoso y
hospitalizar.
B Ambulatorio sintomático, colocar compresas frías y reevaluar en 48 horas.
C Ambulatorio sintomático más ampicilina vía oral y reevaluar en 48 horas.
D Sintomático, realizar radiografía de senos de la cara e iniciar antibiótico.
370- Adolescente femenina de 14 años de edad, refiere que desde hace 3 días
presenta dolor en su oído derecho, el cual ha ido aumentando progresivamente de
intensidad, además desde la noche de ayer presenta alza térmica a 40oC. Su
médico realiza examen otoscópico, y encuentra una moderada supuración de
color amarillento. En un principio prescribe paracetamol 500 mg y amoxicilina 500
mg cada ocho horas durante tres días. Ante la persistencia de la sintomatología
después de cuarenta y ocho horas, sugiere cambiar el esquema antibiótico. ¿Cuál
sería el antibiótico de elección?
A Azitromicina.
B Cotrimoxazol.
C Ceftriaxona.
D Cefuroxima.
371- El medicamento de primera línea para realizar el tratamiento de tiña corporis
es:
Terbinafina tópica.
374- ¿En que etapa de la sífilis se observa la perforación del paladar blando?
A Sífilis prenatal (congénita
B Sífilis secundaria
C Sífilis terciaria
D Sífilis primaria

375- Se trata de un paciente de 35 años de edad, previamente sano, que se


dedica a la venta informal de quesos artesanales. Acude porque desde hace un
par de semanas tiene sensación de fiebre, decaimiento, sudoración nocturna,
dolor corporal y algo de pérdida de peso. No tiene tos. Al examen físico lo
relevante es la temperatura de 37,9 grados centígrados. En los exámenes de
laboratorio las muestras de BAAR en orina son negativas, tiene leucocitos de
7.000/mm3, con 50% de linfocitos y 45% de neutrófilos de características
normales. Aglutinaciones para brucella son positivas 1:640. Hasta que los
resultados de los cultivos estén listos, ¿Qué antibióticos debería prescribir?
A Vancomicina y gentamicina.
B Dicloxacilina y amikacina.
C Ampicilina y claritromicina.
D Estreptomicina y doxiciclina.
377- A su consulta acude un paciente de 3 años de edad por presentar tos desde
hace 4 días. Al examen físico el paciente se encuentra febril (38 grados C),
frecuencia cardiaca de 95 por minuto, frecuencia respiratoria de 52 por minuto y
saturación de oxígeno 93%. No se evidencia estridor y a la auscultación pulmonar
no presenta disminución de murmullo vesicular ni sibilancias. El diagnóstico del
paciente es:
A Resfriado comun.
B Crup viral.
C Neumonía.
D Bronquitis aguda.

379- Mujer de 27 años, presenta desde hace 11 días, rinorrea purulenta anterior y
posterior, obstrucción nasal bilateral, dolor facial, hiposmia y fiebre, ¿Cuál es la
mejor conducta terapéutica?
A Oximetazolina.
B Azitromicina.
C Cotrimoxazol.
D Loratadina.

380- Carolina de 24 años de edad presenta fiebre de 39°C tres días después de
haber sido sometida a cesárea. Su hijo nació en buenas condiciones, sin signos
de enfermedad y se encuentra en buen estado de salud. Se palpa el fondo uterino
blando y doloroso y se aprecian loquios de mal olor. Al tacto vaginal presenta dolor
en cuello que se incrementa con los movimientos. Se solicita biometría hemática
que reporta: hemoglobina 11 g/dl, leucocitos 12500 (segmentados 75%, linfocitos
25%); análisis de orina normal. ¿Cuál de los siguientes esquemas antibióticos es
el más adecuado para el tratamiento de la endometritis?
A Ampicilina - Sulbactam
B Metronidazol – Clindamicina
C Clindamicina - Gentamicina
D Gentamicina – Ampicilina

381- Identifique cuál de los siguientes signos clínicos nos permite realizar una
diferenciación entre una conjuntivitis gonocócica y no gonocócica:
A Inyección conjuntival.
B Secreción purulenta o mucopurulenta.
C Presencia de úlcera corneal.
D Ojo rojo.

385- Un paciente de 54 años de edad con antecedentes de alcoholismo


importante. Acude a emergencias porque presenta decaimiento y fiebre de 24
horas de evolución. En el examen físico es evidente el mal estado nutricional, pero
no existe un foco infeccioso claro, ni está en estado de choque “shock”. En la
biometría lo llamativo es que tiene 800 leucocitos /mm3, con 30% de linfocitos,
46% de neutrófilos, 20% de monocitos y 4% de células con núcleo en forma de
cayado. Usted decide iniciar un antibiótico como monoterapia. En este paciente,
¿Cuál de los siguientes elige?
A Ampicilina sulbactam.
B Ceftazidima.
C Tobramicina.
D Moxifloxacina.

386- Una niña de 5 años de edad, presenta desde hace 2 días dolor abdominal,
fiebre de 39,5 0C y deposiciones blandas con sangre con una frecuencia de 5 por
día. ¿Adicionalmente a la hidratación oral cuál es la mejor alternativa terapéutica?
A Amoxicilina.
B Cotrimoxazol.
C Dicloxacilina.
D Cefalexina.

390- Varón de 42 años de edad, con antecedentes de asma en la infancia.


Consulta por presentar un cuadro clínico de 6 meses de evolución. Se caracteriza
por hiposmia, epistaxis ocasionales, congestión nasal, rinorrea y dolor facial. En la
rinoscopia anterior se visualiza tumoraciones con aspecto de uva en ambas fosas
nasales, son blandas, de color gris rosado y translúcidas, ¿Cuál es su impresión
diagnóstica?
A Rinitis vasomotora.
B Pólipos nasales.
C Quistes naso alveolares.
D Hipertrofia de cornetes.

También podría gustarte